0% found this document useful (0 votes)
53 views125 pages

4 MAT-101 Math-1 - Differential and Integral Calculas (P1-32)

The document outlines the module specification for Differential & Integral Calculus (MTH-101) in the Mathematics department of the Engineering faculty for the academic year 2023. It includes details on the module's duration, credit value, teaching methodologies, evaluation methods, and fundamental concepts related to real numbers and interval notation. The document also covers the definition of functions and their representation in various forms.

Uploaded by

jardybdit
Copyright
© © All Rights Reserved
We take content rights seriously. If you suspect this is your content, claim it here.
Available Formats
Download as PDF, TXT or read online on Scribd
0% found this document useful (0 votes)
53 views125 pages

4 MAT-101 Math-1 - Differential and Integral Calculas (P1-32)

The document outlines the module specification for Differential & Integral Calculus (MTH-101) in the Mathematics department of the Engineering faculty for the academic year 2023. It includes details on the module's duration, credit value, teaching methodologies, evaluation methods, and fundamental concepts related to real numbers and interval notation. The document also covers the definition of functions and their representation in various forms.

Uploaded by

jardybdit
Copyright
© © All Rights Reserved
We take content rights seriously. If you suspect this is your content, claim it here.
Available Formats
Download as PDF, TXT or read online on Scribd
You are on page 1/ 125

Department: Mathematics

Faculty: Engineering
Full Module Specification

Module Title/Course Name Differential & Integral Calculus

Module Code : MTH -101

Module Level :

Academic Year : 2023

Module Lecturer :

Contact Address:

Counseling Hour: Before or after class or by appointment

Module Credit 3

Pre Requisites :

Co Requisites :

Duration of Module : 6 months

Grading : As outlined in the University policy

Teaching Methodology Class room lecturer, ,discussion, group study,


Class performance, Assignment etc.

Method of Evaluation Attendance =10


Continuous Assessment =10
Term 01 =25
Term 02 =25
Final =30
TOTAL =100
Module 01
Sets
REAL NUMBERS GEOMETRIC REPRESENTATION OF REAL
The following types of numbers are already familiar to the NUMBERS
student: The geometric representation of real numbers as points on a line
Natural numbers: 𝟏; 𝟐; 𝟑; 𝟒; . . . ; also called positive called the real axis, as in the figure below, is also well known
integers, are used in counting members of a set. The symbols to the student. For each real number there corresponds one and
varied with the times, e.g., the Romans used 𝑰, 𝑰𝑰, 𝑰𝑰𝑰, 𝑰𝑽, . .. The only one point on the line and conversely, i.e., there is a one-to-
sum 𝒂 + 𝒃 and product 𝒂 . 𝒃 or 𝒂𝒃 of any two natural numbers one (Figure 1) correspondence between the set of real numbers
𝒂 and 𝒃 is also a natural number. This is often expressed by and the set of points on the line. Because of this we often use
saying that the set of natural numbers is closed under the point and number interchangeably.
operations of addition and multiplication, or satisfies the
closure property with respect to these operations. Negative
integers and zero denoted by −𝟏, −𝟐, −𝟑 and 𝟎, respectively,
arose to permit solutions of equations such as 𝒙 + 𝒃 = 𝒂, Figure 1
where 𝒂 and 𝒃 are any natural numbers. This leads to the
operation of subtraction, or inverse of addition, and we write
(The interchangeability of point and number is by no means
𝒙 = 𝒂 − 𝒃. The set of positive and negative integers and zero
self-evident; in fact, axioms supporting the relation of geometry
is called the set of integers.
and numbers are necessary. The Cantor–Dedekind Theorem is
Rational numbers: or fractions such as , −
𝟐 𝟓
,... arose to fundamental.) The set of real numbers to the right of 0 is called
𝟑 𝟒 the set of positive numbers; the set to the left of 0 is the set of
permit solutions of equations such as 𝒃𝒙 = 𝒂 for all integers a
negative numbers, while 0 itself is neither positive nor negative.
and b, where b 𝟔 ≠ 𝟎. This leads to the operation of division, (Both the horizontal position of the line and the placement of
or inverse of multiplication, and we write 𝒙 = 𝒂/𝒃 or 𝒂 ÷ 𝒃 positive and negative numbers to the right and left, respectively,
where 𝒂 is the numerator and 𝒃 the denominator. are conventions.) Between any two rational numbers (or
The set of integers is a subset of the rational numbers, since irrational numbers) on the line there are infinitely many rational
integers correspond to rational numbers where 𝑏 = 1. (and irrational) numbers. This leads us to call the set of rational
Irrational numbers: such as √𝟐 and 𝝅 are numbers which are (or irrational) numbers an everywhere dense set.
not rational, i.e., they cannot be expressed as 𝒂/𝒃 (called the
quotient of a and b), where 𝒂 and 𝒃 are integers and 𝒃 ≠ 𝟎. Interval Notation
The set of rational and irrational numbers is called the set of
Interval notation is a method to represent an interval on a
real numbers.
number line. In other words, it is a way of writing subsets of
Decimal representation of real numbers: Any real number
the real number line. An interval comprises the numbers lying
can be expressed in decimal form, e.g., 17/10 = 1.7,
between two specific given numbers. For example, the set of
9/100 = 0.09, 1/6 = 0.16666. .. . In the case of a rational
numbers x satisfying 0 ≤ 𝑥 ≤ 5 is an interval that contains
number the decimal expansion either terminates, or if it does
0, 5, and all numbers between 0 and 5.
not terminate, one or a group of digits in the expansion will
1
ultimately repeat, as for example, in = Let us understand the interval notation and different types of
7
0.142857 142857 142 … . In the case of an irrational number intervals in detail using solved examples.
What is Interval Notation?
such as √2 = 1.41423 ... or 𝜋 = 3.14159 ... no such repetition
can occur. We can always consider a decimal expansion as Interval Notation is a way of expressing a subset of real
unending, e.g., 1.375 is the same as 1.37500000 . . . or numbers by the numbers that bound them. We can use this
1.3749999 . . . . To indicate recurring decimals we sometimes notation to represent inequalities. We know an interval
1
place dots over the repeating cycle of digits, e.g., = expressed as 1 < 𝑥 < 5 denotes a set of numbers lying
7
19 between 1 and 5.
0.14287, = 3.16. The decimal system uses the ten digits
6
0,1, 2, . . . ; 9. (These symbols were the gift of the Hindus. They
were in use in India by 600 A.D. and then in ensuing centuries
were transmitted to the western world by Arab traders.) It is
possible to design number systems with fewer or more digits,
e.g. the binary system uses only two digits 0 and 1.

1|Page
Module 01
Sets
Examples of Interval Notation endpoints, −3 and 1. This is expressed using closed interval
notation: [−3,1].
Suppose we want to express the set of real numbers
{𝑥 |−2 < 𝑥 < 5} using an interval. This can be expressed as
interval notation (−2, 5).

The set of real numbers can be expressed as (−∞, ∞). Half-Open Interval

This type of interval includes only one of the endpoints of the


inequality. For example, the set {𝑥 | − 3 ≤ 𝑥 < 1} include
the endpoint −3. This is expressed using half-open interval
notation: [−3,1).
Different Types of Intervals

Intervals can be classified based on the numbers the set


comprises. Some sets include the endpoints specified in the
notation, while some might partially or not include the
endpoints. In general, there are three types of intervals given
as,

Notations for Different Types of Intervals


 Open Interval
We can follow certain rules and symbols to represent the
 Closed Interval
interval notation for different types of intervals. Let us
 Half-Open Interval
understand different symbols that can be used to write a
particular type of interval.

Symbol for Interval Notation

The notations we use for different intervals are:


Open Interval
 [ ]: This is a square bracket that is used when both
the endpoints are included in the set.
This type of interval does not include the endpoints of the
inequality. For example, the set {𝑥 | − 3 < 𝑥 < 1} does not  ( ): This is a round bracket that is used when both the
include the endpoints, −3 and 1. This is expressed using open endpoints are excluded in the set.
interval notation: (−3, 1).  ( ]: This is a semi-open bracket that is used when the
left endpoint is excluded and the right endpoint is
included in the set.
 [ ): This is also a semi-open bracket that is used when
the left endpoint is included and the right endpoint is
excluded in the set.

Number Line Representation of Different Types


Closed Interval of Intervals

This type of interval includes the endpoints of the inequality. Different types of interval notation can be represented on a
For example, the set {𝑥 | − 3 ≤ 𝑥 ≤ 1} include the number line. Look at the handy table that distinguishes
2|Page
Module 01
Sets
between all the types of intervals using their representation on
a number line. Interval
Inequality Number Line
Notation

Type
Interval
Number of 𝑥 ≤ 3 (−∞, 3]
Notatio Inequality
Line Interva
n
l

𝑥 < 5 (−∞, 5)
{𝑥 | 𝑎 < 𝑥 Open
(𝑎, 𝑏)
< 𝑏} Interval

𝑥 ≥ 2 (2, ∞]
{𝑥 | 𝑎 ≤ 𝑥 Closed
[𝑎, 𝑏]
≤ 𝑏} interval

Half- Important Notes on Interval Notation:


[𝑎, ∞) {𝑥 | 𝑥 ≥ 𝑎} Open
Interval  Interval notation is used to express the set of inequalities.
 There are 3 types of interval notation: open interval
Half- closed interval, and half-open interval.
(𝑎, ∞) {𝑥 | 𝑥 > 𝑎} Open  The interval with no infinity symbol is called a bounded
Interval interval.
 The interval containing the infinity symbol is called an
unbounded interval.
Half-
(−∞, 𝑎) {𝑥 | 𝑥 < 𝑎} Open
Interval
Function
Half-
(−∞, 𝑎] {𝑥 | 𝑥 ≤ 𝑎} Open
DEFINITION OF FUNCTION:
Many scientific laws and engineering principles describe how
Interval
one quantity depends on another. This idea was formalized in
1673 by Gottfried Wilhelm Leibniz (see p. xx) who coined the
term function to indicate the dependence of one quantity on
another, as described in the following definition.
Converting Inequality to Interval Notation
1.1 Definition If a variable y depends on a variable x in such a
way that each value of x determines exactly one value of y,
Follow the steps mentioned below to convert an inequality to
then we say that y is a function of x.
interval notation.
Four common methods for representing functions are:
 Numerically by tables
 Graph the solution set of the interval on a number line.
 Geometrically by graphs
 Write the numbers in the interval notation with a smaller
 Algebraically by formulas
number appearing first on the number line on the left.
 Verbally
 If the set is unbounded on the left, use the symbol " − ∞"
1.2 Definition A function f is a rule that associates a unique
and if it is unbounded on right, use the symbol " + ∞".
output with each input. If the input is denoted by 𝑥, then the
output is denoted by 𝑓 (𝑥) (read “𝑓 𝑜𝑓 𝑥”).
Let's take a few examples of inequality and convert them to
In this definition the term unique means “exactly
interval notation.
one.” Thus, a function cannot assign two different outputs to
the same input.

3|Page
Module 01
Sets
If 𝑓 is a real-valued function of a real variable, then the graph
of 𝑓 in the 𝑥𝑦-plane is defined to be the graph of the equation
𝑦 = 𝑓 (𝑥). For example, the graph of the function 𝑓 (𝑥) = 𝑥
is the graph of the equation 𝑦 = 𝑥, shown in Figure 3. That
figure also shows the graphs of some other basic functions that
may already be familiar to you.

Figure 2

INDEPENDENT AND DEPENDENT VARIABLES


For a given input 𝑥, the output of a function 𝑓 is called the
value of 𝑓 at 𝑥 or the image of 𝑥 under 𝑓 . Sometimes we will
want to denote the output by a single letter, say 𝑦, and write
𝑦 = 𝑓 (𝑥)
This equation expresses y as a function of 𝑥; the variable 𝑥 is
called the independent variable (or argument) of 𝑓 , and the
variable 𝑦 is called the dependent variable of 𝑓 . This
terminology is intended to suggest that 𝑥 is free to vary, but Figure 3
that once 𝑥 has a specific value a corresponding value of 𝑦 is
determined. For now, we will only consider functions in which Graphs can provide valuable visual information about a
the independent and dependent variables are real numbers, in function. For example, since the graph of a function 𝑓 in the 𝑥𝑦-
which case we say that 𝑓 is a real-valued function of a real plane is the graph of the equation 𝑦 = 𝑓 (𝑥), the points on the
variable. Later, we will consider other kinds of functions. graph of 𝑓 are of the form (𝑥, 𝑓 (𝑥)); that is, the 𝑦-coordinate
of a point on the graph of 𝑓 is the value of 𝑓 at the corresponding
Example 1 Table 1 describes a functional relationship y = f 𝑥-coordinate (Figure 4). The values of x for which 𝑓 (𝑥) = 0
(x) for which are the 𝑥-coordinates of the points where the graph of f
x 0 1 2 3 intersects the 𝑥-axis (Figure 5). These values are called the
zeros of 𝑓 , the roots of 𝑓 (𝑥) = 0, or the x-intercepts of the
y 3 4 -1 6
graph of 𝑦 = 𝑓 (𝑥).
Table 1

𝑓 (0) = 3, 𝑓 associates 𝑦 = 3 with 𝑥 = 0.


𝑓 (1) = 4 , 𝑓 associates 𝑦 = 4 with 𝑥 = 1.
𝑓 (2) = −1, 𝑓 associates 𝑦 = −1 with 𝑥 = 2.
𝑓 (3) = 6 , 𝑓 associates 𝑦 = 6 with 𝑥 = 3.
Example 2 The equation
𝑦 = 3𝑥 2 − 4𝑥 + 2
has the form 𝑦 = 𝑓 (𝑥) in which the function 𝑓 is given by
the formula
𝑓 (𝑥) = 3𝑥 2 − 4𝑥 + 2
For each input 𝑥, the corresponding output 𝑦 is obtained by Figure 4: The y-coordinate of a point on the graph of y = f(x)
substituting 𝑥 in this formula. For example, is the value of f at the corresponding x-coordinate.
𝑓 (0) = 3(0)2 − 4(0) + 2 = 2,
𝑓 associates 𝑦 = 2 with 𝑥 = 0.
𝑓 (−1.7) = 3(−1.7)2 − 4(−1.7) + 2 = 17.47,
𝑓 associates 𝑦 = 17.47 with 𝑥 = −1.7.
2
𝑓 (√2 ) = 3( √2) − 4 √2 + 2 = 8 − 4 √2, 𝑓
associates 𝑦 = 8 − 4 √2 with 𝑥 = √2.

GRAPHS OF FUNCTIONS

4|Page
Module 01
Sets
expressed as the product of 𝑛 linear factors; with our
restriction to real numbers, it is possible that 2𝑘 of the
roots may be complex. In this case, the 𝑘 factors
generating them will be quadratic. (The corresponding
roots are in complex conjugate pairs.) The polynomial
𝑥 3 − 5𝑥 2 + 11𝑥 − 15 = (𝑥 − 3)(𝑥 2 − 2𝑥 + 5)
illustrates this thought.
2. Algebraic functions are functions 𝒚 =
𝒇(𝒙)satisfying an equation of the form
𝒑𝟎 (𝒙)𝒚𝒏 + 𝒑𝟏 (𝒙)𝒚𝒏−𝟏 + ⋯ + 𝒑𝒏−𝟏 (𝒙)𝒚 (2)
Figure 5: 𝑓 has zeros at 𝑥1 , 0, 𝑥2 , and 𝑥3 + 𝒑𝒏 (𝒙) = 𝟎
where 𝑝0 (𝑥), … 𝑝𝑛 (𝑥) are polynomials in 𝑥.
A function 𝑓 establishes a set of ordered pairs (𝑥, 𝑦)of real If the function can be expressed as the
𝑝(𝑥)
numbers. The plot of these pairs(𝑥, 𝑓(𝑥)) in a coordinate quotient of two polynomials, i.e., where 𝑝(𝑥)and 𝑄(𝑥)
𝑄(𝑥)
system is the graph of 𝑓. The result can be thought of as a are polynomials, it is called a rational algebraic function;
pictorial representation of the function. otherwise it is an irrational algebraic function.
For example, the graphs of the functions described by𝑦 = 3. Transcendental functions are functions which are
𝑥 2 , −1 ≤ 𝑥 ≤ 1, and 𝑦 2 = 𝑥, 0 ≤ 𝑥 ≤ 1, 𝑦 ≥ 1 appear in not algebraic, i.e., they do not satisfy equations of the
Figure 6 form (2)
Note the analogy with real numbers, polynomials
corresponding to integers, rational functions to rational
numbers, and so on.
TRANSCENDENTAL FUNCTIONS
The following are sometimes called elementary
transcendental functions.
1) Exponential function: 𝒇(𝒙) = 𝒂𝒙 , 𝒂 ≠ 𝟎, 𝟏
Figure 6
2) Logarithmic function:𝒇(𝒙) = 𝒍𝒐𝒈𝒂 𝒙, 𝒂 ≠ 𝟎, 𝟏. This
and the exponential function are inverse functions. If
𝒂 = 𝒆 = 𝟐. 𝟕𝟏𝟖𝟐𝟖 …, called the natural base of
TYPES OF FUNCTIONS logarithms, we write𝒇(𝒙) = 𝐥𝐨𝐠 𝒆 𝒙 = 𝒍𝒏𝒙, called
It is worth realizing that there is a fundamental pool of functions the natural logarithm of 𝒙.
at the foundation of calculus and advanced calculus. These are 3) Trigonometric functions (Also called circular
called elementary functions. Either they are generated from a functions because of their geometric interpretation
real variable 𝑥 by the fundamental operations of algebra, with respect to the unit circle):
including powers and roots, or they have relatively simple 𝐬𝐢𝐧 𝒙 , 𝐜𝐨𝐬 𝒙 , 𝐭𝐚𝐧 𝒙 , … 𝒆𝒕𝒄
geometric interpretations. As the title ‘‘elementary functions’’ THE VERTICAL LINE TEST
suggests, there is a more general category of functions (which, Not every curve in the 𝑥𝑦-plane is the graph of a function. For
in fact, are dependent on the elementary ones). Some of these example, consider the curve in Figure 7 , which is cut at two
will be explored later in the book. The elementary functions are distinct points, (𝑎, 𝑏) and (𝑎, 𝑐), by a vertical line. This curve
described below. cannot be the graph of 𝑦 = 𝑓 (𝑥) for any function 𝑓 ;
1. Polynomial functions have the form otherwise, we would have
𝑓(𝑥) = 𝑎0 𝑥 𝑛 + 𝑎1 𝑥 𝑛−1 + ⋯ + 𝑎𝑛−1 𝑥 + 𝑎𝑛 (1) 𝑓 (𝑎) = 𝑏 and 𝑓 (𝑎) = 𝑐
where 𝑎0 . .. 𝑎𝑛 are constants and n is a positive
integer called the degree of the polynomial if 𝑎0 ≠ 0.
The fundamental theorem of algebra states that in the
field of complex numbers every polynomial equation
has at least one root. As a consequence of this theorem,
it can be proved that every 𝑛𝑡ℎ degree polynomial has
𝑛 roots in the complex field. When complex numbers
are admitted, the polynomial theoretically may be
5|Page
Module 01
Sets
properties of absolute value If 𝑎 and 𝑏 are real numbers, then
(a) | − 𝑎| = |𝑎| A number and its negative have the same
absolute value.
(b) |𝑎𝑏| = |𝑎| |𝑏| The absolute value of a product is
the product of the absolute values.
𝑎 |𝑎|
(c) | | = |𝑏| , 𝑏 ≠ 0 The absolute value of a ratio is the
𝑏
ratio of the absolute values.
(d) |a + b|≤|a|+|b| The triangle inequality
The graph of the function 𝑓 (𝑥) = |𝑥| can be obtained by
Figure 7 :This curve cannot be the graph of a function. graphing the two parts of the equation
−𝑥, 𝑥<0
which is impossible, since 𝑓 cannot assign two different 𝑦={
𝑥, 𝑥≥0
values to 𝑎. Thus, there is no function 𝑓 whose graph is the separately. Combining the two parts produces the V-shaped
given curve. This illustrates the following general result, graph in Figure 9. Absolute values have important
which we will call the vertical line test. relationships to square roots. To see why this is so, recall from
algebra that every positive real number 𝑥 has two square roots,
The vertical line test A curve in the 𝑥𝑦-plane is the graph of
one positive and one negative. By definition, the symbol √𝑥
some function 𝑓 if and only if no vertical line intersects the
denotes the positive square root of 𝑥. Care must be exercised
curve more than once.
in simplifying expressions of the form √𝑥 2 , since it is not
Example 3: The graph of the equation always true that √𝑥 2 = 𝑥. This equation is correct if 𝑥 is
nonnegative, but it is false if 𝑥 is negative. For example, if
𝑥 2 + 𝑦 2 = 25
𝑥 = −4, then
is a circle of radius 5 centered at the origin and hence there are
vertical lines that cut the graph more than once (Figure 8).
Thus this equation does not define 𝑦 as a function of 𝑥.

Figure 9

√𝑥 2 = √−42 = √16 = 4 ≠ 𝑥

Figure 8 A statement that is correct for all real values of 𝑥 is


√𝑥 2 = |𝑥|
PIECEWISE-DEFINED FUNCTIONS
The absolute value function 𝑓 (𝑥) = |𝑥| is an example of a
THE ABSOLUTE VALUE FUNCTION function that is defined piecewise in the sense that the formula
Recall that the absolute value or magnitude of a real number for f changes, depending on the value of 𝑥.
𝑥 is defined by Example 4 Sketch the graph of the function defined piecewise
−𝑥, 𝑥<0 by the formula
|𝑥| = {
𝑥, 𝑥≥0 0, 𝑥 ≤ −1
The effect of taking the absolute value of a number is to strip 𝑓(𝑥) = {√1 − 𝑥 2 , −1<𝑥 <1
away the minus sign if the number is negative and to leave the 𝑥, 𝑥≥1
number unchanged if it is nonnegative. Thus,
4 4
|5| = 5, |− | = , |0| = 0
7 7
However, for convenience we provide the following summary
of its algebraic properties.
6|Page
Module 01
Sets
Even and Odd Functions:
1. A function given by 𝑦 = 𝑓(𝑥) is even if, for each x in the
domain,
𝑓(−𝑥) = 𝑓(𝑥).
Example: f (x)  x 2
2. A function given by y = f(x) is odd if, for each x in the
domain,
𝑓(−𝑥) = − 𝑓(𝑥).
Figure 10 Example: f (x)  x 3
Solution: The formula for f changes at the points 𝑥 = −1
and 𝑥 = 1. (We call these the breakpoints for the formula.) A Problem 1: Prove that 𝑓(𝑥) = 𝑠𝑖𝑛 2 𝑥 × 𝑐𝑜𝑠 6 𝑥 is an even
good procedure for graphing functions defined piecewise is to function.
graph the function separately over the open intervals Proof : Given, 𝑓(𝑥) = 𝑠𝑖𝑛2 𝑥 × 𝑐𝑜𝑠 6 𝑥
determined by the breakpoints, and then graph 𝑓 at the Replacing 𝑥 𝑏𝑦 − 𝑥, we get
breakpoints themselves. For the function f in this example the 𝑓(−𝑥) = {𝑠𝑖𝑛(−𝑥)}2 × {cos(−𝑥)6 }
graph is the horizontal ray 𝑦 = 0 on the interval (−∞, −1], it = 𝑠𝑖𝑛2 𝑥 × 𝑐𝑜𝑠 6 𝑥
is the semicircle 𝑦 = √1 − 𝑥 2 on the interval (−1, 1), and it = 𝑓(𝑥)
is the ray 𝑦 = 𝑥 on the interval [1, +∞). The formula for f Given function is an even function.
Problem 2: Prove that 𝑓(𝑥) = 𝑠𝑖𝑛 3 𝑥 × 𝑐𝑜𝑠 6 𝑥 is an odd
specifies that the equation 𝑦 = 0 applies at the breakpoint −1
function.
[so 𝑦 = 𝑓 (−1) = 0], and it specifies that the equation 𝑦 =
Proof : Given, 𝑓(𝑥) = 𝑠𝑖𝑛3 𝑥 × 𝑐𝑜𝑠 6 𝑥
𝑥 applies at the breakpoint 1 [so 𝑦 = 𝑓 (1) = 1]. The graph
of f is shown in Figure 10. Replacing 𝑥 𝑏𝑦 − 𝑥, we get
𝑓(−𝑥) = {𝑠𝑖𝑛(−𝑥)}3 × {cos(−𝑥)}6 }
Example 5 Increasing the speed at which air moves over a = {−𝑠𝑖𝑛3 𝑥 × 𝑐𝑜𝑠 6 𝑥}= −𝑓(𝑥)
person’s skin increases the rate of moisture evaporation and Given function is an odd function.
makes the person feel cooler. (This is why we fan ourselves in Problem 3: Prove that 𝑓(𝑥) = ln(𝑥 + √𝑥 2 + 1)is an odd
hot weather.) The wind chill index is the temperature at a function of x.
wind speed of 4 𝑚𝑖/ℎ that would produce the same sensation
Proof : Given, 𝑓(𝑥) = ln(𝑥 + √𝑥 2 + 1)
on exposed skin as the current temperature and wind speed
Now
combination. An empirical formula (i.e., a formula based on
𝑓(𝑥) + 𝑓(−𝑥)
experimental data) for the wind chill index 𝑊 at 320 𝐹 for a
wind speed of 𝑣 𝑚𝑖/ℎ is = ln(𝑥 + √𝑥 2 + 1) + ln (−𝑥 + √(−𝑥)2 + 1)
= ln(𝑥 + √𝑥 2 + 1) + ln(−𝑥 + √𝑥 2 + 1)
−32, 0≤𝑣≤3
𝑊={
55.628 − 22.07𝑣 0.16 , 3<𝑣 = ln(𝑥 + √𝑥 2 + 1) . ln (−𝑥 + √(−𝑥)2 + 1)
= ln(√𝑥 2 + 1 + 𝑥) . (√𝑥 2 + 1 − 𝑥)
= ln(𝑥 2 + 1 − 𝑥 2 )
= ln 1=0
So,
𝑓(𝑥) +𝑓(−𝑥)=0
𝑓(𝑥) = − 𝑓(−𝑥)
Given function is an odd function.
Try Yourself:
Classify the function is even or odd:
Figure 11 1. f(𝑥) = 𝑠𝑖𝑛4 𝑥 + 𝑐𝑜𝑠 4 𝑥.
2. 𝑓(𝑥) = 1 + 3𝑥 2 − 𝑥 4 .
A computer-generated graph of W(v) is shown in Figure 11. 3. 𝑓(𝑥) = 2𝑥 5 − 3𝑥 2 + 2 .

7|Page
Module 01
Sets
Periodic function: domain of f, and the set of outputs (𝑦-values) that result when
A function 𝑦 = 𝑓(𝑥) is said to be Periodic function of period 𝑥 varies over the domain is called the range of f.
T if it satisfies the condition 𝑓(𝑥 + 𝑇) = 𝑓(𝑥), where T is the
least positive real number. Question: Find the domain and range of the following
Example: sin 𝑥 is a Periodic function of 2𝜋 . equations
1 1
That is , Periodic function is =
𝑓𝑢𝑛𝑑𝑎𝑚𝑒𝑛𝑡𝑎𝑙 𝑝𝑒𝑟𝑖𝑜𝑑 𝑜𝑓 𝑡ℎ𝑒 𝑓𝑢𝑛𝑡𝑖𝑜𝑛 (𝑖) 𝑓(𝑥) = (𝑖𝑖) 𝑓(𝑥) =
𝑥−2 2𝑥+1
𝐵×𝑛
𝑥−3 𝑥 2𝑥+5
(𝑖𝑖𝑖)𝑓(𝑥) = (𝑖𝑣) 𝑓(𝑥) = (v) f (𝑥) = ,
2𝑥+1 𝑥+1 𝑥−5
Problem 1: Find the Periodic of the function f(𝑥) = 𝑥 2 −4
(vi) f (𝑥) = .
𝑠𝑖𝑛4 (2𝑥 + 5) + 𝑐𝑜𝑠 4 (3𝑥 + 4). 𝑥−2

Solution: Given, f(𝑥) = 𝑠𝑖𝑛4 (2𝑥 + 5) + 𝑐𝑜𝑠 4 (3𝑥 + 4) Solution:


1
Now the Periodic of the function 𝑠𝑖𝑛4 (2𝑥 + 5) is = (i) 𝑓(𝑥) =
𝑥−2
𝑝𝑒𝑟𝑖𝑜𝑑 𝑜𝑓 𝑡ℎ𝑒 sin 𝑥 𝑓(𝑥) gives the real values for all values of 𝑥
𝐶𝑜𝑒𝑓𝑓𝑖𝑐𝑖𝑒𝑛𝑡 𝑡𝑓 𝑥(𝐵)×𝑝𝑜𝑤𝑒𝑟𝑜𝑓 sin 𝑥( 𝑛) except 𝑥 = 2.
= So, 𝐷𝑓 = ℝ − {2}
2𝜋 𝜋
= . Again,
2×4 4
Again, the Periodic of the function 𝑐𝑜𝑠 4 (3𝑥 + 4) is = 1
y  f ( x) 
𝑝𝑒𝑟𝑖𝑜𝑑 𝑜𝑓 𝑡ℎ𝑒 sin 𝑥
x2
𝐶𝑜𝑒𝑓𝑓𝑖𝑐𝑖𝑒𝑛𝑡 𝑡𝑓 𝑥(𝐵)×𝑝𝑜𝑤𝑒𝑟𝑜𝑓 sin 𝑥( 𝑛)
1
=  x2
2𝜋 𝜋 y
= .
3×4 6
𝜋 𝜋 𝜋 1
Here the LCM of
4
and
6
is
2
.
 x  2
Therefore f(𝑥) is a periodic function with period y
𝜋
. x gives real values for all real values of y except
2
y  0.
𝑅𝑓 = ℝ − {0}
1
Problem 2: Find the Periodic of the function f(𝑥) = (ii) 𝑓(𝑥) =
2𝑥+1
𝑠𝑖𝑛(2𝑥 + 5) + 𝑡𝑎𝑛(3𝑥 + 7). 𝑓(𝑥) gives the real values for all values of 𝑥
1
Solution: Given, f(𝑥) = 𝑠𝑖𝑛(2𝑥 + 5) + 𝑡𝑎𝑛(3𝑥 + 7) except 𝑥 = − .
2
Now the Periodic of the function 1
𝑝𝑒𝑟𝑖𝑜𝑑 𝑜𝑓 𝑡ℎ𝑒 sin 𝑥 2𝜋
So, 𝐷𝑓 = ℝ − {− }
2
𝑠𝑖𝑛(2𝑥 + 5) is = = =𝜋. Again,
𝐶𝑜𝑒𝑓𝑓𝑖𝑐𝑖𝑒𝑛𝑡 𝑡𝑓 𝑥(𝐵)×𝑝𝑜𝑤𝑒𝑟𝑜𝑓 sin 𝑥( 𝑛) 2×1
1
y  f ( x) 
Again, the Periodic of the function
𝑝𝑒𝑟𝑖𝑜𝑑 𝑜𝑓 𝑡ℎ𝑒 𝑡𝑎𝑛𝑥 𝜋 𝜋
2x 1
𝑡𝑎𝑛(3𝑥 + 7) is = = . 1
 2x 1 
𝐶𝑜𝑒𝑓𝑓𝑖𝑐𝑖𝑒𝑛𝑡 𝑡𝑓 𝑥(𝐵)×𝑝𝑜𝑤𝑒𝑟𝑜𝑓𝑡𝑎𝑛𝑥( 𝑛) 3×1 3

𝜋
y
Here the LCM of 𝜋 and is 𝜋 . Therefore f(𝑥) is a periodic
3
11 
function with  x    1
period. 2 y 
x gives real values for all real values of y except
Try Yourself: y  0.
Find the period of the following function: So, 𝑅𝑓 = ℝ − {0}
1. f(𝑥) = 𝑠𝑖𝑛(2𝑥) + 𝑐𝑜𝑠(3𝑥). 𝑥−3
(iii) 𝑓(𝑥) =
2. f(𝑥) = 𝑡𝑎𝑛(5𝑥) + 𝑠𝑖𝑛(3𝑥) . 2𝑥+1

1
𝑓(𝑥) gives the real values for all values of 𝑥
3. f(𝑥) = 𝑠𝑖𝑛(2𝑥) + 𝑐𝑜𝑠 ( 𝑥). 1
2 except 𝑥 = − .
2
Domain and Range: If 𝑥 and 𝑦 are related by the equation 𝑦 = So, 𝐷𝑓 = ℝ − {− }
1
2
𝑓(𝑥), then the set of all allowable inputs (𝑥-values) is called the
Again,

8|Page
Module 01
Sets
x 3 𝑦(𝑥 − 2) = 𝑥 2 − 4
y  f ( x) 
2x 1 (𝑥−2)(𝑥+2)
 2 xy  y  x  3 𝑦= (𝑥−2)

 x  2 y  1  ( y  3)
𝑦 = (𝑥 + 2)
∴ 𝑥 = (𝑦 − 2)
( y  3) 𝑓(𝑥) gives the real values for all values of 𝑦
x
 2 y  1 except 𝑦 = 2.
So, 𝐷𝑓 = ℝ − {2}

x gives real values for all real values of y except Inverse function:
Let f be a function whose domain is the set X, and whose image
1
y . (range) is the set Y. Then f is invertible if there exists a function
2 g with domain Y and image X, with the property: If f is
1
So, 𝑅𝑓 = ℝ − { } invertible, the function g is unique, which means that there is
2
𝑥 exactly one function g satisfying this property (no more, no
(iv) 𝑓(𝑥) =
𝑥+1 less). That function g is then called the inverse of f, and is
𝑓(𝑥) gives the real values for all values of 𝑥 usually denoted as f −1.
except 𝑥 = −1.
Example: If f (x)  2 x  3 then the inverse function is f-1(x)
So, 𝐷𝑓 = ℝ − {−1}
= (x+3)/2.
Again,
x
y  f ( x)  Problem: Find the inverse of the function f (x)  2 x  3
x 1
 xy  y  x Solution: Let, y  f (x)  2x 3
y
x  x  f 1 (y)
y 1
y  2x  3
x gives real values for all real values of y except
y  3  2x
y  1.
So, 𝑅𝑓 = ℝ − {1}
2x  y  3
y3
2𝑥+5
x
(v) Given, f (𝑥) = 2
𝑥−5
𝑓(𝑥) gives the real values for all values of 𝑥 y3
 f 1 (y) 
except 𝑥 = 5. 2
So, 𝐷𝑓 = ℝ − {5} x3
Again,  f 1 (x) 
2𝑥+5
2
𝑦 = 𝑓(𝑥) =
𝑥−5
𝑥𝑦 − 5𝑦 = 2𝑥 + 1
𝑥(𝑦 − 2) = 5𝑦 + 1
5𝑦+1
∴𝑥=
𝑦−2
x gives real values for all real values of y except 𝑦 = 2 . .
𝑅𝑓 = ℝ − {2}.
𝑥 2 −4
(vi) Given, f(𝑥) =
𝑥−2
𝑓(𝑥) gives the real values for all values of 𝑥
except 𝑥 = 2.
So, 𝐷𝑓 = ℝ − {2}
𝑥 2 −4
𝑦 = 𝑓(𝑥) =
𝑥−2
𝑥𝑦 − 2𝑦 = 𝑥 2 − 4
9|Page
Module 01
Sets
Problem: Find the inverse of the functions,
i  f  x   2x  5
(ii ) f ( x)  x 2  3
4x  7
(iii ) f ( x) 
2x  4
(iv) f ( x)  1  x 2
Solution:
i  Let y =f  x   2 x  5
 2x  y  5
y5
x
2
y 5
 f 1  y  
2
x  5
 f 1  x  
2
 ii  Let y =f  x   x 2  3
 x2  y  3
x y3
 f 1  y   y  3
 f 1  x   x  3 when x  3
4x  7
 iii  Let y =f  x  
2x  4
 2 xy  4 y  4 x  7
 x(2 y  4)  4 y  7
4y  7
x
2y  4
4y  7
 f 1  y  
2y  4
4x  7
 f 1  x  
2x  4
 iv  Let y =f  x   1  x 2
 y 2  1  x2
 x2  1  y 2
 x  1 y2
 f 1  y   1  y 2
 f 1  x   1  x 2

10 | P a g e
Module-02
Limit & Continuity
𝑥
Limit of function: If the values of 𝑓(𝑥) become = (𝑥+2)
arbitrarily close to a single number 𝑙 as the values 𝑥
lim 𝑓(𝑥) = lim
of a variable 𝑥 approaches to 𝑐 from both sides of 𝑥→2 𝑥→2 𝑥+2
2 1
c (Right and Left) then 𝑙 is called the limit of the =4=2
function𝑓(𝑥). It is denoted by lim 𝑓(𝑥) = 𝑙. Note: We were able to cancel (𝑥 − 2) above
𝑥→𝑐
𝑥 2 −9 because 𝑥 ≠ 2 and (𝑥 − 2) ≠ 0.
For Example, 𝑓(𝑥) = 𝑥−3
32 −9
When 𝑥 = 3, 𝑓(𝑥) = ONE-SIDED LIMITS
3−3
0 Right-hand Limit
=0
0
If 𝑓(𝑥) becomes arbitrarily close to a single
The value of is meaningless and is undefined number 𝐿 as 𝑥 approaches 𝑎 on a small interval
0
since it is not possible to divide any value by 0. (𝑐, 𝑏) immediately to the right of 𝑐 then 𝐿 is called
0 ∞ −∞
Note : 0 , ∞ , −∞ , (∞, −∞) are called indeterminate. the right hand limit of the function and denoted by
𝐿𝑡
When 𝑥 approaches 3, for example, 𝑥 = 3.001 or 𝑥→𝑐 + 𝑓(𝑥) = 𝐿
2.999, 𝑥 − 3 is then not equal to 0.Thus 𝑓(𝑥) =
𝑥 2 −9 Left-hand Limit
𝑥−3 If 𝑓(𝑥) becomes arbitrarily close to a single
(𝑥+3)(𝑥−3)
= number 𝐿 as 𝑥 approaches 𝑎 on a small interval
(𝑥−3)
=𝑥+3 (𝑎, 𝑐) immediately to the left of 𝑐 then 𝐿 is called
This implies that when 𝑥 approaches 3, 𝑓(𝑥) the left hand limit of the function and denoted by
𝐿𝑡
approaches 6. 𝑥→𝑐 − 𝑓(𝑥) = 𝐿
Now study carefully the following computations Some standard limits:
𝑠𝑖𝑛𝑥 𝑥 𝑎
which will help to clarify the points mentioned: (𝑖) 𝐿𝑡 = 1 (𝑖𝑖) 𝐿𝑡 (1 + 𝑎)𝑥 = 𝑒
𝑥→0 𝑥 𝑥→0
When 𝑥 = 3.1 , 𝑓(𝑥) = 6.1 𝑡𝑎𝑛𝑥 𝑎𝑥 −1
(𝑖𝑖𝑖) 𝐿𝑡 =1 (𝑖𝑣) 𝐿𝑡 = Ine 𝑎
When 𝑥 = 3.01 , 𝑓(𝑥) = 6.01 𝑥→0 𝑥 𝑥→0 𝑥
When 𝑥 = 3.001 , 𝑓(𝑥) = 6.001 Limit Rules:
𝐿𝑡 𝐿𝑡
When 𝑥 = 3.0001 , 𝑓(𝑥) = 6.0001 If 𝑥→+∞ 𝑓(𝑥) and 𝑥→+∞ 𝑔(𝑥) exist, then for
Notice that by allowing 𝑥 to approach sufficiently constants 𝑎 and 𝑏:
close to 3, 𝑓(𝑥) gets closer and closer to 6. Power rule:
𝐿𝑡 𝒏 𝐿𝑡 𝒏
We write 𝑥 → 3 to represent “𝑥 approaches 3”. 𝑥→+∞ [𝑓(𝑥)] = [ 𝑥→+∞ 𝑓(𝑥)]
𝐿𝑡
The limit is denoted by lim 𝑓(𝑥) = 6. Linearity rule: 𝑥→+∞ [𝑎𝑓(𝑥) + 𝑏𝑔(𝑥)] =
𝑥→3 𝐿𝑡 𝐿𝑡
𝑥 2 −2𝑥 𝑎 𝑥→+∞ 𝑓(𝑥) + 𝑏 𝑥→+∞ 𝑔(𝑥)
Example : Find the limit of 𝑓(𝑥) = when 𝐿𝑡
𝑥 2 −4 Product rule: 𝑥→+∞ [𝑓(𝑥)𝑔(𝑥)] =
𝑥 → 2. 𝐿𝑡 𝐿𝑡
𝑥→+∞ 𝑓(𝑥)][ 𝑥→+∞ 𝑔(𝑥)]
𝑥 2 −2𝑥 𝑓(𝑥) 𝐿𝑡 𝑓(𝑥)
Solution: f(𝑥) = Quotient rule: 𝐿𝑡 = 𝑥→+∞
𝑥 2 −4 𝑔(𝑥) 𝐿𝑡 𝑔(𝑥)
𝑥→+∞ 𝑥→+∞
22 −2(2) 𝐿𝑡
When 𝑥 = 2, f(𝑥) = if 𝑥→+∞ 𝑔(𝑥) ≠ 0.
22 −4
0 𝑓(𝑥)
=0 Rules to determine the value of 𝐿𝑡 :
𝑥→∞ 𝑔(𝑥)
0 (𝑖) Divide the numerator and denominator of the
Since f(𝑥) = 0 , f(𝑥) is not defined at 𝑥 = 2.
𝑥(𝑥−2) fraction by highest power of x present in the
However, when 𝑥 → 2, f(𝑥) = (𝑥+2)(𝑥−2) function.
11 | P a g e
Module-02
Limit & Continuity
1 1
(𝑖𝑖) Putting 𝑥 → ∞, then have , … … … → 0. 𝑥 2 − 6𝑥 + 9
𝑥 𝑥2 (𝑖𝑣) lim
Question: Prove that, 𝑥→3 𝑥−3
Since 3 is a zero of both the numerator and
𝑥−1 1
(𝑖) lim 2 = denominator. They share a common factor of𝑥 − 3.
𝑥→1 𝑥 + 2𝑥 − 3 4
𝑥+1 The limit can be obtained as follows:
(𝑖𝑖) lim =1 𝑥 2 − 6𝑥 + 9
𝑥→0 2𝑥 + 1
lim
𝑥2 − 4 𝑥→3 𝑥−3
(𝑖𝑖𝑖) lim =4 (𝑥 − 3)2
𝑥→2 𝑥 − 2
= lim
𝑥 2 − 6𝑥 + 9 𝑥→3 𝑥 − 3
(𝑖𝑣) lim =0 = lim (𝑥 − 3)
𝑥→3 𝑥−3 𝑥→3
Solution: =3−3
𝑥−1 = 0.
(𝑖) lim 2
𝑥→1 𝑥 + 2𝑥 − 3
𝑥−1 Evaluate: lim
1
x x0
 1 x  1 x 
= lim 2
𝑥→1 𝑥 + 3𝑥 − 𝑥 − 3

= lim
𝑥−1 Solution: lim
1
x0 x
 1 x  1 x 
  
𝑥→1 𝑥(𝑥 + 3) − 1(𝑥 + 3)
𝑥−1 1 x  1 x 1 x  1 x
= lim
 
= lim
𝑥→1 (𝑥 + 3)(𝑥 − 1) x0
x 1 x  1 x
1
   
= lim 2 2
𝑥→1 𝑥 + 3 1 x  1 x
1

 1 x 
= 4. = lim
x a
x 1 x 
𝑥+1
(𝑖𝑖) lim 1  x  (1  x)
𝑥→0 2𝑥 + 1
= lim
=
0+1 x 0 x( 1  x  1  x )
2∗0+1 1 x 1 x
1 = lim
=
0+1
x 0 x( 1  x  1  x )
1 2x
= = lim
1 x 0 x( 1  x  1  x )
= 1.
2
𝑥2 − 4 = lim
(𝑖𝑖𝑖) lim x 0 1  x  1  x
𝑥→2 𝑥 − 2
Since 2 is a zero of both the numerator and 2
denominator. They share a common factor of𝑥 − 2. =
1 0  1 0
The limit can be obtained as follows:
2
𝑥2 − 4 =
lim 11
𝑥→2 𝑥 − 2
(𝑥 + 2)(𝑥 − 2) 2
= lim 
𝑥→2 𝑥−2 2
= lim (𝑥 + 2) 1
𝑥→2
=2+2
= 4.
12 | P a g e
Module-02
Limit & Continuity

Problem 1: Evaluate Since x2 is the highest power of x in the


a. lim x 2  2 x  4 denominator of our function, we have
x 1

x2  9 2 x 2  7 lim  2 x2  7   x2
lim  x 
b. lim
x 3 x  3
x  6  3 x 2 lim  6  3x 2   x 2
x 
Solution:  7
lim  2  2 
a. lim x 2  2 x  4  (1)2  2(1)  4  5 x 
 x 
x 1 
 6 
x 2  9 lim x2  9 lim  2  3 
b. lim  x 3 x  x
 
x 3 x  3 lim x  3
x 3 2  0 2
 

0 03 3
0 Definition of Piecewise Function:
When you get 0/0 you have what is called an in In mathematics, a piecewise-defined function is a
determinant form and you must try other function defined by multiple sub-functions, where
techniques to determine the limit. In this case, each sub-function applies to a different interval in
factor both the numerator and denominator and the domain
cancel common terms to remove the zero in the Limits of Piecewise- Defined Functions:
denominator. Then, apply the limit rules to the 𝐿𝑡
Example 1: Find 𝑥→0 𝑓(𝑥), where 𝑓(𝑥) =
simplified expression. 𝑥 + 5 𝑖𝑓 𝑥 > 0
{
x2  9 ( x  3)( x  3) 𝑥 𝑖𝑓 𝑥 < 0
lim  lim  lim x  3 = 𝐿𝑡 −
𝑓(𝑥) = 𝑥→0 𝐿𝑡 −
𝑥=0
x 3 x  3 x 3 ( x  3) x 3 Solution 𝑥→0
3+3=6 𝑓(𝑥) = 𝑥 to the left of 0.
𝐿𝑡 𝐿𝑡
𝑥→0+ 𝑓(𝑥) = 𝑥→0+ (𝑥 + 5) = 5
2 x2  7
Problem 2 : Evaluate lim 𝑓(𝑥) = 𝑥 + 5 to the right of 0.
x  6  3 x 2
The left-hand and right-hand limits are not the
2 x 2  7 lim 2 x2  7  same, we conclude that 𝑥→0 𝐿𝑡
𝑓(𝑥) does not exist.
Solution lim  x 

x  6  3 x 2 lim 6  3x 2  𝐿𝑡
Example 2: Find 𝑥→0 𝑔(𝑥), where 𝑔(𝑥) =
x 

  
𝑥+1 𝑖𝑓 𝑥 > 0
or or are all also known as in { 2
   𝑥 +1 𝑖𝑓 𝑥 < 0
determinant forms. When this form occurs when Solution
𝐿𝑡 𝐿𝑡 2
finding limits at infinity (or negative infinity) with 𝑥→0− 𝑔(𝑥) = 𝑥→0− (𝑥 + 1) = 0 + 1 = 1
𝐿𝑡 𝐿𝑡
rational functions, divide every term in the 𝑥→0+ 𝑔(𝑥) = 𝑥→0+ (𝑥 + 1) = 0 + 1 = 1
numerator and denominator by the highest power The left-hand and right-hand limits are the same,
𝐿𝑡
of x in the denominator to determine the limit. we conclude that 𝑥→0 𝑔(𝑥) exists.

13 | P a g e
Module-02
Limit & Continuity

When 𝑥 = 0 then𝑓(0) = 𝑎. Therefore,


Continuity lim+ 𝑓(𝑥) = lim− 𝑓(𝑥) = 𝑓(0) .
Continuous Function: A function 𝑓(𝑥) is said to 𝑥→0 𝑥→0

be continuous in an interval (𝑎, 𝑏) if it is So 𝑓(𝑥) is continuous at = 0 .


continuous at every point of the interval.  x  2 when x < 2
Example-3: If f ( x)   2
 x -1 when x  2
Definition: A function f(x) is continuous at x = Test the continuity of the function 𝑓(𝑥) at 𝑥 = 2.
a, if all of the following are true: Solution: When 𝑥 = 2 then lim+ 𝑓(𝑥) =
𝑥→2
1. f(a) is defined
lim (𝑥 2 − 1)
2. lim f ( x) exists 𝑥→2+
x a
= 4−1 =3
3. lim f ( x)  f (a) And lim− 𝑓(𝑥) = lim− (𝑥 + 2)
x a
𝑥→2 𝑥→2
x
For example, e ,sin x,cos x are all continuous = 2+2 =4
When 𝑥 = 2 then𝑓(2) = 3. Therefore,
function.
When dose a function to be continuous? lim+ 𝑓(𝑥) ≠ lim− 𝑓(𝑥) ≠ 𝑓(0) .
𝑥→2 𝑥→2
lim f (x)  lim f (x)  f(a) So 𝑓(𝑥) is discontinuous at = 2 .
When x a  x a

 R.H .L  L.H .L  functional 2 x  1, x  2


Problem 4: Show that f ( x)   is
Discontinuous Function: If 𝑓(𝑥) is not 4 , x=2
continuous at 𝑥 = 𝑎 then it is said to be discontinuous at x = 2.
discontinuous. Solution: Since f(2) = 4 and
 x when x  1 lim f ( x)  lim (2x  1)  5  f (2) ,
Example-1: If f ( x)   x 2 x 2

2x-1 when x>1 so f(x) is discontinuous at x = 2.


Test the continuity of the function 𝑓(𝑥)at 𝑥 = 1. Problem 5: Let the function f(x) be defined for all
Solution: values of x by
When 𝑥 = 1 then 
 x  1, for x  0
2

f ( x)   2
Lim+ 𝑓(𝑥) = lim+(2𝑥 − 1) 
𝑥→1 𝑥→1  x  1, for x  0
= 2.1 − 1 = 1 Draw the graph of the function f(x) and test the
And lim− 𝑓(𝑥) = lim− 𝑥 = 1 continuity from the graph.
𝑥→1 𝑥→1
When 𝑥 = 1 then𝑓(1) = 1. Therefore,
lim+ 𝑓(𝑥) = lim− 𝑓(𝑥) = 𝑓(1) .
𝑥→1 𝑥→1
So 𝑓(𝑥) is continuous at = 1 .
a  x when x  0
Example-2: If f ( x)  
a  x when x <0
Test the continuity of the function 𝑓(𝑥) at 𝑥 = 0.
Solution:
When 𝑥 = 0 then
lim+ 𝑓(𝑥) = lim+(𝑎 + 𝑥) = 𝑎 + 0 = 𝑎
𝑥→0 𝑥→0
And The function f(x) is discontinuous at x = 0
lim− 𝑓(𝑥) = lim−(𝑎 − 𝑥) = 𝑎 − 0 = 𝑎
𝑥→0 𝑥→0

14 | P a g e
Module-02
Limit & Continuity

Because lim f ( x)  1 and lim 𝑓(𝑥) = 1 and exists. If f is differentiable at each point of the open
h 0  ℎ→0+
interval (𝑎, 𝑏), then we say that it is differentiable on
𝑓(0) = 1. (𝒂, 𝒃), and similarly for open intervals of the form
In this case lim f ( x) does not exist. (𝑎, +∞), (−∞, 𝑏), and (−∞, +∞). In the last case we
h0
say that f is differentiable everywhere.
Problem 6: Determine the value of x at which the
1
function f ( x)  is continuous. Geometrically, a function 𝑓 is differentiable at 𝑥0 if the
2 x graph of f has a tangent line at 𝑥0 . Thus, f is not
Solution. The domain of the function is (, 2) . The differentiable at any point 𝑥0 where the secant lines
function is not defined at x = 2. Thus the function is from 𝑃(𝑥0 , 𝑓 (𝑥0 )) to points 𝑄(𝑥, 𝑓 (𝑥)) distinct from
continuous for all values of x in the interval (, 2) . P do not approach a unique nonvertical limiting
See the graph of f(x). position as 𝑥 → 𝑥0 .

Above figure illustrates two common ways in which a


function that is continuous at 𝑥0 can fail to be
differentiable at 𝑥0 . These can be described informally
Possible Questions: as
Define limit and continuity.  corner points
Let the function f(x) be defined for all values of x by
 points of vertical tangency

 x  1,
2
for x  0 At a corner point, the slopes of the secant lines have
f ( x)  

 x  1, for x  0
2
different limits from the left and from the right, and
Draw the graph of the function f(x) and test the hence the two-sided limit that defines the derivative
continuity from the graph. does not exist Figure below.
2 x2  7
lim
Evaluate x 6  3x
2

lim x 2  2 x  4
x 1

DIFFERENTIABILITY:
It is possible that the limit that defines the derivative of
a function 𝑓 may not exist at certain points in the
domain of 𝑓 . At such points the derivative is
undefined. To account for this possibility, we make the At a point of vertical tangency, the slopes of the
following definition. secant lines approach +or −from the left and from the
Definition: A function 𝑓 is said to be differentiable at right Figure below
𝒙𝟎 if the limit
𝑓(𝑥0 + ℎ) − 𝑓(𝑥0 )
𝑓 ′ (0) = lim
𝑥→0 ℎ

15 | P a g e
Module-02
Limit & Continuity
Solution (a): the formula
𝑓(𝑥0 + ℎ) − 𝑓(𝑥0 )
𝑓 ′ (0) = lim
𝑥→0 ℎ
with 𝑥0 = 0, the value of 𝑓(0), if it were to exist,
would be given by
𝑓(0 + ℎ) − 𝑓(0) 𝑓(ℎ) − 𝑓(0)
𝑓 ′ (0) = lim = lim
𝑥→0 ℎ 𝑥→0 ℎ
|ℎ| − |0|
so again the limit that defines the derivative does = lim
𝑥→0 ℎ
not exist. But,
Differentiability at 𝑥0 can also be described |ℎ| 1, ℎ>0
={
informally in terms of the behavior of the graph of ℎ −1, ℎ<0
so that,
𝑓 under increasingly stronger magnification at the |ℎ| |ℎ|
lim− = −1 and lim− =1
point 𝑃(𝑥0 , 𝑓(𝑥0)) in figure below 𝑥→0 ℎ 𝑥→0 ℎ
Since these one-sided limits are not equal, the two-
sided limit does not exist, and hence 𝑓 is not
differentiable at 𝑥 = 0.

Solution (b): A formula for the derivative of 𝑓(𝑥) =


|𝑥| can be obtained by writing |𝑥| in piecewise form
and treating the cases 𝑥 > 0 and 𝑥 < 0 separately. If
If 𝑓 is differentiable at 𝑥0 , then under sufficiently strong 𝑥 > 0, then 𝑓 (𝑥) = 𝑥 and 𝑓(𝑥) = 1; if 𝑥 < 0,
magnification at 𝑃 the graph looks like a no vertical line then 𝑓 (𝑥) = −𝑥 and 𝑓(𝑥) = −1. Thus,
(the tangent line); if a corner point occurs at 𝑥0 , then no 1, 𝑥>0
f’′(x) = {
matter how great the magnification at 𝑃 the corner −1, 𝑥<0
persists and the graph never looks like a no vertical line;
and if vertical tangency occurs at 𝑥0 , then the graph of
𝑓 looks like a vertical line under sufficiently strong
magnification at 𝑃.
Example: The graph of 𝑦 = |𝑥| in Figure has a
corner at x = 0, which implies that 𝑓 (𝑥) = |𝑥| is not
differentiable at
𝑥 = 0.
The graph of 𝑓 is shown in Figure above. Observe that
f is not continuous at 𝑥 = 0, so this example shows
that a function that is continuous everywhere may have
a derivative that fails to be continuous everywhere.

(a) Prove that 𝑓(𝑥) = |𝑥| is not differentiable at


𝑥 = 0 by showing that the limit in Definition
does not exist at 𝑥 = 0.
(b) Find a formula for 𝑓(𝑥).
16 | P a g e
Module 03
Derivative
In Mathematics, the derivative of a function of a real Start directly with the definition of the derivative
variable measures the rate of change of that function.
function with respect to a variable. Derivatives are
𝑓(𝑥 + ℎ) − 𝑓(𝑥)
fundamental to the solution of problems 𝑓 ′ (𝑥) = lim
ℎ→0 ℎ
in calculus and differential equations. In general,
(𝑥 + ℎ)2 − 𝑥 2
scientists observe changing systems (dynamical = lim
ℎ→0 ℎ
systems) to obtain the rate of change of some variable
𝑥 2 + 2𝑥ℎ+ℎ2 − 𝑥 2
of interest, incorporate this information into = lim
ℎ→0 ℎ
some differential equation, and
use integration techniques to obtain a function that can 2𝑥ℎ
= lim
be used to predict the behavior of the original system ℎ→0 ℎ

under diverse conditions. ∴ 𝑓 ′ (𝑥) = 2𝑥


The derivative of a function 𝑓(𝑥) is usually represented Applications of derivatives:
𝑑 𝑑𝑓
by 𝑑𝑥 (𝑓(𝑥)) or 𝑑𝑥 or 𝐷𝑓(𝑥) or 𝑓 ′ (𝑥).
The applications of derivatives are:
Definition of Derivatives:
1. To determine the rate of change of a quantity
The derivative of 𝑓(𝑡) with respect to 𝑡 is the function with respect to another changing quantity.
𝑓 ′ (𝑡) and is defined as 2. To determine maximum, minimum are saddle
𝑓(𝑡 + ∆𝑡) − 𝑓(𝑡) points of a function.
𝑓 ′ (𝑡) = lim
∆𝑡→0 ∆𝑡 3. To determine the concavity and convexity of a
′ (𝑡)
We often read 𝑓 as “𝑓 prime of 𝑡”. function.

The ratio on the right is the average velocity over a short 4. For approximations
time ∆𝑡. The derivative on the left side, is its limit as the
5. To obtain the tangent and normal of a function
step ∆𝑡(delta 𝑡) approaches to zero.
at a point.
Go slowly and look at each piece. The distance at time
Basic Differentiation Formulas
𝑡 + ∆𝑡 is 𝑓(𝑡 + ∆𝑡) and the distance at time 𝑡 is 𝑓(𝑡).
Subtraction gives the change in distance, between those Suppose 𝑓 and 𝑔 are differentiable functions, 𝑐 is any
times. We often write ∆𝑓 for this difference: ∆𝑓 = real number, then
𝑓(𝑡 + ∆𝑡) − 𝑓(𝑡). The average velocity is the ratio 𝑑
∆𝑓⁄∆𝑡; the change in distance divided by change in  (𝑥 𝑛 ) = 𝑛𝑥 𝑛−1
𝑑𝑥
time.
Example:
Note that ∆𝑓 is not ∆ time 𝑓. It is the change in 𝑓. 𝑑
(𝑥 8 ) = 8𝑥 8−1 = 8𝑥 7 .
Similarly, ∆𝑡 is not ∆ time 𝑡. It is the change in 𝑡. It is 𝑑𝑥
the time step; positive or negative and eventually small. 𝑑 𝑑
 𝑑𝑥
{𝑐𝑓(𝑥)} = 𝑐 𝑑𝑥 {𝑓(𝑥)}; where 𝑐 is a
Finding the Derivative of a Power Function: constant.
Find the derivative of 𝑓(𝑥) = 𝑥 2 . Example:
Solution 𝑑 𝑑
(10𝑥 5 ) = 10 𝑑𝑥 (𝑥 5 )
𝑑𝑥
Given that,
= 10.5𝑥 5−1 = 50𝑥 4 .
2
𝑓(𝑥) = 𝑥
𝑑
 𝑑𝑥
(𝑐) = 0, where 𝑐 is a constant.
17 | P a g e
Module 03
Derivative
Example: Example:
𝑑 𝑑
(100) = 0. (cos 2𝑥) = −2 sin 2𝑥.
𝑑𝑥 𝑑𝑥

𝑑 𝑑
 𝑑𝑥
(𝑥) =1  𝑑𝑥
(tan 𝑥) = 𝑠𝑒𝑐 2 𝑥

Example: 𝑑
 𝑑𝑥
(cot 𝑥) = −𝑐𝑜𝑠𝑒𝑐 2 𝑥
𝑑 𝑑
(14𝑥) = 14 𝑑𝑥 (𝑥) = 14.1 = 14. 𝑑
𝑑𝑥  (sec 𝑥) = sec 𝑥 . tan 𝑥
𝑑𝑥
𝑑 1
 (√𝑥) = 𝑑
𝑑𝑥 2√𝑥  𝑑𝑥
(cot 𝑥) = −𝑐𝑜𝑠𝑒𝑐 𝑥. cot 𝑥
𝑑 𝑑 𝑑
 𝑑𝑥
{𝑓(𝑥) + 𝑔(𝑥)} = 𝑑𝑥 {𝑓(𝑥)} + 𝑑𝑥 {𝑔(𝑥)} 
𝑑
(𝑠𝑖𝑛−1 𝑥) =
1
.
𝑑𝑥 √1−𝑥 2
Example: 𝑑 −1
 (𝑐𝑜𝑠 −1 𝑥) = .
𝑑𝑥 √1−𝑥 2
𝑑 𝑑 𝑑
(𝑥 7 ± 𝑥) = (𝑥 7 ) ± (𝑥)
𝑑𝑥 𝑑𝑥 𝑑𝑥 𝑑 1
 (𝑡𝑎𝑛−1 𝑥) = 𝑥 2 +1.
𝑑𝑥
= 7𝑥 7−1 ± 1 = 7𝑥 6 ± 1.
𝑑 −1
𝑑 𝑑 1  (𝑐𝑜𝑡 −1 𝑥) = 𝑥 2 +1.
𝑑𝑥
 𝑑𝑥
(log 𝑥) = 𝑑𝑥
(ln 𝑥) = 𝑥
𝑑 1
𝑑 1  (𝑠𝑒𝑐 −1 𝑥) = .
𝑑𝑥 |𝑥|√𝑥 2 −1
 𝑑𝑥
(log 𝑎 𝑥) = 𝑥
. log 𝑎 𝑒.
𝑑 −1
𝑑  (𝑐𝑜𝑠𝑒𝑐 −1 𝑥) =
 (𝑒 𝑥 ) =𝑒 𝑥 𝑑𝑥 |𝑥|√𝑥 2 −1
𝑑𝑥
𝑑 𝑑 𝑑
𝑑  (𝑢𝑣) =𝑢 (𝑣) + 𝑣 (𝑢)
 (𝑒 𝑚𝑥 ) = 𝑚𝑒 𝑚𝑥 𝑑𝑥 𝑑𝑥 𝑑𝑥
𝑑𝑥
𝑑 𝑑
𝑑 𝑢 𝑣 (𝑢)−𝑢 (𝑣)
Example:  ( ) = 𝑑𝑥 𝑑𝑥
𝑑𝑥 𝑣 𝑣2
𝑑
(𝑒 3𝑥 ) = 3𝑒 3𝑥 . Differentiation of algebraic, exponential and
𝑑𝑥

𝑑
logarithmic function
 (𝑎 𝑥 ) = 𝑎 𝑥 ln 𝑥.
𝑑𝑥 𝑑𝑦
Question: Find 𝑑𝑥 if
Example:
𝑑
(i) 𝑦 = 𝑥 3 − 2𝑥 2 + 36𝑥 − 10
(2𝑥 ) 𝑥
= 2 ln 2.
𝑑𝑥
(ii) 𝑦 = 𝑥 5 − 𝑥 2 + 78𝑥 − 100
𝑑
 (sin 𝑥) = cos 𝑥
𝑑𝑥 (iii) 𝑦 = 7𝑥 4 + 3𝑥 3 − 9𝑥 2 + 2𝑥 −1 + 6
3 4

𝑑
(sin 𝑚𝑥) = 𝑚 cos 𝑚𝑥 (iv) 𝑦 = √𝑥 + 𝑥
𝑑𝑥
(v) 𝑦 = 𝑒 𝑥 + 5𝑥 2 + 𝑙𝑜𝑔𝑒 𝑥
Example: 1
(vi) 𝑦 = 𝑎 𝑥 + 𝑒 −4𝑥 − 2 𝑙𝑜𝑔𝑒 𝑥 + 𝑥
𝑑 Solution: (i) Given that,
(sin 2𝑥) = 2 cos 2𝑥.
𝑑𝑥

𝑑 𝑦 = 𝑥 3 − 2𝑥 2 + 36𝑥 − 1
 𝑑𝑥
(cos 𝑥) = −sin 𝑥
𝑑 𝑑 3
𝑑 ∴ (𝑦) = (𝑥 − 2𝑥 2 + 36𝑥 − 1)
 (cos 𝑚𝑥) = −𝑚 sin 𝑚𝑥 𝑑𝑥 𝑑𝑥
𝑑𝑥

18 | P a g e
Module 03
Derivative
𝑑𝑦 𝑑 3 𝑑 𝑑 𝑑 𝑑𝑦 𝑑 1 𝑑
⇒ = (𝑥 ) − (2𝑥 2 ) + (36𝑥) − (1) ⇒ = (𝑥 3 ) + 4 (𝑥 −1 )
𝑑𝑥 𝑑𝑥 𝑑𝑥 𝑑𝑥 𝑑𝑥 𝑑𝑥 𝑑𝑥 𝑑𝑥
𝑑𝑦 𝑑𝑦 1 1−1
⇒ = 3𝑥 3−1 − 2.2𝑥 2−1 + 36.1 − 0 ⇒ = 𝑥 3 + 4(−1)𝑥 −1−1
𝑑𝑥 𝑑𝑥 3
𝑑𝑦 𝑑𝑦 1 −2
∴ = 3𝑥 2 − 4𝑥 + 36 ∴ = 𝑥 3 − 4𝑥 −2
𝑑𝑥 𝑑𝑥 3
(ii) Given that, (v) Given that,

𝑦 = 𝑥 5 − 𝑥 −2 + 78𝑥 − 100 𝑦 = 𝑒 𝑥 + 5𝑥 2 + 𝑙𝑜𝑔𝑒 𝑥


𝑑 𝑑 5 𝑑 𝑑 𝑥
∴ (𝑦) = (𝑥 − 𝑥 −2 + 78𝑥 − 100) ∴ (𝑦) = (𝑒 + 5𝑥 2 + 𝑙𝑜𝑔𝑒 𝑥)
𝑑𝑥 𝑑𝑥 𝑑𝑥 𝑑𝑥
𝑑𝑦 𝑑 5 𝑑 −2 𝑑 𝑑 𝑑𝑦 𝑑 𝑥 𝑑 𝑑
⇒ = (𝑥 ) − (𝑥 ) + (78𝑥) − (100) ⇒ = (𝑒 ) + (5𝑥 2 ) + (log 𝑒 𝑥)
𝑑𝑥 𝑑𝑥 𝑑𝑥 𝑑𝑥 𝑑𝑥 𝑑𝑥 𝑑𝑥 𝑑𝑥 𝑑𝑥
𝑑𝑦 𝑑𝑦 1
⇒ = 5𝑥 5−1 − (−2)𝑥 −2−1 + 78.1 − 0 ⇒ = 𝑒 𝑥 + 5.2𝑥 2−1 +
𝑑𝑥 𝑑𝑥 𝑥
𝑑𝑦 𝑑𝑦 1
∴ = 5𝑥 4 + 4𝑥 −3 + 78 ∴ = 𝑒 𝑥 + 10𝑥 +
𝑑𝑥 𝑑𝑥 𝑥
(iii) Given that,

𝑦 = 7𝑥 4 + 3𝑥 3 − 9𝑥 2 + 2𝑥 −1 + 6 (vi)Given that,
𝑑 𝑑 1
∴ (𝑦) = (7𝑥 4 + 3𝑥 3 − 9𝑥 2 + 2𝑥 −1 + 6) 𝑦 = 𝑎 𝑥 + 𝑒 −4𝑥 − 2 𝑙𝑜𝑔𝑒 𝑥 +
𝑑𝑥 𝑑𝑥 𝑥
𝑑𝑦 𝑑 𝑑 𝑑 1
⇒ = (7𝑥 4 + 3𝑥 3 − 9𝑥 2 + 2𝑥 −1 + 6) ∴ (𝑦) = (𝑎 𝑥 + 𝑒 −4𝑥 − 2 𝑙𝑜𝑔𝑒 𝑥 + )
𝑑𝑥 𝑑𝑥 𝑑𝑥 𝑑𝑥 𝑥
𝑑𝑦 𝑑 𝑑 𝑑 𝑑𝑦 𝑑 𝑥
⇒ = (7𝑥 4 ) + (3𝑥 3 ) − (9𝑥 2 ) ⇒ = (𝑎 + 𝑒 −4𝑥 − 2 𝑙𝑜𝑔𝑒 𝑥 + 𝑥 −1 )
𝑑𝑥 𝑑𝑥 𝑑𝑥 𝑑𝑥 𝑑𝑥 𝑑𝑥
𝑑 𝑑
+ (2𝑥 −1 ) + (6) 𝑑𝑦 𝑑 𝑥 𝑑 −4𝑥 𝑑
𝑑𝑥 𝑑𝑥 ⇒ = (𝑎 ) + (𝑒 ) − (2 𝑙𝑜𝑔𝑒 𝑥)
𝑑𝑥 𝑑𝑥 𝑑𝑥 𝑑𝑥
𝑑𝑦 𝑑 −1
⇒ = 7.4𝑥 4−1 + 3.3𝑥 3−1 − 9.2𝑥 2−1 + (𝑥 )
𝑑𝑥 𝑑𝑥
+ 2(−1)𝑥 −1−1 + 0
𝑑𝑦 1
⇒ = 𝑎 𝑥 ln 𝑎 + (−4)𝑒 −4𝑥 − 2 + (−1)𝑥 −1−1
𝑑𝑦 𝑑𝑥 𝑥
∴ = 28𝑥 3 + 9𝑥 2 − 18𝑥 − 2𝑥 −2
𝑑𝑥 𝑑𝑦 2
∴ = 𝑎 𝑥 ln 𝑎 − 4𝑒 −4𝑥 − − 𝑥 −2
(iv) Given that, 𝑑𝑥 𝑥

3 4
𝑦 = √𝑥 +
𝑥 Differentiation of chain rule function
𝑑 𝑑 3 4 𝑑𝑦
∴ (𝑦) = ( √𝑥 + ) Question: Find 𝑑𝑥 if
𝑑𝑥 𝑑𝑥 𝑥
𝑑𝑦 𝑑 1
(i) 𝑦 = (𝑥 2 − 6𝑥 + 5)5
⇒ = (𝑥 3 + 4𝑥 −1 )
𝑑𝑥 𝑑𝑥 (ii) 𝑦 = 𝑠𝑖𝑛2 (log sec 𝑥)

19 | P a g e
Module 03
Derivative
2 −6𝑥+5)5 𝑑𝑦 5 𝑑
(iii) 𝑦 = 𝑒 (𝑥 ⇒
2
= 𝑒 (𝑥 −6𝑥+5) . 5(𝑥 2 − 6𝑥 + 5)5−1 (𝑥 2 − 6𝑥
𝑑𝑥 𝑑𝑥
(iv) 𝑦 = 𝑙𝑛{ √1 + 𝑙𝑛 𝑥 − 𝑠𝑖𝑛 𝑥} + 5)

Solution: 𝑑𝑦 2 5
⇒ = 𝑒 (𝑥 −6𝑥+5) . 5(𝑥 2 − 6𝑥 + 5)4 (2𝑥 − 6)
𝑑𝑥
(i) Given that,
𝑑𝑦 2 5
∴ = 5(2𝑥 − 6)(𝑥 2 − 6𝑥 + 5)4 𝑒 (𝑥 −6𝑥+5)
𝑦 = (𝑥 2 − 6𝑥 + 5)5 𝑑𝑥
Differentiating w.r.to x we get (iv) Given that,
𝑑 𝑑 2 𝑦 = 𝑙𝑛{ √1 + 𝑙𝑛 𝑥 − 𝑠𝑖𝑛 𝑥}
∴ (𝑦) = (𝑥 − 6𝑥 + 5)5
𝑑𝑥 𝑑𝑥 Differentiating w. t. to 𝑥 we get,
𝑑𝑦 𝑑 𝑑 𝑑
⇒ = 5(𝑥 2 − 6𝑥 + 5)5−1 (𝑥 2 − 6𝑥 + 5) ∴ (𝑦) = (𝑙𝑛{ √1 + 𝑙𝑛 𝑥 − 𝑠𝑖𝑛 𝑥})
𝑑𝑥 𝑑𝑥 𝑑𝑥 𝑑𝑥
𝑑𝑦 𝑑𝑦 1 𝑑
∴ = 5(𝑥 2 − 6𝑥 + 5)4 (2𝑥 − 6) ⇒ = {(√1 + 𝑙𝑛 𝑥)
𝑑𝑥 𝑑𝑥 √(1 + 𝑙𝑛 𝑥) − 𝑠𝑖𝑛 𝑥 𝑑𝑥

(ii) Given than − 𝑠𝑖𝑛 𝑥}


𝑦 = 𝑠𝑖𝑛2 (log sec 𝑥) 𝑑𝑦 1 1 𝑑
⇒ = [ (1
𝑑𝑥 √(1 + 𝑙𝑛 𝑥) − 𝑠𝑖𝑛 𝑥 2√(1 + 𝑙𝑛𝑥) 𝑑𝑥
Differentiating w.r.to x we get
+ 𝑙𝑛 𝑥) − 𝑐𝑜𝑠 𝑥]
𝑑𝑦 𝑑
= {𝑠𝑖𝑛2 (log sec 𝑥)}
𝑑𝑥 𝑑𝑥 𝑑𝑦 1 1
⇒ = [ − 𝑐𝑜𝑠 𝑥]
𝑑𝑦 𝑑 𝑑𝑥 √(1 + 𝑙𝑛 𝑥) − 𝑠𝑖𝑛 𝑥 2𝑥√(1 + 𝑙𝑛𝑥)
⇒ = 2 sin (log sec 𝑥) {sin (log sec 𝑥)}
𝑑𝑥 𝑑𝑥
Differentiation of product of functions
𝑑𝑦
⇒ Question: Find 𝑑𝑥 if
𝑑𝑦
𝑑𝑥
𝑑
= 2 sin (log sec 𝑥) 𝑐𝑜𝑠 (log sec 𝑥) (log sec 𝑥) (i) 𝑦 = 𝑥 log 𝑥
𝑑𝑥
(ii) 𝑦 = 𝑥𝑒 𝑥
𝑑𝑦
⇒ (iii) 𝑦 = 𝑥𝑐𝑜𝑠𝑥
𝑑𝑥
1 (iv) 𝑦 = 𝑥𝑥
= 2 sin (log sec 𝑥) cos(log sec 𝑥) . sec 𝑥 tan 𝑥 (v) 𝑦 = (sec 𝑥)𝑡𝑎𝑛𝑥
sec 𝑥
(vi) 𝑦 = √𝑥𝑒 𝑥 𝑠𝑒𝑐𝑥
𝑑𝑦 𝑥
∴ = sin 2 (log sec 𝑥) tan 𝑥 (vii) 𝑦 = 𝑥𝑥
𝑑𝑥
Solution:

(i) Given that


(iii) Given than 𝑦 = 𝑥 2 log 𝑥
2 −6𝑥+5)5
𝑦 = 𝑒 (𝑥 𝑑 𝑑 2
∴ (𝑦) = (𝑥 log 𝑥)
Differentiating w.r.to x we get 𝑑𝑥 𝑑𝑥

𝑑𝑦 𝑑 (𝑥 2 −6𝑥+5)5 𝑑𝑦 𝑑 𝑑 2
= ⇒ = 𝑥2 (log 𝑥) + log 𝑥 (𝑥 )
{𝑒 } 𝑑𝑥 𝑑𝑥 𝑑𝑥
𝑑𝑥 𝑑𝑥
𝑑𝑦 1
𝑑𝑦 2 5 𝑑
⇒ = 𝑥 2 + log 𝑥 . 2𝑥 2−1
⇒ = 𝑒 (𝑥 −6𝑥+5) {(𝑥 2 − 6𝑥 + 5)5 } 𝑑𝑥 𝑥
𝑑𝑥 𝑑𝑥
20 | P a g e
Module 03
Derivative
𝑑𝑦 ⇒ log 𝑦 = tan 𝑥 log sec 𝑥
∴ = 𝑥 + 2𝑥 log 𝑥
𝑑𝑥
Differentiating w. r. to 𝑥, we get
(ii) Given that
𝑦 = 𝑥𝑒 𝑥 𝑑 𝑑
∴ (log 𝑦) = (tan 𝑥 log sec 𝑥)
𝑑 𝑑 𝑑𝑥 𝑑𝑥
∴ (𝑦) = (𝑥𝑒 7𝑥 ) 1 𝑑 𝑑
𝑑𝑥 𝑑𝑥 ⇒ (𝑦) = tan 𝑥 (log sec 𝑥)
𝑦 𝑑𝑥 𝑑𝑥
𝑑𝑦 𝑑 7𝑥 𝑑 𝑑
⇒ =𝑥 (𝑒 ) + 𝑒 7𝑥 (𝑥) + log sec 𝑥 (tan 𝑥)
𝑑𝑥 𝑑𝑥 𝑑𝑥 𝑑𝑥
𝑑𝑦 1 𝑑𝑦 1 𝑑
⇒ = 7𝑥𝑒 7𝑥 + 𝑒 7𝑥 ⇒ = tan 𝑥 (sec 𝑥) + log sec 𝑥 . 𝑠𝑒𝑐 2 𝑥
𝑑𝑥 𝑦 𝑑𝑥 sec 𝑥 𝑑𝑥
(iii) Given that 1 𝑑𝑦 1
𝑦 = 𝑥𝑐𝑜𝑠𝑥 ⇒ = tan 𝑥 . sec 𝑥 . tan 𝑥 + 𝑠𝑒𝑐 2 𝑥 log sec 𝑥
𝑦 𝑑𝑥 sec 𝑥
𝑑 𝑑
∴ (𝑦) = (𝑥 cos 𝑥) 𝑑𝑦
𝑑𝑥 𝑑𝑥 ⇒ = 𝑦(𝑡𝑎𝑛2 𝑥 + 𝑠𝑒𝑐 2 𝑥 log sec 𝑥)
𝑑𝑦 𝑑 𝑑 𝑑𝑥
⇒ =𝑥 (cos 𝑥) + cos 𝑥 (𝑥)
𝑑𝑥 𝑑𝑥 𝑑𝑥 𝑑𝑦
∴ = (sec 𝑥)tan 𝑥 (𝑡𝑎𝑛2 𝑥 + 𝑠𝑒𝑐 2 𝑥 log sec 𝑥)
𝑑𝑦 𝑑𝑥
⇒ = −𝑥 sin 𝑥 + cos 𝑥
𝑑𝑥

(vi) Given that,


(iv) Given that 𝑦 = √𝑥𝑒 𝑥 𝑠𝑒𝑐 𝑥
𝑥
𝑦=𝑥 Differentiating w. r. to 𝑥 we get,
Taking log on both sides 𝑑 𝑑
(𝑦) = (√𝑥𝑒 𝑥 𝑠𝑒𝑐 𝑥)
𝑑𝑥 𝑑𝑥
log 𝑦 = log(𝑥 𝑥 ) 𝑑𝑦 𝑑 𝑥
⇒ = √𝑥 (𝑒 𝑠𝑒𝑐 𝑥)
⇒ log 𝑦 = 𝑥 log 𝑥 𝑑𝑥 𝑑𝑥
𝑑
Differentiating w. r. to 𝑥, we get + 𝑒 𝑥 𝑠𝑒𝑐 𝑥 (√𝑥)
𝑑𝑥
𝑑𝑦 𝑑
𝑑 𝑑 ⇒ = √𝑥 {𝑒 𝑥 (𝑠𝑒𝑐 𝑥)
(log 𝑦) =
∴ (𝑥 log 𝑥) 𝑑𝑥 𝑑𝑥
𝑑𝑥 𝑑𝑥 𝑑 𝑥
1 𝑑 𝑑 𝑑 + 𝑠𝑒𝑐 𝑥 (𝑒 )}
⇒ (𝑦) = 𝑥 (log 𝑥) + log 𝑥 (𝑥) 𝑑𝑥
𝑦 𝑑𝑥 𝑑𝑥 𝑑𝑥 1
+ 𝑒 𝑥 𝑠𝑒𝑐 𝑥 .
1 𝑑𝑦 1 2√𝑥
⇒ = 𝑥 + log 𝑥 . 1 𝑑𝑦
𝑦 𝑑𝑥 𝑥 ⇒ = √𝑥{𝑒 𝑥 𝑠𝑒𝑐 𝑥 𝑡𝑎𝑛 𝑥 + 𝑠𝑒𝑐 𝑥 . 𝑒 𝑥 }
𝑑𝑥
𝑑𝑦 1
⇒ = 𝑦(1 + log 𝑥) + 𝑒 𝑥 𝑠𝑒𝑐 𝑥 .
𝑑𝑥 2√𝑥
𝑑𝑦 𝑑𝑦 1
∴ = 𝑥 𝑥 (1 + log 𝑥) ⇒ = 𝑒 𝑥 𝑠𝑒𝑐 𝑥 (√𝑥 𝑡𝑎𝑛 𝑥 + √𝑥 + )
𝑑𝑥 𝑑𝑥 2√𝑥
𝑑𝑦 𝑒 𝑥 𝑠𝑒𝑐 𝑥 (2𝑥 𝑡𝑎𝑛 𝑥 + 2𝑥 + 1)
(v) Given that ∴ = .
𝑑𝑥 2√𝑥
𝑦 = (sec 𝑥)tan 𝑥 (vii) Given that,
Taking log on both sides 𝑥
𝑦 = 𝑥𝑥
log 𝑦 = log(sec 𝑥)tan 𝑥 Differentiating w. r. to 𝑥 we get,

21 | P a g e
Module 03
Derivative
𝑑𝑦 𝑥 𝑑 (i) Given that,
∴ 𝑑𝑥 = 𝑥 𝑥 𝑑𝑥
{𝑥 𝑥 𝑙𝑛 𝑥} =
𝑥 𝑑 𝑑
log 𝑥
𝑥 𝑥 {𝑥 𝑥 𝑑𝑥 (𝑙𝑛 𝑥) + 𝑙𝑛 𝑥 𝑑𝑥 (𝑥 𝑥 )} 𝑦=
𝑥2
𝑑𝑦 𝑥 1
⇒ 𝑑𝑥 = 𝑥 𝑥 {𝑥 𝑥 . 𝑥 + 𝑙𝑛 𝑥 . 𝑥 𝑥 𝑑𝑥 (𝑥 𝑙𝑛 𝑥)}
𝑑 Differentiating w. r. to 𝑥 we get,
𝑑 𝑑 log 𝑥
𝑑𝑦 𝑥 1 1 (𝑦) = ( )
⇒ = 𝑥 𝑥 . 𝑥 𝑥 { + 𝑙𝑛 𝑥 (𝑥. + 𝑙𝑛 𝑥)} 𝑑𝑥 𝑑𝑥 𝑥 2
𝑑𝑥 𝑥 𝑥
2 𝑑 𝑑 2
𝑑𝑦 𝑥 1 𝑑𝑦 𝑥 𝑑𝑥 (log 𝑥) − log 𝑥 𝑑𝑥 (𝑥 )
⇒ = 𝑥 𝑥 . 𝑥 𝑥 { + 𝑙𝑛 𝑥 (1 + 𝑙𝑛 𝑥)} ⇒ =
𝑑𝑥 𝑥 𝑑𝑥 (𝑥 2 )2
21
Try yourself: 𝑑𝑦 𝑥 𝑥 − log 𝑥 . 2𝑥
⇒ =
𝑑𝑦 𝑑𝑥 𝑥4
Simple differentiation find if 𝑑𝑦 1 − log 𝑥
𝑑𝑥
∴ =
𝑑𝑥 𝑥3
(i) 𝑦 = 𝑥 8 log 4𝑥 (ii) Given that,
(ii) 𝑦 = 𝑥 3 𝑒 −5𝑥
(iii) 𝑦 = 𝑥 sin 9𝑥 𝑥3 + 𝑥
𝑦=√ 2
(iv) 𝑦 = 𝑥 sin 𝑥 𝑥 +5
𝑑𝑦 𝑠𝑖𝑛2 (𝑎+𝑦)
Problem: Prove that, 𝑑𝑥 = sin 𝑎
where sin 𝑦 =
𝑥 sin(𝑎 + 𝑦) Differentiating w. r. to 𝑥 we get,

Proof: Given that 𝑑 𝑑 𝑥3 + 𝑥


(𝑦) = (√ 2 )
𝑑𝑥 𝑑𝑥 𝑥 +5
sin 𝑦 = 𝑥 sin(𝑎 + 𝑦)
𝑑𝑦 1 𝑑 𝑥3 + 𝑥
sin 𝑦 ⇒ = ( 2 )
⇒𝑥= 𝑑𝑥 𝑥 3 + 𝑥 𝑑𝑥 𝑥 + 5
sin(𝑎 + 𝑦) √
2 2
𝑥 +5
Differentiating w.r.to 𝑥 we get 𝑑𝑦

𝑑𝑥 sin(𝑎 + 𝑦) cos 𝑦 − sin 𝑦 cos(𝑎 + 𝑦) 𝑑𝑥
∴ = 𝑑 3 𝑑
√𝑥 2 + 5 (𝑥 2 + 5) (𝑥 + 𝑥) − (𝑥 3 + 𝑥) (𝑥 2 + 5)
𝑑𝑦 𝑠𝑖𝑛2 (𝑎 + 𝑦) = { 𝑑𝑥 𝑑𝑥 }
2√𝑥 3 + 𝑥 (𝑥 2 + 5)2
𝑑𝑦 sin(𝑎 + 𝑦) cos 𝑦 − cos(𝑎 + 𝑦) sin 𝑦
⇒ =
𝑑𝑥 𝑠𝑖𝑛2 (𝑎 + 𝑦) 𝑑𝑦

𝑑𝑥
𝑑𝑦 sin{(𝑎 + 𝑦) − 𝑦)}
⇒ = √𝑥 2 + 5 (𝑥 2 + 5)(3𝑥 2 + 1) − (𝑥 3 + 𝑥)(2𝑥)
𝑑𝑥 𝑠𝑖𝑛2 (𝑎 + 𝑦) = { }
2√𝑥 3 + 𝑥 (𝑥 2 + 5)2
𝑑𝑦 sin 𝑎
⇒ = 2 𝑑𝑦 √𝑥 2 + 5 3𝑥 4 + 16𝑥 2 + 5 − 2𝑥 4 − 2𝑥 2
𝑑𝑥 𝑠𝑖𝑛 (𝑎 + 𝑦) ⇒ = { }
𝑑𝑥 2√𝑥 3 + 𝑥 (𝑥 2 + 5)2
𝑑𝑦 𝑠𝑖𝑛2 (𝑎 + 𝑦)
∴ =
𝑑𝑥 sin 𝑎 𝑑𝑦 √𝑥 2 + 5 𝑥 4 + 14𝑥 2 + 5
⇒ = { }
𝑑𝑥 2√𝑥 3 + 𝑥 (𝑥 2 + 5)2
Problem: Find the differential coefficient of the
followings: Successive Differentiation
log 𝑥
(i) 𝑦= 4 𝑑2𝑦
𝑥2 Problem: 01. If 𝑦 = 2𝑥 + 𝑥 then prove that 𝑥 2 𝑑𝑥 2 +
𝑥 3 +𝑥 𝑑𝑦
(ii) 𝑦=√ 𝑥 𝑑𝑥 − 𝑦 = 0
𝑥 2 +5
Solution: Solution: Given that
22 | P a g e
Module 03
Derivative
4 ⇒ √1 − 𝑥 2 𝑦1 = 𝑎𝑦
𝑦 = 2𝑥 +
𝑥
⇒ (1 − 𝑥 2 )𝑦1 2 = 𝑎2 𝑦 2
⇒ 𝑦 = 2𝑥 + 4𝑥 −1
Again, differentiating both sides w.r.to 𝑥 we get
Differentiating w. r. to 𝑥 we get,
𝑑 𝑑 2 2
𝑑𝑦 {(1 − 𝑥 2 )𝑦1 2 } = (𝑎 𝑦 )
= 2 + 4(−1)𝑥 −2 𝑑𝑥 𝑑𝑥
𝑑𝑥
⇒ (1 − 𝑥 2 )2𝑦1 𝑦2 + 𝑦1 2 (0 − 2𝑥) = 𝑎2 . 2𝑦𝑦1
𝑑𝑦
∴ = 2 − 4𝑥 −2
𝑑𝑥 ⇒ 2(1 − 𝑥 2 )𝑦1 𝑦2 − 2𝑥𝑦1 2 = 2𝑎2 𝑦𝑦1
Again, ⇒ (1 − 𝑥 2 )𝑦2 − 𝑥𝑦1 = 𝑎2 𝑦
𝑑2 𝑦 ∴ (1 − 𝑥 2 )𝑦2 − 𝑥𝑦1 = 𝑎2 𝑦
= 0 − 4(−2)𝑥 −3
𝑑𝑥 2
(𝑝𝑟𝑜𝑣𝑒𝑑)
= 8𝑥 −3
Problem -03: If 𝑦 = sec 𝑥 then show that 𝑦2 =
L.H.S. 𝑦 (2𝑦 2 − 1).
𝑑2 𝑦 𝑑𝑦
𝑥 2 𝑑𝑥 2 + 𝑥 𝑑𝑥 − 𝑦 Solution: Given that,

= 𝑥 2 . 8𝑥 −3 + 𝑥(2 − 4𝑥 −2 ) − (2𝑥 + 4𝑥 −1 ) 𝑦 = sec 𝑥

= 8𝑥 −3+2 + 2𝑥 − 4𝑥 −2+1 − 2𝑥 − 4𝑥 −1 Differentiating w.r.to 𝑥 we get

= 8𝑥 −1 + 2𝑥 − 4𝑥 −1 − 2𝑥 − 4𝑥 −1 𝑦1 = sec 𝑥 tan 𝑥

=0 Again, differentiating both sides w.r.to 𝑥 we get

= 𝑅. 𝐻. 𝑆. 𝑑 𝑑
𝑦2 = sec 𝑥 (tan 𝑥) + tan 𝑥 (sec 𝑥)
𝑑𝑥 𝑑𝑥
𝑑2 𝑦 𝑑𝑦
Hence 𝑥 2 𝑑𝑥 2 + 𝑥 𝑑𝑥 − 𝑦 = 0(proved) = sec 𝑥. 𝑠𝑒𝑐 2 𝑥 + tan 𝑥 . sec 𝑥 tan 𝑥
−1 𝑥
Problem-02. If 𝑦 = 𝑒 𝑎 sin , then show that = 𝑠𝑒𝑐 3 𝑥 + 𝑡𝑎𝑛2 𝑥 . sec 𝑥
(1 − 𝑥 2 )𝑦2 - x𝑦1 =𝑎2 𝑦.
= (𝑠𝑒𝑐 2 𝑥 + 𝑡𝑎𝑛2 𝑥 )sec 𝑥
Solution: Given that = (𝑠𝑒𝑐 2 𝑥 + 𝑠𝑒𝑐 2 𝑥 − 1 ) sec 𝑥
−1 𝑥
𝑦 = 𝑒 𝑎 sin = (2 𝑠𝑒𝑐 2 𝑥 − 1 ) sec 𝑥

Differentiating w. r. to 𝑥 we get, 𝑦2 = 𝑦 (2 𝑦 2 − 1)
(proved)
𝑑 𝑑 𝑎 sin−1 𝑥
(𝑦) = (𝑒 )
𝑑𝑥 𝑑𝑥 d y 2
dy
Question: Find and for the following
⇒ 𝑦1 = 𝑒 𝑎 sin
−1 𝑥 𝑎
;[
𝑑
(sin−1 𝑥)
dx dx 2
√1 − 𝑥2 𝑑𝑥 functions:
1
= ]
√1 − 𝑥 2 (a) y  3x 2  5 x  1 (b) y  x  4
⇒ √1 − 𝑥 2 𝑦1 = 𝑎 𝑒 𝑎 sin
−1 𝑥 x3  x
(c ) y  2
x 5
23 | P a g e
Module 03
Derivative
(d) If y  x 2  2 x  3 , prove that d 2 y d  dy  d  1 1 / 2 
     x  4  
dx  dx  dx  2
1 d
x  41/ 2  
 2 dx
2
3 dx
 dy   d y 
2 2
    2   4 y
 dx   dx  1 1
   .x  4 x  4
1/ 2 1 d

2 2 dx
2
3x  4 d y
(e) If y  , calculate the value of at the
  . x  4  1  0 
1 3 / 2
4x  1 dx 2
4
 1 11 
point  , .
  . x  4 
1
2 2  3 / 2
(Answer)
4
2 x2  3 2
2 d y dy
(f) If y  , prove that x x  y x3  x
x dx 2
dx (c) Given, y  2
x 5
(g) Suppose
d3y dy d  x 3  x 
y  10 x  6 x  2 x  3x  10 x  10 find 3   
dx dx  x 2  5 
5 4 3 2

dx
where x = -1.
x2  5 
d 3
dx
 
x  x  x3  x
d 2
dx
x 5     

x2  5
2
 
Solution:

(a) Given, y  3x  5 x  1
2

x 2
  
 5 3x 2  1  x 3  x 2 x  0 
x 4  10 x 2  25

3x 4  x 2  15 x 2  5  2 x 4  2 x 2
dy d

dx dx

3x 2  5 x  1  
x 4  10 x 2  25


d
dx
 
3 x 2  5 x   1  3.2 x   5.1  0  6 x  5
d
dx
d
dx
x 4  14 x 2  5
 4
(Answer)
x  10 x 2  25
d 2 y d  dy  d
    6 x  5  6 x   5  6.1  0  6 d 2y  d  dy   d  x 4  14 x 2  5 
d d 2

2
dx  dx  dx  
dx dx dx dx dx  dx  dx  x 4  10 x 2  25 
(Answer)
x 4
 10 x 2  25  dxd x 4
 
 14 x 2  5  x 4  14 x 2  5  dxd x 4
 10 x 2  25 
(b) Given, y  x4 
x 4
 10 x 2  25  2

dy d
 x4 
1 d
x  4 
x 4
  
 10 x 2  25 4 x 3  28 x  0  x 4  14 x 2  5 4 x 3  20 x  0  
dx dx 2 x  4 dx x 4
 10 x  252
2

  x  4  1  0 
1 1 / 2
(Answer) 
x 4
  
 10 x 2  25 4 x 3  28 x  x 4  14 x 2  5 4 x 3  20 x  
2
x 4
 10 x 2  25 
2

 x  4
1 1 / 2
(Answer)
2

24 | P a g e
Module 03
Derivative
y  x 2  2 x  3 , prove that 12 x  3  12 x  16
(d) If 
3 4 x  12
 dy   d y 
2 2
    2   4 y
 19.4 x  1
2
 dx   dx 

Given, y  x 2  2x  3
d 2 y d  dy  d
dx 2
  
dx  dx  dx

 19.4 x  1  19 4 x  1
2 d
dx
2

dy d 2

dx dx

x  2x  3 
dx

d 2
dx
 
x  2 x   3  2 x  2  0  2 x  2
d d
dx
 19. 2 4 x  1 4 x  1
 2 1 d
dx
d 2 y d  dy  d
    2 x  2  2 x   2  2.1  0  2
d d
 38.4 x  1 .4
3
dx 2
dx  dx  dx dx dx
152
 dy   d y 
2 2 3


L.H.S.=     2  4 x  13
 dx   dx 
 1 11  1
At the point  , ; x
 2 x  2  2
2 3
2 2  2

 4 x 2  8x  4  8 d2y 152 152 152


   3  152
 4 x 2  8x  12 dx 2
 1 
3
2  1 1
3

 4.  1

 4 x2  2x  3  (Answer)
 2 

 4 y  R.H .S (Proved)
2 x2  3 2
2 d y dy
(f) If y  , prove that x 2
x  y.
d y 2 x dx dx
3x  4
(e) If y  , calculate the value of at the
4x  1 dx 2 2x 2  3
 1 11  Given y 
point  , . x
2 2 
3x  4 
dy d 2 x  3
2

 
x  2 x 2  3  2 x 2  3 x 
d d
  
Given, y      dx dx
4x  1 dx dx  x  x 2

dy d  3x  4 
 
dx dx  4 x  1 



x2.2 x  0  2 x 2  3 .1 
x2
4 x  1 d 3x  4  3x  4 d 4 x  1
 dx dx 4x 2  2x 2  3
4 x  1 2 
x2


4 x  1. 3  3x  4. 4 2x2  3
4 x  12 
x2

25 | P a g e
Module 03
Derivative
d 2 y d  dy  d  2 x 2  3 
    
dy

d

10 x 5  6 x 4  2 x 3  3 x 2  10 x  10 
dx 2 dx  dx  dx  x 2  dx dx


d

10 x 5 
d

6x 4 
d
 
2x3 
d
 
3x 2  
x   
d
dx
2x2  3  2x 2  3
2
 
d 2
dx
x    dx dx dx dx
  10 x   10 
d d
x2
2
  dx dx
 10.5x 4  6.4x3  2.3x 2  3.2x  10.1  0

x 2.2x  0  2x
2 2

 3 2 x 
4  50 x 4  24 x 3  6 x 2  6 x 10
x


4x3  4x3  6x d2y d
dx 2

dx

50 x 4  24 x 3  6 x 2  6 x  10 
x4
6x

x4 
d
dx
50.x 4 
d
dx
 
24 x 3 
d

dx
 
6x2 
d
dx
6 x   d 10
dx
6
  50.4 x 3  24.3x 2  6.2 x  6  0
x3
 200 x 3  72 x 2  12 x  6
d2y dy
L.H .S.  x 2 2  x
dx dx d3y d
dx 3

dx

200 x 3  72 x 2  12 x  6 
 6  2x  3 
2
 x 2 . 3   x. 
x     
2
 x  
d
200 x 3 
d
72 x 2 
d
12 x   d 6
dx dx dx dx
6 2x2  3
   200.3x 2  72.2x  12.1  0
x x
 600 x 2 144 x  12
6  2x  3
2
 For x  1
x
d3y
2x  3  600. 1  144. 1  12
2 2

 dx 3 (Answer)
x
 600  144  12  756
 y =R.H.S. (Proved)  y  2 x  3 
2
Home work:
 x 
1.If y  Aemx  Be mx then prove that
(g) Given that, y2  m2 y  0
y  10 x 5  6 x 4  2 x 3  3x 2  10 x  10 2.If y  Ae x  Be  x then prove that
.
y2  y  0

3. If y  x 4  4 x3  10 then find y2 and y3 .

26 | P a g e
Module 03
Derivative
If y  8 x 5  4 x 3  14 x 2 dy d  1  d  2
1
 
1
4.   x   x  x 2 
x  dx 
then find y2 , y3 and y4 . 
dx dx  

d  2  d  2 
1 1

Question: Find the gradient of the tangent of the   x    x 


following curves: dx   dx  

2x 1 2 1  1   2 1
1 1
(i) y  at the point where x=1.
x2  x    x
2  2
16 x3  1
(ii) y 
1 3
at the point where x=1. 1  1 
x2  x 2 x 2
2 2
1
(iii) y  x at the point (4, 2.5). 
1

1
x 1
2
3
2x 2x 2
2x
(iv) y  at the point where the curve crosses At the point (4, 2.5): x = 4
x2
the x-axis.  Gradient of the curve At the point (4, 2.5)

Solution (i): dy  1 1
1
 3
2x dx
Given curve: y  2.4 2
2.4 2
x2
1 1
Gradient of the curve at any x.  
2 .2 2 .8

1 1
dy d  2  x 
x
d
dx
2  x   2  x 
2 d 2
dx
x    
4 16
  
dx dx  x 2  x2
2
  
4 1

3
(Answer)
x 2 0  1  2  x 
. 2x  x 2  4x  2x 2
16 16
 = = 2x
x4 x4 Solution (iv):Given curve: y  --------(1)
x2
x 2  4x x  4
= 3 Gradient of the curve at the Gradient of the curve at any x.
x4 x
point where x = 1. 

dy 1  4
 3  3 (Answer) dy d  2  x 
x2
d
dx dx
 
2  x   2  x  d x 2
dx 1   
dx dx  x 2  x  
2 2

1
Solution (ii):Given curve: y  x 
x x 2 0  1  2  x 
. 2x
 4
Gradient of the curve at any x. x

 x 2  4x  2x 2
=
x4
27 | P a g e
Module 03
Derivative
x 2  4x  x  3 .( x  5) 6 (11x  1) (Answer)
3
=
x4
dy
Again if 0
x4 dx
=
x3
 x  3 .(x  5) 6 (11x  1)  0
3

The value of y co-ordinate is zero at the point where it


crosses x axis. Either Or, Or,
From (1), we have
( x  5) 6  0 (11x  1)  0

0
2 x x  3 3
0  11x  1
x2  ( x  5)  0
 x  3  0 1
 0  2 x x  5 x  
11
 x  3
x  2
 The values of x are -3 or 5 or -1/11 (Answer)
Gradient of the curve at the point where it crosses x
axis will be obtained for x=2. Differentiation of implicit function

dy x  4 2  4  2
i. e.     1 (Answer)
dx x 2 2 dy
20. Find for the following curves:
y  x  3 x  5 , find dy and the
4 7 dx
Question: If
dx (a) 𝑦 = 𝑥 2 + 𝑦 2 + 3𝑥 − 6𝑦 + 6 = 5
dy
values of x for which 0. 1 1
dx (b) 𝑦 = 𝑥 2 − 2𝑥𝑦 + 3𝑦 2 = 9 (c) 𝑥 + 𝑦 = 4

Solution:
(d) If x 2 y  xy 2  2 find the value of
dy
at the
y  x  3 x  5
4 7 dx
Given that, point (2,1).


dy

dx dx
d

x  34 x  57  (e) If 5x 2 y  2 x 2 y 2  3xy 2  xy  5 find
dy
at the
dx
point (1,2).
  x  3 x  57  x  57 d x  34
4 d
dx dx Solution:

(a) Given that


  x  3 .7( x  5) 6 x  5  x  57 .4( x  3) 3 d x  3
4 d
dx dx x 2  y 2  3x  6 y  6  5
Differentiate both sides with respect to x
 7.x  3 .( x  5) 1  0  4x  5 ( x  3) 1  0
4 6 7 3
d 2

x  y 2  3x  6 y  6 
d
5 
 x  3 .( x  5) 7( x  3)  4( x  5)
3 6
dx dx


d 2
 
x 
d 2
y 
d
 
3x   d 6 y   d 6  d 5
 x  3 .( x  5) 6 7 x  21  4 x  20
3 dx dx dx dx dx dx

28 | P a g e
Module 03
Derivative

 2x  2 y
dy
 3 .1  6
dy
00
Differentiate both sides with respect to x
dx dx

d
 y  x   d 4 xy 

dy
2 y  6  2 x  3 dx dx
dx

d
 y   d x   4 x d  y   y d 4 x 
dy  2 x  3 dx dx dx dx
  (Answer)
dx 2y  6 dy dy
 1  4x  y.4
dx dx
(b) Given that

x 2  2 xy  3 y 2  9 
dy
1  4 x   4 y  1
dx

Differentiate both sides with respect to x dy 4 y  1


  (Answer)
dx 1  4 x
d 2

x  2 xy  3 y 2 d
9
dx dx
(d) If x 2 y  xy 2  2 find the value of
dy
at the
dx

d 2
dx
 
x  2 xy  
d
dx
d
dx
3y2  0   point (2,1)

Given that
 dy d  dy
 2 x   2 x  y (2 x)   3.2 y  0
 dx dx  dx x 2 y  xy 2  2

 dy  dy Differentiate both sides with respect to x


 2 x   2 x  y.2   6 y  0
 dx  dx d 2
dx

x y  xy 2 
d
dx

2 
dy dy
 2x  2x  2y  6y 0
dx dx

d 2
dx

x y 
d

dx
 
xy 2  0

dy
 2 x  6 y   2 y  2 x
dx
 x2
d
 
 y   y d ( x 2 )  x d y 2  y 2 d ( x)  0
dy 2 y  x 
dx dx dx dx
 
dx 23 y  x   x2
dy
 y.2 x   x.2 y
dy
 y 2 .1  0
dx dx
dy yx
  (Answer) dy dy
dx 3y  x  x2  2 xy  2 xy  y2  0
dx dx
(c) Given that

dy 2
dx
 
x  2 xy   y 2  2 xy
1 1
 4
x y dy  y 2  2 xy
  2
dx x  2 xy
yx
 4
xy At the point (2,1): x = 2, y = 1

 y  x  4 xy

29 | P a g e
Module 03
Derivative
dy  12  2.2.1  5 dy
 2  (Answer) 1. Find for the following curves [(a) to (c)]:
dx 2  2.2.1 8 dx
2𝑥+𝑦
(a) = 𝑥𝑦
(e) If 5x y  2 x y  3xy  xy  5
2 2 2 2
find
dy
at the 𝑥−𝑦
dx
(b) 5𝑥 3 𝑦 2 + 𝑥 4 𝑦 3 = 71
point (1,2).
(c) 2𝑥𝑦 2 − 3𝑥 2 𝑦 = 20
Given that
2. Find the gradient of the tangent to the curve
5 x 2 y  2 x 2 y 2  3xy 2  xy  5
x 3 y  2 x 2 y 2  xy 3  14 at the point (1, -2).
Differentiate both sides with respect to x Rolle’s Theorem:
d
dx

5 x 2 y  2 x 2 y 2  3 xy 2  xy 
d
dx
5  Let 𝑓 be continuous on the closed interval [𝑎, 𝑏] and
differentiable on the open interval (𝑎, 𝑏). If 𝑓(𝑎) = 0
and 𝑓(𝑏) = 0 then there is at least one point 𝑐 in the

d
dx

5x 2 y 
d

dx

2x2 y 2 
d
dx
 d
dx

3 xy 2   xy   0  interval (𝑎, 𝑏) such that 𝑓 ′ (𝑐) = 0.

Proof:
 5x 2
d
 
 y   y d (5 x 2 )  2 x 2 d y 2  y 2 d (2 x 2 ) We will divide the proof into three cases: (i) the case
dx dx  dx dx 
where 𝑓(𝑥) = 0 for all 𝑥 in (𝑎, 𝑏), (ii) the case where
 3x
d 2
  d  d d 
y  y 2 (3 x)   x  y   y ( x)  0 𝑓(𝑥) > 0 at some point in (𝑎, 𝑏) and (iii) the case
dx dx  dx dx 
where 𝑓(𝑥) < 0 at some point in (𝑎, 𝑏).
dy  dy  Case I: If 𝑓(𝑥) = 0 for all 𝑥 in (𝑎, 𝑏), then 𝑓 ′ (𝑐) = 0
 5x 2  y (10.x)  2 x 2 .2 y  y 2 (4 x)
dx  dx  at every point 𝑐 in (𝑎, 𝑏) because 𝑓 is a constant
function on that interval.
dy  dy 
 3 x.2 y  y 2 (3)   x  y.(1)  0
dx  dx  Case II: Assume that 𝑓(𝑥) > 0 at some point in
(𝑎, 𝑏). Since 𝑓 is continuous on [𝑎, 𝑏], it follows from
dy dy the Extreme-Value Theorem that 𝑓 has an absolute
 5x 2  10 xy  4 x 2 y  4 xy 2
dx dx maximum on [𝑎, 𝑏]. The absolute maximum value
dy dy cannot occur at an endpoint of [𝑎, 𝑏] because we have
 6 xy  3y 2  x  y  0
dx dx assumed that 𝑓(𝑎) = 𝑓(𝑏) = 0, and that 𝑓(𝑥) > 0 at
some point in (𝑎, 𝑏). Thus, the absolute maximum

dy
 
5 x 2  4 x 2 y  6 xy  x  10 xy  4 xy 2  3 y 2  y must occur at some point 𝑐 in (𝑎, 𝑏), this critical point
dx must be a stationary point; that is 𝑓 ′ (𝑐) = 0.

dy  10 xy  4 xy 2  3 y 2  y Case III: Assume that 𝑓(𝑥) < 0 at some point in


  (𝑎, 𝑏). The proof of this case is similar to case II and
dx 5 x 2  4 x 2 y  6 xy  x
will be omitted.
At the point (1,2): x = 1, y = 2
The Mean-Value Theorem:
dy  10.1.2  4.1.2  3.2  2  20  16  12  2  14
2 2
7
   Let 𝑓 be continuous on the closed interval [𝑎, 𝑏] and
dx 5.1  4.1 .2  6.1.2  1
2 2
5  8  12  1 8differentiable
4 on the open interval (𝑎, 𝑏). Then there is
(Answer) at least one point 𝑐 in (𝑎, 𝑏) such that
Try yourself:

30 | P a g e
Module 03
Derivative
𝑓(𝑏) − 𝑓(𝑎)
𝑓 ′ (𝑐) =
𝑏−𝑎
Proof:

Let 𝑦 = 𝑓(𝑥) and the secant line joining (𝑎, 𝑓(𝑎)) and
(𝑏, 𝑓(𝑏)).

Since the two-point form of the equation of the secant


line joining (𝑎, 𝑓(𝑎)) and (𝑏, 𝑓(𝑏)) is

𝑓(𝑏) − 𝑓(𝑎)
𝑦 − 𝑓(𝑎) = (𝑥 − 𝑎)
𝑏−𝑎
𝑓(𝑏) − 𝑓(𝑎)
⇒𝑦= (𝑥 − 𝑎) + 𝑓(𝑎) … … … (1)
𝑏−𝑎
The difference 𝑣(𝑥) between the height of the graph of
𝑓 and the height of the secant line is

𝑓(𝑏) − 𝑓(𝑎)
𝑣(𝑥) = 𝑓(𝑥) − [ (𝑥 − 𝑎)
𝑏−𝑎

+ 𝑓(𝑎)] … … … (2)

Since 𝑓(𝑥) is continuous on [𝑎, 𝑏] and differentiable


on (𝑎, 𝑏), so is 𝑣(𝑥). Moreover, 𝑣(𝑎) = 0 and 𝑣(𝑏) =
0 so that 𝑣(𝑥) satisfies the hypotheses of Rolle’s
theorem on the interval [𝑎, 𝑏]. Thus, there is a point 𝑐
in (𝑎, 𝑏) such that 𝑣 ′ (𝑐) = 0. But from equation (2)

𝑓(𝑏) − 𝑓(𝑎)
𝑣 ′ (𝑥) = 𝑓 ′ (𝑥) −
𝑏−𝑎
𝑓(𝑏) − 𝑓(𝑎)
⇒ 𝑣 ′ (𝑐) = 𝑓 ′ (𝑐) −
𝑏−𝑎
Since 𝑣 ′ (𝑐) = 0, we have

𝑓(𝑏) − 𝑓(𝑎)
0 = 𝑓 ′ (𝑐) −
𝑏−𝑎
𝑓(𝑏) − 𝑓(𝑎)
⇒ 𝑓 ′ (𝑐) =
𝑏−𝑎

31 | P a g e
Module 04
The derivative in graphing and applications
(c) 𝑓 is constant on the interval if 𝑓(𝑥1 ) = 𝑓(𝑥2 ))
Here, we will study various applications of the for all points 𝑥1 and 𝑥2 .
derivative. For example, we will use methods of
calculus to analyze functions and their graphs. In
the process, we will show how calculus and
graphing utilities, working together, can provide
most of the important information about the
behavior of functions. Another important
application of the derivative will be in the solution
of optimization (Maximum & Minimum) The following figure suggests that a differentiable
problems. function f is increasing on any interval where each
tangent line to its graph has positive slope, is
Several operations of function: decreasing on any interval where each tangent line
to its graph has negative slope, and is constant on
1. Analysis of Functions I: Increase, any interval where each tangent line to its graph has
Decrease, and Concavity zero slope.
2. Analysis of Functions III: Tangents &
Normal
3. Absolute Maxima and Minima
4. Applied Maximum and Minimum
Problems
Increasing and Decreasing Functions:
The terms increasing, decreasing, and constant are
used to describe the behavior of a function as we Therefore, Let f be a function that is continuous on
travel left to right along its graph. a closed interval [𝑎, 𝑏] and differentiable on the
For example, the function graphed in figure below open interval (𝑎, 𝑏).
can be described as increasing to the left of 𝑥 = 0, (a) If 𝑓 ′ (𝑥) > 0 for every value of 𝑥 in (𝑎, 𝑏),
decreasing from 𝑥 = 0 to 𝑥 = 2, increasing then 𝑓 is increasing on [𝑎, 𝑏].
from x = 2 to 𝑥 = 4, and constant to the right of (b) If 𝑓 ′ (𝑥) < 0 for every value of 𝑥 in (𝑎, 𝑏),
𝑥 = 4 then 𝑓 is decreasing on [𝑎, 𝑏].
(c) If 𝑓 ′ (𝑥) = 0 for every value of 𝑥 in (𝑎, 𝑏),
then 𝑓 is constant on [𝑎, 𝑏].

Concave up & Concave down: If 𝑓 is


differentiable on an open interval, then 𝑓 is said to
be concave up on the open interval if 𝑓′ is
increasing on that interval, and 𝑓 is said to be
Mathematically, Let 𝑓 be defined on an interval, concave down on the open interval if 𝑓′ is
and let 𝑥1 and 𝑥2 denote points in that interval. decreasing on that interval.
(a) 𝑓 is increasing on the interval if 𝑓 (𝑥1 ) < Mathematically, Let 𝑓 be twice differentiable on
𝑓 (𝑥2 ) whenever 𝑥1 < 𝑥2 . an open interval.
(b) 𝑓 is decreasing on the interval if 𝑓 (𝑥1 ) > (a) If 𝑓 ′′ (𝑥) > 0 for every value of x in the open
𝑓 (𝑥2 ) whenever 𝑥1 < 𝑥2 . interval, then f is concave up on that interval.
Module 04
The derivative in graphing and applications
(b) If 𝑓 ′′ (𝑥) < 0 for every value of x in the open
interval, then f is concave down on that
interval.

Property: dy
Property: 0
dy dx
0
dx
Inflection points: Turning/critical point
Inflection points are points where the function
changes concavity, i.e. from being "concave up"
to being "concave down" or vice versa. They can
be found by considering where the second
derivative changes signs.

dy
Property: 0
dx
Concave up Concave down function
function

Critical point (or Stationary point):


The concept of critical point is very important in
Calculus as it is used widely in solving
optimization problems.
Let f (x) be a function and let c be a point in the
domain of the function. The point c is called
Property: d2y
a critical point of f if either f '(c) = 0 or f '(c) does
Property: 0
not exist. d2y dx 2
0
dx 2
Several figure on fluctuation of function:
Inflection point
Increasing Decreasing function
function:
Module 04
The derivative in graphing and applications
d2y d3y dy d  64 
Properties: (i)  0 and (ii) 0    4x  
dx 2 dx 3 dx dx  x 
Local maximum

d
(4 x) 
d

64.x  1 
dx dx
 4  64 1.x  1  1
 4  64 x  2
64
4 2
x
For 𝑥 = 3
dy 64
  4  2  4  7.1  3.1  0
dx 3
Absolute maximum: Maximum of the local Since dy  0, at x  3, the given function is
dx
maximums.
decreasing at 𝑥 = 3.
2
d y
Properties: (i)
dy
 0 and (ii) 0
dx dx 2 Problem 2: Find the intervals on which 𝑓 (𝑥) =
𝑥 2 − 4𝑥 + 3 is increasing and the intervals on
Local minimum which it is decreasing.

Solution.
The graph of f in the following figure suggests that
f is decreasing for 𝑥 ≤ 2 and increasing for
𝑥 ≥2.
To confirm this, we analyze the sign of 𝑓 ′ .
The derivative of f is
𝑓′ (𝑥) = 2𝑥 − 4 = 2(𝑥 − 2)
It follows that 𝑓 ′ (𝑥) < 0 𝑖𝑓 𝑥 <
Absolute minimum: Minimum of the local 2 and 𝑓′ (𝑥) > 0 𝑖𝑓 2 < 𝑥
minimums. Since f is continuous everywhere, it follows that
2
d y f is
Properties: (i)
dy
 0 and (ii) 0
dx dx 2 decreasing on (−∞, 2]
f is
increasing on [2, +∞)
Examples on Fluctuation of function: These conclusions are consistent with the graph of
Problem 1: Test whether the following functions in Figure.
are increasing or decreasing.
64
f ( x)  4 x  at x  3
x
Solution:
64
Given function y  f ( x)  4 x 
x
Module 04
The derivative in graphing and applications

Problem 3: Find the intervals on which 𝑓 (𝑥) =


𝑥 3 is increasing and the intervals on which it is
decreasing.
Solution.
The graph of 𝑓 in the following Figure suggests
that f is increasing over the entire x-axis.
To confirm this, we differentiate f to obtain
𝑓′ (𝑥) = 3𝑥 2 .
∴ 𝑓 ′ (𝑥) > 0 𝑖𝑓 𝑥 < 0
𝑓′ (𝑥) > 0 𝑖𝑓 0 < 𝑥
Since f is continuous everywhere,
f is increasing on (−∞, 0]
f is increasing on [0, +∞)
Since 𝑓 is increasing on the adjacent intervals (−∞,
0] and [0, +∞), it follows that f is increasing on
their union (−∞, +∞).

Problem 5: Find the intervals on which 𝑓 (𝑥) =


6 − 9𝑥 − 𝑥 2 is increasing and the intervals on
which it is decreasing.
Solution:

Problem 4: Find the intervals on which 𝑓 (𝑥) =


𝑥 2 + 2𝑥 − 5 is increasing and the intervals on
which it is decreasing.
Solution:
Module 04
The derivative in graphing and applications

Problem 7: Find the intervals on which 𝑓(𝑥) =


5 + 36𝑥 + 3𝑥 2 − 2𝑥 3 is increasing and the
intervals on which it is decreasing.

Problem 6: Find the intervals on which 2𝑥 3 −


12𝑥 2 + 18𝑥 + 15 is increasing and the intervals
on which it is decreasing.
Solution:
Problem 8: Show that 𝑓(𝑥) = 𝑐𝑜𝑠 2 𝑥 is decreasing
𝜋
function on (0, 2 ).
Solution:
Module 04
The derivative in graphing and applications
d2y

d

3x 2  30 x  75 
dx 2 dx
 32 x   30.1  0
 6 x  30
At 𝑥 = −8
d2y
 6 8  30  48  30  18  0
dx 2
d2y
Since  0 for x=-8, the given function is
dx 2
concave down at x=-8.
Problem 2: Test whether the following functions
are concave up or concave down.
f ( x)  12  6 x 2  x3 at x  3
Exercises:
Problem 1: Find the intervals on which 𝑓 (𝑥) = Solution: Given,
3𝑥 4 + 4𝑥 3 − 12𝑥 2 + 2 is increasing and the f ( x)  12  6 x 2  x3
intervals on which it is decreasing.
 f ' ( x)  12 x  3x 2
Problem 2: Find the intervals on which 2𝑥 3 −
21𝑥 2 + 60𝑥 − 30 is increasing and the intervals  f '' ( x)  12  6 x
on which it is decreasing. Now,
Problem 3: Find the intervals on which 10 − 6𝑥 −
f '' ( x)  12  6 x
2𝑥 2 is increasing and the intervals on which it is
decreasing.
At x  3
Examples on Concavity test: f '' (3)  12  6(3)
 12  18
Problem 1: Test whether the following functions  30  0
are concave up or concave down.
Since f  0 for x=-3, the given function is
''

f ( x)  x  15 x  75 x at x  8
3 2
concave up at x=-3.
Solution:
Given function:
y  f ( x)  x 3  15x 2  75x
dy d 3

dx dx
x  15 x 2  75 x 

d 3
dx
 
x 
d
dx
 
15 x 2  75 x 
d
dx
 3x  15.2 x   75
2

 3x 2  30 x  75
Module 04
The derivative in graphing and applications
Problem 3: Find the interval(s) where the
1
following function is concave down. 𝑦 = 3 𝑥 3 +
2𝑥 2 − 5𝑥 − 6.

Solution:

Exercises:
Problem 1: For the function𝑓(𝑥) = 𝑥 3 −
6𝑥 2 + 9𝑥 + 30, determine all intervals where f
is concave up and all intervals where f is concave
down. List all inflection points for f.

Problem2: Test whether the following functions


are concave up or concave down.
f ( x)  x3  3x 2  4 at x  5

Examples on Inflection point:

Problem 4: Find the interval(s) where the Problem 1: Determine the inflection point for the
1 given function 𝑓(𝑥) = 𝑥 4 – 24𝑥 2 + 11
following function is concave down. 𝑦 = 3 𝑥 3 −
5𝑥 2 + 9𝑥 + 5. Solution:
Given function: 𝑓(𝑥) = 𝑥 4 – 24𝑥 2 + 11
Solution:
The first derivative of the function is
𝑓’(𝑥) = 4𝑥 3 – 48𝑥
The second derivative of the function is
𝑓”(𝑥) = 12𝑥 2 – 48
Set 𝑓”(𝑥) = 0,
Module 04
The derivative in graphing and applications
12𝑥 2 – 48 = 0 d2y d
2

dx

3x 2  6 x  3
Divide by 12 on both sides, we get dx
 32 x   6.1  0
𝑥2 – 4 = 0
 6x  6
𝑥2 = 4 d3y d
 6 x  6  6.1  0  6  0
Therefore, 𝑥 = ± 2 dx 3 dx
To check or 𝑥 = 2, substitute 𝑥 = 1 and 3 in d3y
Since  0 , so inflection point obtained for
𝑓”(𝑥) dx 3
d2y
So, 𝑓”(1) = 12. 12 – 48 = −36 (negative) 0
dx 2
𝑓”(3) = 12. 32 – 48 = 276 (positive)  6x  6  0
To check for 𝑥 = −2, substitute 𝑥 = 0 and -3 in  6x  6
𝑓”(𝑥)  x 1
Putting x=-1 in (i) we have ,
So, 𝑓”(0) = 12. 02 – 48 = −48 (negative)
2
y  13  3.12  3  1  3  3  1
𝑓”(3) = 123 – 48 = 276 (positive)
Inflection point is (1,1)
Hence, proved
Exercises
Now, substitute 𝑥 = ± 2 in 𝑓”(𝑥)
Therefore, it becomes Problem 1: Determine the inflection point for the
given function 𝑓(𝑥) = 3𝑥 2 − 9𝑥 + 6.
𝑓”(2) = 12. 22 – 48 = −69
Problem 2: Determine the inflection point for the
𝑓”(−2) = 12. (−2)2 – 48 = −69
given function 𝑓(𝑥) = 2𝑥 3 – 12𝑥 2 + 4𝑥 − 27.
Therefore, the inflection points are (2, −69), and
Problem 3: Determine the inflection point for the
(−2, −69).
given function 𝑓(𝑥) = 𝑥 4 – 2 = 𝑥 2 + 1

Problem 2: Given the function y  x  3x  3x ,


3 2
Examples on Critical points
find the point of inflexion. Problem 1: Find the critical points of the function
𝑓(𝑥) = 𝑥 3 + 3𝑥 2 − 24𝑥.
Solution:
Solution:
Given function: y  x  3x  3x -------------(i)
3 2

Given, 𝑓(𝑥) = 𝑥 3 + 3𝑥 2 − 24𝑥.


dy d 3

dx dx

x  3x 2  3x  At first we should find the first derivative of 𝑓(𝑥).

d 3
dx
 
x 
d
dx
 
3x 2 
d
dx
3x  Now,

 3 x 2  3.2 x   3 𝑓(𝑥) =
 3x 2  6 x  3 𝑥 + 3𝑥 2 − 24𝑥.
3

∴ 𝑓′(𝑥) = 3𝑥 2 + 6𝑥 − 24.
We set 𝑓 ′ (𝑥) = 0 to find critical point
Module 04
The derivative in graphing and applications
∴ 3𝑥 2 + 6𝑥 − 24 = 0 Problem 2: Find the critical points of the function
𝑓(𝑥) = 𝑥 + 𝑒 −𝑥 .
⇒ 𝑥 2 + 2𝑥 − 8 = 0
⇒ 𝑥 2 + 4𝑥 − 2𝑥 − 8 = 0
Tangent and Normal:
⇒ 𝑥(𝑥 + 4) − 2(𝑥 + 4) = 0 Tangent and Normal Lines:
∴ 𝑥 = −4, 2 The derivative of a function has many applications
to problems in calculus. It may be used in curve
Therefore the critical points for the given function sketching; solving maximum and minimum
are −4 and 2. problems; solving distance; velocity, and
See the following figure. acceleration problems; solving related rate
problems; and approximating function values.

The derivative of a function at a point is the slope


of the tangent line at this point. The normal
line is defined as the line that is perpendicular to
the tangent line at the point of tangency. Because
the slopes of perpendicular lines (neither of which
is vertical) are negative reciprocals of one another,
the slope of the normal line to the graph of f(x) is
Problem 2: Find the critical points of the function −1/ f′(x).
𝑓(𝑥) = 𝑥 5 − 5𝑥 4 + 5𝑥 3 − 1.
Define Tangent and Normal: Let, P(x,y) be any
Solution: Given, 𝑓(𝑥) = 𝑥 5 − 5𝑥 4 + 5𝑥 3 − 1. on the curve APB whose equation y=f(x). Let, the
At first we should find the first derivative of 𝑓(𝑥). tangent and the normal at P meet the x-axis in T
and G respectively. Draw PN perpendicular a x-
Now,𝑓(𝑥) = 𝑥 5 − 5𝑥 4 + 5𝑥 3 − 1. axis.
∴ 𝑓 ′ (𝑥) = 5𝑥 4 − 20𝑥 3 + 15𝑥 2 .
We set 𝑓 ′ (𝑥) = 0 to find critical point

∴ 5𝑥 4 − 20𝑥 3 + 15𝑥 2 = 0
⇒ 5𝑥 2 (𝑥 2 − 4𝑥 − 3) = 0
⇒ 5𝑥 2 = 0 and (𝑥 2 − 4𝑥 − 3) = 0
⇒ 𝑥 = 0 and (𝑥 − 1)(𝑥 − 3) = 0
∴ 𝑥 = 0,1, 3 Figure

Therefore the critical points for the given function (a). The portion of tangent intercepted between the
are 0, 1 and 3. point of contact and the axis of x is called the length
of the tangent. Length of the tangent
PT  PN .Cosec
Exercises
(b).The portion of normal at any point on the curve
Problem 1: Find the critical points of the function
intercepted between the curve and the axis of x is
𝑓(𝑥) = 72𝑥 − 30𝑥 2 − 8𝑥 3 + 3𝑥 4 .
Module 04
The derivative in graphing and applications
called the length of the normal. Length of the At the point (−1,2), 𝑓′(−1) = −½ and the
normal PG  PN .S ec equation of the line is

Problem 1: Find the equation of the tangent and


normal to the curve y  x  2 x  1 at the point
2

(1,4).

Solution: Given that, y  x  2 x  1


2

Problem 3: Find the equation of the normal line


dy
  2x  2 to the graph of at the point (−1, 2).
dx

dy Solution: From problem 2, you find


At point (1,4), 224 that 𝑓′(−1) = −½ and the slope of the normal
dx
line is −1/ 𝑓′(−1) = 2; hence, the equation of
Equation of the tangent at the point of (1,4) is the normal line at the point (−1,2) is

dy
 y  y1  (x  x1 )
dx
 y  4  4(x  1)
4 x y  0

Equation of the normal at the point of (1,4) is


Problem 4: Find the equation of the tangent and
normal to the curve y  x  3x  9 x  15 at the
3 2
1
 y  y1  (x  x1 )
dy point (-3,-1).
dx
1 Solution: Given that, y  x  3x  9 x  15
3 2
 y  4  (x  1)
4
 4 y  16   x  1 dy
  3x 2  6 x  9
 x  4 y  17  0 dx

Problem 2: Find the equation of the tangent line dy


At point (3,-1),  27  18  9  36
to the graph of at the point (−1,2). dx

Solution: Equation of the tangent at the point of (−3, −1)


is

dy
 y  y1  (x  x1 )
dx
 y  1  36(x  3)
 36 x  y 107  0
Module 04
The derivative in graphing and applications
Equation of the normal at the point of (−3, −1) Solution: Given that, y  x  x  1
4 3

is
dy
1   4 x 3  3x 2
 y  y1  (x  x1 ) dx
dy
dx dy
1
At point (1,3)  4.13  3.12  7
 y  1  (x  3) dx
36
 36 y  36   x  3 Equation of the tangent at the point of (1,3) is
 x  36 y  39  0
dy
 y  y1  (x  x1 )
Problem-5: Find the equation of the tangent and dx
 y  3  7( x  1)
normal to the curve y  5 x  x  1 at the point
2

 y 3  7 x  7
(1,-2)
 y 7 x  4  0
Solution: Given that, y  5 x  x  1
2

Equation of the normal at the point of (1,3) is


dy
  10 x  1
dx 1
 y  y1  (x  x1 )
dy
dy
At point (1,-2),  10  1  11 dx
dx
1
 y  4  ( x  1)
at the point of (1, −2) is 7
Equation of the tangent
 7 y  28   x  1
dy  7 y  x  29  0
 y  y1  (x  x1 )
dx
 y  2  11( x  1) Problem-7: Find the equation of the tangent and
normal to the curve y  x  2 x  1 at the point
3
 y  2  11x  11
 y  11x  13  0 (2,3).
Solution: Given that, y  x  2 x  1
3

Equation of the normal at the point of (1, −2) is


dy
1   3x 2  2
 y  y1  (x  x1 ) dx
dy
dx dy
At point (2,3),  3.22  2  3.4  2  14
1 dx
 y  2  ( x  1)
11
 11y  22   x  1 Equation of the tangent at the point of (2,3) is
11y  x  21  0

Problem-6: Find the equation of the tangent and


normal to the curve y  x  x  1 at the point
4 3

(1,3).
Module 04
The derivative in graphing and applications
dy The maximum value will be the absolute maximum
 y  y1  (x  x1 )
dx or the greatest, whereas the minimum value will be
 y  3  14( x  2) the absolute minimum or the least value of the
 y  3  14 x  28 function.
The diagram below shows part of a function y =
 y  14 x  25  0
f(x)
Equation of the normal at the point of (2,3) is

1
 y  y1  (x  x1 )
dy
dx
1
 y  3  ( x  2)
14
 14 y  42   x  2
14 y  x  44  0 The point A is a local maximum and the point B is
a local minimum. At each of these points the
Exercises tangent to the curve is parallel to the x-axis so the
derivative of the function is zero. Both of these
Problem 1: Find the equation of the tangent and points are therefore stationary points of the
normal to the curve y  3x  4 x  1 at the
3 2
function. The term local is used since these points
point (-1,3). are the maximum and minimum in this particular
region.
Problem 2: Find the equation of the tangent and At a critical point, a function could have a local
normal to the curve y  2 x  3x  1 at the
4 2 maximum or a local minimum or neither.

point (1,-2).
Condition of Maximum and Minimum values of
a function:
Problem 3: Find the equation of the tangent and
𝑑𝑦 𝑑2 𝑦
For Maximum value: (𝑖) 𝑑𝑥 = 0 , (𝑖𝑖) 𝑑𝑥 2 < 0
normal to the curve y  x  3x  9 at the point
3

𝑑𝑦 𝑑2 𝑦
(1,7). For Minimum value: (𝑖) 𝑑𝑥 = 0 , (𝑖𝑖) 𝑑𝑥 2 > 0

Maximum and Minimum values of a function Problem-1: Find the maximum and minimum
values of the function 2𝑥 3 − 21𝑥 2 + 36𝑥 − 20
In calculus, we might have learnt how to find the
local maximum and local minimum values of a Solution: Let 𝑦(𝑥) = 2𝑥 3 − 21𝑥 2 + 36𝑥 − 20
function using the first derivative test and second 𝑑𝑦
derivative test. As we know, the function has = 6𝑥 2 − 42𝑥 + 36
𝑑𝑥
neither a local maximum value nor a local When 𝑦(𝑥) is a maximum or a minimum,
minimum value. In this article, we will learn how 𝑑𝑦
to find the minimum and maximum values of a =0
𝑑𝑥
function. => 6𝑥 2 − 42𝑥 + 36 = 0
=> 6(𝑥 2 − 7𝑥 + 6) = 0
Definition: =>𝑥 2 − 7𝑥 + 6 = 0
=> 𝑥 2 − 6𝑥 − 𝑥 + 6 = 0
Module 04
The derivative in graphing and applications
=>𝑥(𝑥 − 6) − 1(𝑥 − 6) = 0 𝑦(1) = 2. 13 − 9. 12 + 12.1 − 3
=>(𝑥 − 6)(𝑥 − 1) = 0 = 2 − 9 + 12 − 3
Hence 𝑥 = 6 𝑜𝑟 1 = 2(Ans.)
𝑑2 𝑦 And also 𝑦(𝑥)is a minimum at 𝑥 = 2 . Then
Again ,𝑑𝑥 2 = 12𝑥 − 42
the minimum value is
𝑑2 𝑦
When 𝑥 = 1 , 𝑑𝑥 2 = 12.1 − 42 = −30 < 0 𝑦(2) = 2. 23 − 9. 22 + 12.2 − 3
(maximum) = 16 − 36 + 24 − 3
𝑑2 𝑦 = 1(Ans.)
When 𝑥 = 6 , 𝑑𝑥 2 = 12.6 − 42 = 30 > 0
Problem 3: Find the maximum and minimum
(minimum)
values of the following functions:
i.e. 𝑦(𝑥) is a maximum at 𝑥 = 1 . Then the
x3
maximum value is f ( x)   x 2  8 x  4
𝑦(1) = 2. 13 − 21. 12 + 36.1 − 20 3
= 2 − 21 + 36 − 20 Solution:
= −3 (Ans.) x3
Given function: y  f ( x)   x 2  8 x  4 -------
And also 𝑦(𝑥)is a minimum at 𝑥 = 6 . Then 3
the minimum value is ------(i)
𝑦(6) = 2. 63 − 21. 62 + 36.6 − 20 dy d  x 3 
= 432 − 756 + 216 − 20    x 2  8 x  4 
dx dx  3 
= −128 (Ans.)
d x  d 2
 
3
x  8 x   4
d d
   
Problem 2: Find the maximum and minimum dx  3  dx dx dx
values of the function 2𝑥 3 − 9𝑥 2 + 12𝑥 − 3 1
 .3x 2  2 x  8.1  0
Solution: Let 𝑦(𝑥) = 2𝑥 3 − 9𝑥 2 + 12𝑥 − 3 3
𝑑𝑦  x 2  2x  8
= 6𝑥 2 − 18𝑥 + 12
𝑑𝑥
When 𝑦(𝑥)is a maximum or a minimum , d2y d 2
 
x  2x  8 
𝑑𝑦 dx 2 dx
=0
𝑑𝑥  2 x  2 .1  0
=> 6𝑥 2 − 18𝑥 + 12 = 0
 2x  2
=> 6(𝑥 2 − 3𝑥 + 2) = 0
=>𝑥 2 − 3𝑥 + 2 = 0
For local maximum or local minimum value of y
=> 𝑥 2 − 2𝑥 − 𝑥 + 2 = 0
we have,
=>𝑥(𝑥 − 2) − 1(𝑥 − 2) = 0
dy
=>(𝑥 − 2)(𝑥 − 1) = 0 0
Hence𝑥 = 2 𝑜𝑟 1 dx
𝑑2 𝑦  x2  2x  8  0
Again ,𝑑𝑥 2 = 12𝑥 − 18
 x2  4 x  2 x  8  0
𝑑2 𝑦
When 𝑥 = 1 , 𝑑𝑥 2 = 12.1 − 18 = −6 < 0  x  x  4  2  x  4  0
(maximum)   x  4  x  2   0
𝑑2 𝑦
When 𝑥 = 2 , 𝑑𝑥 2 = 12.2 − 18 = 6 > 0 Either x  4  0 or x  2  0
(minimum)  x  4 or x  2
i.e. 𝑦(𝑥) is a maximum at 𝑥 = 1 . Then the
maximum value is
Module 04
The derivative in graphing and applications
d2y
For x  4,  2 4  2  6  0
dx 2
d2y
And for x  2,  22  2  6  0
dx 2
So for x  4 , given function has a maximum
value and the maximum value of y from (i)
(4) 3 92
y max   (4) 2  8.(4)  4  (Answer)
3 3
Again for x  2 , given function has a minimum
value and the minimum value of y from (i) Maximum and Minimum values of a function
(2) 3 16 in a closed interval:
y min   (2) 2  8.(2)  4   (Answer)
3 3 Consider the below-given graph of a continuous
Problem 4: Find the stationary point of the function f(x) defined on a closed interval [a, b].
function 𝑦 = 𝑥 2 − 2𝑥 + 3 and hence
determine the nature of this point.
Solution:
Given,
𝑦 = 𝑥 2 − 2𝑥 + 3
∴ 𝑑𝑦/ 𝑑𝑥 = 2𝑥 − 2
And 𝑑 2 𝑦/ 𝑑𝑥 2 = 2.
From the above graph, we can write the below
𝑑𝑦 points.
Now 𝑑𝑥 = 2𝑥 − 2 = 0 when 𝑥 = 1.

The function has only one stationary point when  We can observe that the function f has a
𝑥 = 1 (and 𝑦 = 2). local minimum at x = b, and the local
minimum value is f(b). Also, it has local
𝑑2 𝑦
Since 𝑑𝑥 2 = 2 > 0 for all values of 𝑥, this maxima at x = c, and the local maximum
stationary point is a local minimum. value is f(c).
 Besides, we can identify the absolute
Thus the function 𝑦 = 𝑥 2 − 2𝑥 + 3 has a
maximum value f(a) and absolute minimum
local minimum at the point (1, 2).
value f(d) from the graph that f has.
The graph shows the function 𝑦 = 𝑥 2 − 2𝑥 +  Moreover, the absolute maximum or
3 with the local minimum point at (1,2) clearly minimum value of f varies from the local
visible. maximum or minimum value of the
function f in the closed interval.

Problem 4: Find both the maximum value and the


minimum value of the function f(x) = 3x4 – 8x3 +
12x2 – 48x + 25 on the closed interval [0, 3].
Module 04
The derivative in graphing and applications
Solution: Problem 2: Find the maximum and minimum
values of the following functions:
Given,
f(x) = 3x4 – 8x3 + 12x2 – 48x + 25 2 3 1 2
f ( x)  x  x  6 x  8 (i)
3 2
Interval: [0, 3] 50
Ans. Maximum value is and minimum value is
𝑑 3
f'(x) = 𝑑𝑥 [3x – 8x + 12x – 48x + 25]
4 3 2
19
.
8
= 12x – 24x + 24x – 48
3 2
Problem 3: Find the maximum and minimum
values of the following functions:
= 12(x3 – 2x2 + 2x – 4)
= 12(𝑥 – 2)(𝑥2 + 2) f ( x)  x 3  3 x 2  3 x  5
Consider 𝑓′(𝑥) = 0 Problem 4: Find the stationary point of the
function 𝑦 = 3𝑥 2 − 4𝑥 + 7 and hence
12(𝑥 – 2)(𝑥2 + 2) = 0
determine the nature of this point.
𝑥 – 2 = 0 𝑜𝑟 𝑥2 + 2 = 0
𝑥 = 2
As we know x2 + 2 = 0 does not have real roots.
So, we consider only x = 2 ∈ [0, 3].
Also, the endpoints of the interval are x = 0 and x
= 3.
Now, we have to find the value of f(x) at x = 0, 2,
and 3.
𝑓(0) = 3(0)4 – 8(0)3 + 12(0)2 – 48(0) + 25 = 25

𝑓(2) = 3(2)4 – 8(2)3 + 12(2)2 – 48(2) + 25

= 48 – 64 + 48 – 96 + 25 = −39

𝑓(3) = 3(3)4 – 8(3)3 + 12(3)2 – 48(3) + 25


= 243 – 216 + 108 – 144 + 25
= 16

Therefore, the absolute maximum value of f on [0,


3] is 25 occurring at x = 0 and the minimum value
of f on [0, 3] is –39 occurring at x = 2.
Exercises
Problem 1: Find the maximum and minimum
values of the following functions:
x3 2
f ( x)   x  3 x  7
3
92
Ans: Maximum value is and minimum value is
3
16
.
3
Module-5
Application of Differentiation

d2A
Since  0 , so maximum value of A obtained for
Problem 1: A farmer has an adjustable fence that is 100m long. dx 2
He uses this fence to enclose a rectangular grazing area on three
sides, the fourth side being a fixed hedge. Find the maximum dA
area he can enclose.
0
dx
Solution: Let x and y are the dimensions of the rectangular  4 x  100  0
grazing area and A is the area of the rectangle which are  4 x  100
shown in the following figure.
 x  25
Putting x  25 in (i)
y  100  2.25  100  50  50

 The maximum area can be enclosed,

Amax  25  50 m 2 = 1250 sq. meters (Ans.)

Problem 2: A rectangular plot of land is to be enclosed by a


fence. Fence for the east-west sides costs Tk.100 per running
Since the total length of the fence is 100m. foot, while that for the north-south sides costs Tk.50 per
According to the question we have running foot. What is the maximum area that can be enclosed if
y  100  2 x ------------------------(i) Tk.15000 is available for purchasing the fence?

and Solution: Let the dimension of the east-west sides is x feet


and the dimension of the north-south sides is y feet.
 A  x. y
 A  x100  2 x 
 A  2 x 2  100.x

dA d

dx dx

 2 x 2  100 x 
  2.(2 x)  100.1
 4 x  100
Cost of fencing of east-west sides = (x+x)  100=200x
d2A d
  4 x  100  Cost of fencing of north-south sides = (y+y)  50=100y
dx 2 dx
  4.1  0
 4  0 According to the question

200x+100y=15000

47 | P a g e
Module-5
Application of Differentiation

 2x+y=150 Solution: Let x


inch, x inch and y
 y=150-2x ----------------------(1) inch are the
dimensions of the
Let the area of the rectangle is ‘A’
required box.
 A=x.y=x.(150-2x) [from (1)]
According to the
=150x-2x2 question:

Now

dA d

dx dx

150 x  2 x 2  d2A d
 150  4 x 
dx 2 dx
 150  2.2 x   04
 150  4 x
 4  0
Volume of the box  x.x. y  2592

d2A  x 2 y  2592
Since  0 , so the maximum value of A will be obtained
dx 2
for 2592
y ---------------(i)
x2
dA
0 Cost of the materials of the top and bottom=
dx
 150  4 x  0 x 2

 x 2 .9  18 x 2
 4 x  150
Cost of the materials of the remaining four sides=
x
150
 37.5 xy  xy  xy  xy .6  24 xy
4
Now let C is the total cost of the materials of the box.
From (i): y  150  2  37.5  150  75  75
C  18 x 2  24 xy
Maximum area can be enclosed:
Amax  37.5  75 sq. feet ( Answer )  2592 
 18 x 2  24 x. 2  [From (i)]
Problem 3: Suppose that we require a box of volume 2592
 x 
cubic inches with square top and bottom and rectangular sides. 62208
Side material costs Tk.6 per square inch, and top and bottom  18 x 2 
x
material costs Tk.9 per square inch. Find the dimensions for
which we can minimize the cost of the materials.  18x 2  62208 x 1
dC
dx

d
dx
 
18 x 2  62208.x 1  18.2 x   62208. 1x 11  36 x  62208.x  2

d 2C d
  
36 x  62208.x  2  36  62208. 2x  21  36  124416.x 3
dx 2 dx

48 | P a g e
Module-5
Application of Differentiation
𝑑𝑟
That is = 0.2
𝑑𝑡
For maximum or minimum value of C, we have
𝐴 = 𝜋𝑟 2
dC 𝑑𝐴
0 𝑑𝑟
= 2𝜋𝑟
dx
𝑑𝐴 𝑑𝐴 𝑑𝑟
 36 x  62208.x  2  0 => = ×
𝑑𝑡 𝑑𝑟 𝑑𝑡
62208
 36 x  0 = 2𝜋𝑟 × 0.2
x2
36 x 3  62208 = 0.4𝜋𝑟
 0
x2 When 𝑟 = 5,
𝑑𝐴
= 0.4 × 𝜋 × 5 = 2𝜋
𝑑𝑡
 36 x 3  62208  0
∴ The rate of change of the area = 2𝜋 𝑐𝑚2 /𝑠
 36 x 3  62208
**************
62208
 x3   1728 𝝅𝒙𝟐 (𝟑𝒂−𝒙)
36 Problem 8: The volume of water in vessel is 𝒄𝒎𝟐
𝟒
 x 3  12 3 when the depth of water in the
 x  12 vessel is 𝒙 𝒄𝒎. If water is poured into the vessel at a rate
of 𝒂𝟑 𝒄𝒎𝟑 /𝒔 . Calculate the rate at which the level of the
For x=12, 𝟑𝒂
water rises when the depth is 𝒄𝒎.
d 2C 𝟒
 36  124416 .12  3
 36 
124416
 108  0
dx 2 12 3 Solution:
𝝅𝒙𝟐 (𝟑𝒂−𝒙) 𝑑𝑣
d 2C It is given that 𝑉 = and = 𝑎3
𝟒 𝑑𝑡
Since  0 , so C is minimum for x=12.
dx 2 We need to find
𝑑𝑥
, the rate at which the water level rises
𝑑𝑡
3𝑎
2592 2592 when the depth is 𝑥 =
Putting x=12 in equation (i): y    18 4
12 2 144
𝝅𝒙𝟐 (𝟑𝒂−𝒙)
𝑉=
𝟒
 12 inch, 12 inch and 18 inch are the required dimensions
of the box. (Answer) 3𝜋𝑎𝑥 2 𝜋𝑥 3
= −
4 4

𝑑𝑣 3𝜋𝑎 𝜋
= (2𝑥 × ) − (3𝑥 2 × )
𝑑𝑥 4 4
Problem 7: The radius of a circle increases at a rate of
. 𝟐 𝒄𝒎/𝒔 .Calculate the rate of the increase of the area 6𝜋𝑎𝑥−3𝜋𝑥 2
=
when the radius is 𝟓 𝒄𝒎. 4

𝑑𝑉 𝑑𝑉 𝑑𝑥
Solution : = ×
𝑑𝑡 𝑑𝑥 𝑑𝑡

Let 𝑟 𝑐𝑚 be the radius of the circle and 𝐴 𝑐𝑚2 be the area 𝑑𝑥


𝑑𝑉
𝑑𝑡
of the circle. It is given that the rate = 𝑑𝑉
𝑑𝑡
𝑑𝑥

of change of the radius is . 2 𝑐𝑚/𝑠

49 | P a g e
Module-5
Application of Differentiation

=
𝑎3 ∴ its acceleration after 4 second is 12 𝑚/𝑠 2
6𝜋𝑎𝑥−3𝜋𝑥2
4

4𝑎3
= (iii) When the momentarily at rest ,its velocity is
3𝜋(2𝑎𝑥−𝑥 2 )
zero.
3𝑎 𝑑𝑥 4𝑎3
When 𝑥 =
4
,
𝑑𝑡
= 3𝑎 3𝑎 2 i.e. 𝟑𝒕𝟐 − 𝟏𝟐𝒕 + 𝟗 = 𝟎
3𝜋[2𝑎× −( ) ]
4 4
𝒕𝟐 − 𝟒𝒕 + 𝟑 = 𝟎
4𝑎3 (𝒕 − 𝟑)(𝒕 − 𝟏) = 𝟎
= 6𝑎2 9𝑎2
3𝜋(
4

16
) 𝒕 = 𝟑 or, 𝒕 = 𝟏

4𝑎3 The body is momentarily at rest when 𝑡 = 3 or 𝑡 = 1


= 15𝑎2
3𝜋( )
16

64𝑎
=
45𝜋 (iv) When 𝒕 = 𝟏, 𝒔 = 𝟏𝟑 − 𝟔(𝟏)𝟐 + 𝟗(𝟏) + 𝟓𝟔 =
64𝑎 𝟔𝟎
∴ The rate at which the water level rises is 𝑐𝑚/𝑠
45𝜋 When 𝒕 = 𝟑, 𝒔 = 𝟑𝟑 − 𝟔(𝟑)𝟐 + 𝟗(𝟑) + 𝟓𝟔 =
************************ 𝟓𝟔

Problem 9: A body moves in a straight line and its distance ∴ when the body is momentarily at rest , it is either at a
, s m, from fixed point 𝟎 is distance of 56 m or 60 m from 0.

given by 𝒔 = 𝒕𝟑 − 𝟔𝒕𝟐 + 𝟗𝒕 + 𝟓𝟔, where t is time in Problems: Find the maximum and minimum values of the
seconds after passing 0 .Find following functions:
x3
(i) Its velocity after 4 second (i) f ( x)   x 2  3x  7
(ii) Its acceleration after 4 seconds 3
(iii) The time when it is momentarily at rest
92 16
(iv) Its distance from 0 when it is momentarily at Ans: Maximum value is and minimum value is .
3 3
rest
2 3 1 2
Solution: (ii ) f ( x)  x  x  6 x  8 (i)
3 2
(i) 𝒔 = 𝒕𝟑 − 𝟔𝒕𝟐 + 𝟗𝒕 + 𝟓𝟔 50 19
𝒅𝒔 Ans. Maximum value is and minimum value is .
3 8
𝑽=
𝒅𝒕
= 𝟑𝒕𝟐 − 𝟏𝟐𝒕 + 𝟗 (iii) f ( x)  x 3  3x 2  3x  5(TryYoursel f )
When 𝑡 = 4, 𝑉 = 3(4)2 − 12(4) + 9

=9

∴ its velocity after 4 second is 9 m/s


𝒅𝒗
(ii) 𝒂=
𝒅𝒕
= 𝟔𝒕 − 𝟏𝟐

𝑤ℎ𝑒𝑛 𝑡 = 4 , 𝑎 = 6(4) − 12

= 12
50 | P a g e
Module 6
Partial Derivatives
In this section we concentrate on the mathematical terms partial differentiation. So to understand this term we
should have knowledge about function of several variables.

Partial Differential equation:


An equation involving partial derivative of a function of one variable with respect to two or more independent
variable is called a partial differential equation.
u u
Example: x  y u
x y
Partial Derivative Formula
If f(x,y) is a function, where f partially depends on x and y and if we differentiate f with respect to x and y then
the derivatives are called the partial derivative of f. The formula for partial derivative of f with respect to x taking
y as a constant is given by;

Calculate Partial Derivatives


We have already seen that the limit definitions are used to find the partial derivatives. But using the limit
formula and computing the limit is not always easy. Thus, we have another method to calculate partial derivatives
that follow right from its definition. In this method, if z = f(x, y) is the function, then we can compute the partial
derivatives using the following steps:
 Step 1: Identify the variable with respect to which we have to find the partial derivative.
 Step 2: Except for the variable found in Step 1, treat all the other variables as constants.
 Step 3: Differentiate the function just using the rules of ordinary differentiation.
Wait! Read Step 3 again. Yes, the rules of ordinary differentiation are as same as that of partial differentiation.
In partial differentiation, just treating variables is different, that's it!
51 | P a g e
Module 6
Partial Derivatives

Order of Partial Derivatives.


The next step is to understand the order of partial derivatives. The example given above is called a first order
partial derivative. However, it is possible to have higher order partial derivatives. A second order or double
partial derivative is found by taking the partial derivative of a function twice. For a function, f(x,y) there are two
possible second order partial derivative, shown in the image.

Example of second order partial derivatives for a function of two variables.

What is the difference between differentiation and partial differentiation?


In differentiation, the derivative of a function with respect to the one variable can be found, as the function
contains one variable in it. Whereas in partial differentiation, the function has more than one variable. Thus, the
partial derivative of the function with respect to one variable can be found by holding some variables as
constant.

Problem-1: Find 1 𝜕
= (𝑥 2 +𝑦 2).𝜕𝑥 (𝑥 2 +𝑦 2 )
f f 1
and of the following functions = (𝑥 2 +𝑦 2). 2𝑥
x y
Again given that 𝑓 = log(𝑥 2 +𝑦 2 )
(i) 𝒇 = 𝒂𝒙𝟐 + 𝟐𝒉𝒙𝒚 + 𝒃𝒚𝟐
𝜕𝑓 𝜕
(𝒊𝒊)𝒇 = 𝐥𝐨𝐠(𝒙𝟐 +𝒚𝟐 ) = 𝜕𝑦 {log (𝑥 2 +𝑦 2 )}
𝜕𝑦
(𝒊𝒊𝒊)𝒇 = 𝒙𝟑 + 𝟑𝒙𝟐 𝒚 + 𝟑𝒙𝒚𝟐 + 𝒚𝟑 1 𝜕
= (𝑥 2 +𝑦 2).𝜕𝑦 (𝑥 2 +𝑦 2 ) =
1
. 2𝑦
(𝑥 2 +𝑦 2 )
Solution of (i) : Given that 𝑓 = 𝑎𝑥 2 + 2ℎ𝑥𝑦 + Solution of (iii) : Given that 𝑓 = 𝑥 + 3

𝑏𝑦 2 3𝑥 2 𝑦 + 3𝑥𝑦 2 + 𝑦 3
𝜕𝑓 𝜕 𝜕𝑓 𝜕
= 𝜕𝑥 (𝑎𝑥 2 + 2ℎ𝑥𝑦 + 𝑏𝑦 2 ) = 𝜕𝑥 (𝑥 3 + 3𝑥 2 𝑦 + 3𝑥𝑦 2 + 𝑦 3 )
𝜕𝑥 𝜕𝑥
𝜕 𝜕 𝜕 𝜕 𝜕 𝜕 𝜕
= 𝜕𝑥 (𝑎𝑥 2 ) − 𝜕𝑥 (2ℎ𝑥𝑦) + 𝜕𝑥 (𝑏𝑦 2 ) = 𝜕𝑥 (𝑥 3 ) + 𝜕𝑥 (3𝑥 2 𝑦) + 𝜕𝑥 (3𝑥𝑦 2 ) + 𝜕𝑥 (𝑦 3 )
= 2𝑎𝑥 + 2ℎ𝑦 + 0 = 3𝑥 2 + 6𝑥𝑦 + 3𝑦 2 + 0
= = 2𝑎𝑥 + 2ℎ𝑦 = 3𝑥 2 + 6𝑥𝑦 + 3𝑦 2
Again given that 𝑓 = 𝑎𝑥 2 + 2ℎ𝑥𝑦 + 𝑏𝑦 2 Again given that 𝑓 = 𝑥 3 + 3𝑥 2 𝑦 + 3𝑥𝑦 2 +
𝜕𝑓 𝜕
= 𝜕𝑦 (𝑎𝑥 2 + 2ℎ𝑥𝑦 + 𝑏𝑦 2 ) 𝑦3
𝜕𝑦 𝜕𝑓 𝜕
𝜕 2 𝜕 𝜕 2 = 𝜕𝑦 (𝑥 3 + 3𝑥 2 𝑦 + 3𝑥𝑦 2 + 𝑦 3 )
= 𝜕𝑦 (𝑎𝑥 ) − 𝜕𝑦 (2ℎ𝑥𝑦) + 𝜕𝑦 (𝑏𝑦 ) 𝜕𝑦

52 | P a g e
Module 6
Partial Derivatives
= 2ℎ𝑥 + 2𝑏𝑦 𝜕 𝜕 𝜕 𝜕
= 𝜕𝑦 (𝑥 3 ) + 𝜕𝑦 (3𝑥 2 𝑦) + 𝜕𝑦 (3𝑥𝑦 2 ) + 𝜕𝑦 (𝑦 3 )
of (ii) : Given that 𝑓 = log (𝑥 2 +𝑦 2 )
𝜕𝑓 𝜕
= 0 + 3𝑥 2 + 6𝑥𝑦 + 3𝑦 2
2 2 )}
= 𝜕𝑥 {log(𝑥 +𝑦 = 3𝑥 2 + 6𝑥𝑦 + 3𝑦 2
𝜕𝑥

Problem-2: Show that 𝒇(𝒙 + 𝒚) = 𝐥𝐧( 𝒙𝟐 + 𝒚𝟐 ) Problem-3: If 𝑢 = 𝑥 2 + 𝑦 2 ln 𝑥 + 2𝑒 −𝑥 𝑦


𝛿2 𝑢 𝛿2 𝑢
2 f 2 f then find and .
satisfies the P.D.E   0. 𝛿𝑥 2 𝛿𝑦 2
x 2 y 2
Solution: Given, 𝑢 = 𝑥 2 + 𝑦 2 ln 𝑥 +
2𝑒 −𝑥 𝑦 … … (1)
Solution:
Differentiating (1) partially with respect to 𝑥
we get,
Given that , 𝑓(𝑥 + 𝑦) = ln( 𝑥 2 + 𝑦 2 )
𝛿𝑢 𝛿
= 𝛿𝑥 (𝑥 2 + 𝑦 2 ln 𝑥 + 2𝑒 −𝑥 𝑦)
𝛿𝑥
Differentiating equation (1) partially w.r.t x & y, we 𝑦2
= 2𝑥 + − 2𝑒 −𝑥 𝑦 … …. (2)
get, 𝑥
Now differentiating (2) partially with respect
f 2x to 𝑥 we get,
 2 ..............(2) 𝛿2 𝑢 𝛿 𝑦2
x x  y 2 𝛿𝑥 2
= 𝛿𝑥 (2𝑥 + 𝑥
− 2𝑒 −𝑥 𝑦)
f 2y 𝑦2
= 2 − 𝑥 2 + 2𝑒 −𝑥 𝑦
 2 ..............(3)
y x  y 2
Again differentiating (1) partially with respect
to 𝑦 we get,
Again, Differentiating equation (2) & (3) partially 𝛿𝑢 𝛿
w.r.t x & y, we get, = 𝛿𝑦 (𝑥 2 + 𝑦 2 ln 𝑥 + 2𝑒 −𝑥 𝑦)
𝛿𝑦
= 0 + 2𝑦 ln 𝑥 + 2𝑒 −𝑥 … …. (3)
Again differentiating (3) partially with respect
to 𝑦 we get
𝛿2 𝑢 𝛿
= 𝛿𝑦 (2𝑦 ln 𝑥 + 2𝑒 −𝑥 )
𝛿𝑦 2
= 2 ln 𝑥
problem-4:If 𝑢 = 𝑒 𝑥 (𝑥 cos 𝑦 − 𝑦 sin 𝑦) then
𝛿2 𝑢 𝛿2 𝑢 𝛿2 𝑢 𝛿2 𝑢
find 𝛿𝑥 2 and . Also show that 𝛿𝑥 2 + =
𝛿𝑦 2 𝛿𝑦 2
0.
Solution: Given, 𝑢 = 𝑒 𝑥 (𝑥 cos 𝑦 −
𝑦 sin 𝑦) … … . (1)
Differentiating (1) partially with respect to 𝑥
we get,

53 | P a g e
Module 6
Partial Derivatives
𝛿𝑢 𝛿
 2 f 2( x 2  y 2 )  2 x.2 x = 𝛿𝑥 (𝑥𝑒 𝑥 cos 𝑦 − 𝑦𝑒 𝑥 sin 𝑦)
 𝛿𝑥
x 2 ( x 2  y 2 )2 𝛿 𝛿
= cos 𝑦 (𝑥𝑒 𝑥 ) − 𝑦 sin 𝑦 (𝑒 𝑥 )
𝛿𝑥 𝛿𝑥
2 x2  2 y2  4 x2
 = cos 𝑦 (𝑥𝑒 𝑥 + 𝑒 𝑥 ) − 𝑦 sin 𝑦 𝑒 𝑥
( x 2  y 2 )2
= 𝑥𝑒 𝑥 cos 𝑦 + 𝑒 𝑥 cos 𝑦 −
2 y2  2 x2 𝑦 sin 𝑦 𝑒 𝑥 … … . (2)

( x 2  y 2 )2 Now differentiating (2) partially with respect
 2 f 2( x 2  y 2 )  2 y.2 y to 𝑥 we get,

y 2 ( x 2  y 2 )2 𝛿2 𝑢 𝛿
= 𝛿𝑥 (𝑥𝑒 𝑥 cos 𝑦 + 𝑒 𝑥 cos 𝑦 −
𝛿𝑥 2
2 x2  2 y2  4 y2 𝑦 sin 𝑦 𝑒 𝑥 )

( x 2  y 2 )2 = (𝑥𝑒 𝑥 + 𝑒 𝑥 ) cos 𝑦 + 𝑥𝑒 𝑥 . 0 +
2 x2  2 y2 𝑒 𝑥 cos 𝑦 + 0 − 𝑦 sin 𝑦 𝑒 𝑥

( x 2  y 2 )2
3
𝛿𝑢
2 f 2 f = −𝑥(𝑥 2 + 𝑦 2 + 𝑧 2 )−2
 L. H .S  2  2 𝛿𝑥
x y
𝛿𝑢
2 y2  2 x2 2 x2  2 y2 ∴ 𝑥 𝛿𝑥 = −𝑥 2 (𝑥 2 + 𝑦 2 +
 
( x 2  y 2 )2 ( x 2  y 2 )2 3
𝑧 2 )−2 … … (1)
0
 R.H .S 𝛿𝑢
Similarly, 𝑦 𝛿𝑦 = −𝑦 2 (𝑥 2 + 𝑦 2 +
3
𝛿2 𝑢
= 𝑥𝑒 𝑥 cos 𝑦 + 2𝑒 𝑥 cos 𝑦 − 𝑧 2 )−2 … … . (2)
𝛿𝑥 2
− 𝑦 sin 𝑦 𝑒 𝑥 … … . (3)

Again differentiating (1) partially with respect


to 𝑦 we get, 𝛿𝑢 3

𝛿𝑢 𝛿 Also, 𝑧 𝛿𝑧 = −𝑧 2 (𝑥 2 + 𝑦 2 + 𝑧 2 )−2 …. ….. (3)


= 𝛿𝑦 (𝑥𝑒 𝑥 cos 𝑦 − 𝑦𝑒 𝑥 sin 𝑦)
𝛿𝑦
𝛿 𝛿 Now adding (1),(2) and (3), we get
= 𝑥𝑒 𝑥 (cos 𝑦) − 𝑒 𝑥 (𝑦 sin 𝑦)
𝛿𝑦 𝛿𝑦
= −𝑥𝑒 sin 𝑦 − 𝑒 𝑥 (𝑦 cos 𝑦 + sin 𝑦)
𝑥
𝛿𝑢 𝛿𝑢 𝛿𝑢
= −𝑥𝑒 𝑥 sin 𝑦 − 𝑦𝑒 𝑥 cos 𝑦 − 𝑥 +𝑦 +𝑧
𝛿𝑥 𝛿𝑦 𝛿𝑧
𝑒 𝑥 sin 𝑦 … … (4) 3
= −𝑥 2 (𝑥 2 + 𝑦 2 + 𝑧 2 )−2
Again differentiating (4) partially with respect 3
to 𝑦 we get − 𝑦 2 (𝑥 2 + 𝑦 2 + 𝑧 2 )−2
𝛿2 𝑢 𝛿 3
𝛿𝑦 2
= 𝛿𝑦 (−𝑥𝑒 𝑥 sin 𝑦 − 𝑦𝑒 𝑥 cos 𝑦 − − 𝑧 2 (𝑥 2 + 𝑦 2 + 𝑧 2 )−2
𝑒 𝑥 sin 𝑦)

54 | P a g e
Module 6
Partial Derivatives
= −𝑥𝑒 𝑥 cos 𝑦 − 𝑒 𝑥 (−𝑦 sin 𝑦 + cos 𝑦) −
𝛿𝑢 𝛿𝑢 𝛿𝑢
𝑒 𝑥 cos 𝑦 => 𝑥 𝛿𝑥 + 𝑦 𝛿𝑦 + 𝑧 𝛿𝑧 = −(𝑥 2 + 𝑦 2 +
= −𝑥𝑒 𝑥 cos 𝑦 + 𝑦𝑒 𝑥 sin 𝑦 − 𝑒 𝑥 cos 𝑦 − 3
𝑧 2 )−2 (𝑥 2 + 𝑦 2 + 𝑧 2 )
𝑒 𝑥 cos 𝑦
𝛿2 𝑢
= −𝑥𝑒 𝑥 cos 𝑦 + 𝑦𝑒 𝑥 sin 𝑦 − 𝛿𝑢 𝛿𝑢 𝛿𝑢
=> 𝑥 𝛿𝑥 + 𝑦 𝛿𝑦 + 𝑧 𝛿𝑧 = −(𝑥 2 + 𝑦 2 +
𝛿𝑦 2
𝑥
2𝑒 cos 𝑦 … . … . (5) 3

𝛿2 𝑢 𝛿2 𝑢
𝑧 2 )−2+1
Now, +
𝛿𝑥 2 𝛿𝑦 2
1
𝛿𝑢 𝛿𝑢 𝛿𝑢
= 𝑥𝑒 cos 𝑦 + 2𝑒 𝑥 cos 𝑦 − 𝑦 sin 𝑦 𝑒 𝑥 −
𝑥
=> 𝑥 𝛿𝑥 + 𝑦 𝛿𝑦 + 𝑧 𝛿𝑧 = −(𝑥 2 + 𝑦 2 + 𝑧 2 )−2
𝑥𝑒 𝑥 cos 𝑦 + 𝑦𝑒 𝑥 sin 𝑦 − 2𝑒 𝑥 cos 𝑦
= 0 (Showed). 𝛿𝑢 𝛿𝑢 𝛿𝑢
1
=> 𝑥 𝛿𝑥 + 𝑦 𝛿𝑦 + 𝑧 𝛿𝑧 = −𝑢 (Showed)

Problem-5: If 𝑢 = (𝑥 2 + 𝑦 2 + 𝑧 2 ) 2 , then
𝛿𝑢 𝛿𝑢 𝛿𝑢
show that 𝑥 𝛿𝑥 + 𝑦 𝛿𝑦 + 𝑧 𝛿𝑧 = −𝑢 .
1
Solution: Given, 𝑢 = (𝑥 2 + 𝑦 2 + 𝑧 2 )−2
Differentiating (1) partially with respect to 𝑥
we get,
1
𝛿𝑢 𝛿
= 𝛿𝑥 ((𝑥 2 + 𝑦 2 + 𝑧 2 )−2 )
𝛿𝑥

1
1 𝛿
= − (𝑥 2 + 𝑦 2 + 𝑧 2 )−2−1 . (𝑥 2 + 𝑦 2 +
2 𝛿𝑥
𝑧2)
3
1
= − 2 (𝑥 2 + 𝑦 2 + 𝑧 2 )−2 (2𝑥 + 0 + 0)

3
1
=− 2 (𝑥 2 + 𝑦 2 + 𝑧 2 )−2 . 2𝑥

55 | P a g e
Module 6
Partial Derivatives
1
Problem-5: If 𝑓 = (𝑥 2 + 𝑦 2 + 𝑧 2 )2 , then
𝛿2 𝑓 𝛿2 𝑓 𝛿2 𝑓
Adding (2),(3) and (4)
find + 𝛿𝑦 2 + 𝛿𝑧 2 .
𝛿𝑥 2
1 𝛿2𝑓 𝛿2𝑓 𝛿2𝑓 𝑦 2 +𝑧 2
Solution: Given, 𝑓 = (𝑥 2 + 𝑦 2 + 𝑧 2 )2 , … … 2
+ 2
+ = 3 +
𝛿𝑥 𝛿𝑦 𝛿𝑧 2 (𝑥 2 +𝑦 2 +𝑧 2 )2
(1)
𝑥 2 +𝑧 2 𝑥 2 +𝑦 2
Differentiating (1) partially with respect to 𝑥 3 + 3
we get, (𝑥 2 +𝑦 2 +𝑧 2 )2 (𝑥 2 +𝑦 2 +𝑧 2 )2
1
𝛿𝑢 𝛿
= 𝛿𝑥 ((𝑥 2 + 𝑦 2 + 𝑧 2 )−2 ) 𝛿2𝑓 𝛿2𝑓 𝛿2𝑓 𝑦 2 +𝑧 2 +𝑥 2 +𝑧 2 +𝑥 2 +𝑦 2
𝛿𝑥
+ + = 3
𝛿𝑥 2 𝛿𝑦 2 𝛿𝑧 2 (𝑥 2 +𝑦 2 +𝑧 2 )2
1
𝛿𝑢 −
= 𝑥(𝑥 2 + 𝑦 2 + 𝑧 2 ) 2 , … . … (2)
𝛿𝑥
𝛿2𝑓 𝛿2𝑓 𝛿2𝑓 2(𝑥 2 +𝑦 2 +𝑧 2 )
+ + = 3
𝛿2 𝑓 𝛿 1 𝛿𝑥 2 𝛿𝑦 2 𝛿𝑧 2 (𝑥 2 +𝑦 2 +𝑧 2 )2
= 𝛿𝑥 (𝑥(𝑥 2 + 𝑦 2 + 𝑧 2 )−2 )
𝛿𝑥 2
𝛿2 𝑓 𝛿2 𝑓 𝛿2 𝑓 2(𝑥 2 +𝑦 2 +𝑧 2 )
𝛿2 𝑓 1 2 )−2−1
1
𝛿 + 𝛿𝑦 2 + 𝛿𝑧 2 = 1
= 𝑥 {− 2 (𝑥 2 + 𝑦 2 + 𝑧 . 𝛿𝑥 (𝑥 2 + 𝛿𝑥 2
(𝑥 2 +𝑦2 +𝑧 2 )(𝑥 2 +𝑦2 +𝑧 2 )2
𝛿𝑥 2
1
𝛿
𝑦 2 + 𝑧 2 )} + (𝑥 2 + 𝑦 2 + 𝑧 2 )−2 (𝑥) 𝛿2 𝑓 𝛿2 𝑓 𝛿2 𝑓 2
𝛿𝑥 + 𝛿𝑦 2 + 𝛿𝑧 2 = 1
𝛿2 𝑓 1 3 𝛿𝑥 2 (𝑥 2 +𝑦2 +𝑧 2 )2
= {− 2 (𝑥 2 + 𝑦 2 + 𝑧 2 )−2 . (2𝑥 + 0 +
𝛿𝑥 2
1
𝛿2 𝑓 𝛿2 𝑓 𝛿2 𝑓 2
0)}+(𝑥 2 + 𝑦 2 + 𝑧 2 )−2 . 1 + 𝛿𝑦 2 + 𝛿𝑧 2 = 𝑓 .
𝛿𝑥 2

3
𝛿2 𝑓
= −𝑥

2(𝑥 2 +𝑦 2 +𝑧 2 ) 2
+ (𝑥 2 + 𝑦 2 + 𝑧 2 )−2
1
Problem-6: If 𝑓 = ln (𝑥3 + 𝑦3 + 𝑧3 −
𝛿𝑥 2 𝛿𝑢 𝛿𝑢 𝛿𝑢
3𝑥𝑦𝑧) , then show that + + =
𝛿𝑥 𝛿𝑦 𝛿𝑧
𝛿2𝑓 −𝑥 2 1
= 3 + 1 3
𝛿𝑥 2 (𝑥 2 +𝑦 2 +𝑧 2 )2 (𝑥 2 +𝑦 2 +𝑧 2 )2 .
𝑥+𝑦+𝑧

𝛿2𝑓 −𝑥 2 +𝑥 2 +𝑦 2 +𝑧 2 Solution : Given 𝑓 = ln(𝑥3 + 𝑦3 + 𝑧3 −


= 3
𝛿𝑥 2 (𝑥 2 +𝑦 2 +𝑧 2 )2 3𝑥𝑦𝑧)
Differentiating partially with respect to
𝛿2𝑓 𝑦 2 +𝑧 2
= 3 … . … . (2) 𝑥, 𝑦 & 𝑧 we get,
𝛿𝑥 2 (𝑥 2 +𝑦 2 +𝑧 2 )2 𝛿𝑢 1
= (𝑥 3 . (3𝑥 2 − 3𝑦𝑧)
𝛿𝑥 +𝑦 3 +𝑧 3 −3𝑥𝑦𝑧)
𝛿2𝑓
Simularly, =
𝛿𝑦 2 𝛿𝑢 1
𝑥 2 +𝑧 2
= (𝑥 3 . (3𝑦 2 − 3𝑥𝑧)
𝛿𝑥 +𝑦 3 +𝑧 3 −3𝑥𝑦𝑧)
3 … … (3)
𝛿𝑢 1
(𝑥 2 +𝑦 2 +𝑧 2 )2 = (𝑥 3 . (3𝑧 2 − 3𝑥𝑦)
𝛿𝑥 +𝑦 3 +𝑧 3 −3𝑥𝑦𝑧)

56 | P a g e
Module 6
Partial Derivatives

𝛿𝑢 𝛿𝑢 𝛿𝑢 (3𝑥 2 −3𝑦𝑧)
𝛿2𝑓 𝑥 2 +𝑦 2 ∴ + + = +
Also, = 3 …. … (4) 𝛿𝑥 𝛿𝑦 𝛿𝑧 (𝑥 3 +𝑦 3 +𝑧 3 −3𝑥𝑦𝑧)
𝛿𝑧 2 (𝑥 2 +𝑦 2 +𝑧 2 )2
1
Problem-7: If 𝑣 = , then find
√(𝑥2 +𝑦2 +𝑧2 )

(3𝑦 2 −3𝑥𝑧) (3𝑧 2 −3𝑥𝑦) 𝑣𝑥𝑥 + 𝑣𝑦𝑦 + 𝑣𝑧𝑧 .


(𝑥 3 +𝑦 3 +𝑧 3 −3𝑥𝑦𝑧)
+(𝑥 3
+𝑦 3 +𝑧 3 −3𝑥𝑦𝑧)
1
Solution : Given 𝑣 =
3(𝑥 2 +𝑦 2 +𝑧 2 −𝑥𝑦−𝑦𝑧−𝑧𝑥) √(𝑥2 +𝑦2 +𝑧2 )
= (𝑥 3 +𝑦 3 +𝑧 3 −3𝑥𝑦𝑧)
3(𝑥 2 +𝑦 2 +𝑧 2 −𝑥𝑦−𝑦𝑧−𝑧𝑥) 1
= 2
𝑣 = (𝑥 + 𝑦2 + 𝑧2 )−2
(𝑥+𝑦+𝑧)(𝑥 2 +𝑦 2 +𝑧 2 −𝑥𝑦−𝑦𝑧−𝑧𝑥)

𝛿𝑢 𝛿𝑢 𝛿𝑢 3 Differentiating partially with respect to 𝑥,


∴ + + = (Showed).
𝛿𝑥 𝛿𝑦 𝛿𝑧 (𝑥+𝑦+𝑧)
3
2
𝑣𝑥 = −𝑥(𝑥 + 𝑦2 + 𝑧2 )−2

Again, Differentiating partially with respect to


𝑥,

3𝑥2 1
𝑣𝑥𝑥 = 2 5 − 2 3 =
(𝑥 +𝑦2 +𝑧2 )−2 (𝑥 +𝑦2 +𝑧2 )−2
2𝑥2 −𝑦2 −𝑧2
2 5
(𝑥 +𝑦2 +𝑧2 )−2

2𝑦−𝑥2 −𝑧2
Similarly, 𝑣𝑦𝑦 = 2 5 Also,
(𝑥 +𝑦2 +𝑧2 )−2
2𝑧−𝑦2 −𝑥2
𝑣𝑧𝑧 = 2 5
(𝑥 +𝑦2 +𝑧2 )−2

2𝑥2 −𝑦2 −𝑧2


∴ 𝑣𝑥𝑥 + 𝑣𝑦𝑦 + 𝑣𝑧𝑧 = 2 5 +
(𝑥 +𝑦2 +𝑧2 )−2
2𝑦−𝑥2 −𝑧2 2𝑧−𝑦2 −𝑥2
2 5 + 2 5
(𝑥 +𝑦2 +𝑧2 )−2 (𝑥 +𝑦2 +𝑧2 )−2

∴ 𝑣𝑥𝑥 + 𝑣𝑦𝑦 + 𝑣𝑧𝑧 = 0.

Try yourself:
57 | P a g e
Module 6
Partial Derivatives
𝟐 𝟐
1) If 𝒗 = √(𝒙 + 𝒚𝟐 + 𝒛𝟐 ) , then find 𝒗𝒙𝒙 + 𝒗𝒚𝒚 + 𝒗𝒛𝒛 . Ans. 𝒗

2) If 𝒇 = (𝒙𝟐 + 𝒚𝟐 + 𝒛𝟐 ) , then find 𝒙𝒗𝒙 + 𝒚𝒗𝒚 + 𝒛𝒗𝒛 . Ans. 𝟐𝒗


𝜹𝒇 𝜹𝒇 𝜹𝟐 𝒇 𝜹𝟐 𝒇
3) Given 𝒇(𝒙, 𝒚) = 𝒙𝟐 𝒚 + 𝐬𝐢𝐧 𝒙 + 𝐜𝐨𝐬 𝒚, find , 𝜹𝒚 , 𝜹𝒙 𝜹𝒚 and .
𝜹𝒙 𝜹𝒚 𝜹𝒙
𝜹𝒇 𝜹𝒇 𝜹𝒇
4) Given 𝒇(𝒙, 𝒚, 𝒛) = 𝒙 𝐜𝐨𝐬 𝒛 + 𝒙𝟐 𝒚𝟑 𝒆𝒛 , find , 𝜹𝒚 & .
𝜹𝒙 𝜹𝒛

5) If 𝒇(𝒙, 𝒚) = 𝟑𝒙𝟐 𝒚 + 𝟓𝒙 − 𝟐𝒚𝟐 + 𝟏, find 𝒇𝒙 , 𝒇𝒙 , 𝒇𝒙 𝒙, 𝒇𝒚𝒚 𝒂𝒏𝒅 𝒇𝒙𝒚 .

58 | P a g e
Module-07
Integration

7.1 Anti differentiation: A function F is called antiderivative of a given function f on an interval L if 𝐹 ′ (𝑥) =
𝑓(𝑥)For all x in L.

Ex: 𝑥 3 is an antiderivative of 3𝑥 2 .

7.2 Define Indefinite Integral: The notation  f (x) dx  F(x)  c .where c is an arbitrary constant means
that F is an anti-derivative of f . It is called indefinite integral of f and satisfies the condition that
F (x)  f (x) for all x in the domain of f .
𝒅
Example: (i) 𝒅𝒙 (𝐭𝐚𝐧 𝒙) = 𝒔𝒆𝒄𝟐 𝒙

∫ 𝑠𝑒𝑐 2 𝑥 𝑑𝑥 = tan 𝑥

𝒅 𝟏
ii)𝒅𝒙 𝐥𝐨𝐠 𝒙 = 𝒙
𝟏
∫ 𝒙 𝒅𝒙 = 𝐥𝐨𝐠 𝒙
7.3Basic integration rules:
𝒙𝒏+𝟏
1. ∫ 𝒙𝒏 𝒅𝒙 = 𝒏+𝟏
2. ∫ 𝒂 𝒅𝒙 =ax+c
𝟏
3. ∫ 𝒙 𝒅𝒙 = 𝒍𝒏𝒙
4. ∫ 𝒆𝒙 𝒅𝒙 = 𝒆𝒙
𝒂𝒙
5. ∫ 𝒂𝒙 𝒅𝒙 = 𝐥𝐧 𝒂
6. ∫ 𝒔𝒊𝒏𝒙 𝒅𝒙 = − 𝐜𝐨𝐬 𝒙
7. ∫ 𝒄𝒐𝒔𝒙 𝒅𝒙 = 𝐬𝐢𝐧 𝒙
8. ∫ 𝒔𝒆𝒄𝒙 𝒕𝒂𝒏𝒙 𝒅𝒙=secx
9. ∫ 𝒄𝒐𝒔𝒆𝒄 𝒙 𝐜𝐨𝐭 𝒙 𝒅𝒙=-cosec x
10. ∫ 𝒔𝒆𝒄𝟐 𝒙𝒅𝒙= tanx
11. ∫ 𝒄𝒐𝒔𝒆𝒄𝟐 𝒙 𝒅𝒙= -cot x
12. ∫ 𝟏 𝒅𝒙 = 𝒙
𝟏
13. ∫ 𝒅𝒙 = 𝑺𝒊𝒏−𝟏 𝒙
√𝟏−𝒙𝟐
𝟏
14. ∫ 𝟏+𝒙𝟐 𝒅𝒙 = 𝒕𝒂𝒏−𝟏 𝒙
𝟏
15. ∫ 𝒅𝒙 = 𝑺𝒆𝒄−𝟏 𝒙
𝒙√𝟏−𝒙𝟐
𝒆𝒎𝒙
16. ∫ 𝒆𝒎𝒙 𝒅𝒙 = 𝒎

59 | P a g e
Module-07
Integration
17. ∫ 𝒔𝒊𝒏𝒉𝒙 𝒅𝒙 = 𝐜𝐨𝐬𝐡 𝒙
18. ∫ 𝒄𝒐𝒔𝒉𝒙 𝒅𝒙 = 𝐬𝐢𝐧 𝒉𝒙
19. ∫ 𝒔𝒆𝒄𝒉𝒙 𝒕𝒂𝒏𝒉𝒙 𝒅𝒙=-sechx
20. ∫ 𝒄𝒐𝒔𝒆𝒄𝒉 𝒙 𝐜𝐨𝐭 𝒉𝒙 𝒅𝒙=-cosec hx
21. ∫ 𝒔𝒆𝒄𝟐 𝒉𝒙 𝒅𝒙= tanhx
22. ∫ 𝒄𝒐𝒔𝒆𝒄𝟐 𝒉𝒙 𝒅𝒙= -cot hx
𝑭′ (𝒙)
23. ∫ 𝒅𝒙 = 𝐥𝐧[𝒇(𝒙)]
𝑭(𝒙)
24. ∫ 𝒕𝒂𝒏𝒙 𝒅𝒙 = 𝐥𝐧 𝒔𝒆𝒄𝒙 = −𝒍𝒏𝒄𝒐𝒔𝒙
25. ∫ 𝒄𝒐𝒕𝒙 𝒅𝒙 = 𝐥𝐧 𝒔𝒊𝒏𝒙
𝒅𝒖
26. ∫ 𝒖. 𝒗 𝒅𝒙 = 𝒖 ∫ 𝒗 𝒅𝒙 − ∫ 𝒅𝒗 [ ∫ 𝒗 𝒅𝒙]𝒅𝒙
27. In terms of this integration √𝒂 + 𝒙 we have to putx+a=𝒛𝟐
𝒂−𝒙
28. In terms of this integration √𝒂+𝒙 we have to put 𝒙 = 𝒂 𝒄𝒐𝒔𝟐𝜽
29. In terms of this integration 𝒆𝒙 we have to put 𝒆𝒙 = 𝒛
𝟏 𝟏 𝒙
30. ∫ 𝒙𝟐 +𝒂𝟐 𝒅𝒙 = 𝒂 𝒕𝒂𝒏−𝟏 𝒂
𝟏 𝟏 𝒙−𝒂
31. ∫ 𝒙𝟐 −𝒂𝟐 𝒅𝒙 = 𝟐𝒂 𝒍𝒏 𝒙+𝒂
𝟏 𝟏 𝒂+𝒙
32. ∫ 𝟐 𝒅𝒙 = 𝟐𝒂 𝒍𝒏 𝒂−𝒙
𝒂𝟐−𝒙
𝟏 𝒙
33. ∫ 𝒅𝒙 = 𝑺𝒊𝒏−𝟏 𝒂
√ 𝒂𝟐 −𝒙𝟐
𝟏
34. ∫ 𝒅𝒙 = 𝐥𝐧(𝒙 + √ 𝒙𝟐 − 𝒂𝟐 )
√ 𝒂𝟐 +𝒙𝟐
𝟏 𝟏 𝒙
35. ∫ 𝒅𝒙 = 𝒂 𝑺𝒆𝒄−𝟏 𝒂
𝒙√ 𝒙𝟐 −𝒂𝟐

7.4Find the following indefinite integrals:


x4  1 1  x  dx
2
i.  5 x3dx ii.  dx iii. 2 dx
x x
iv. 
x

v.  x 2  e x  2x dx 
  4x   1  3x 1  x  dx   3x 
1
vi. 3
 3x 2  2 x  5 dx vii. viii.  4 x 2  3x  8 dx

 
ix.  8e x  4a x  3x1  4 x dx x.   2 x  9 dx xi. 
5
5x  7  dx
3

 
xii.  x3  2 .3x 2 dx xiii.  x log
1
dx xiv.  x 2e3 x dx xv.  x3e2 x dx
2

e x log e (log e x)

Solution:

60 | P a g e
Module-07
Integration
i.  5 x3dx  5 x 3dx
x31
5 c (F-2)
3 1
5x4
  c.
4

2 1
ii.  x dx  2 x dx
 2log e x  c (F-3)

x4  1  x4 1 
iii.  2 dx    2  2  dx    x 2  x 2  dx
x x x 
  x 2 dx   x 2 dx
x 21 x 21
=  c (F-2)
2  1 2  1
x 3 x 1
=  c
3 1
x3 1
=  c
3 x
 1 x  1 x 1
iv.    dx      dx   dx   1dx
 x  x x x
=loge x  x  c (F-3 and F-1)

x 
 e x  2 x dx   x 2 dx   e x dx   2 x dx
2
v.
x 21 2x
=  ex   c (F-2,F-4 and F-5)
2 1 log e 2
x3 2x
=  ex  c
3 log e 2

 4x 
 3 x 2  2 x  5 dx   4 x 3dx   3 x 2 dx   2 xdx   5dx
3
vi.
= 4 x3dx  3 x 2 dx  2 xdx   5dx
x31 x 21 x11
=4 3 2  5x  c
3 1 2 1 11
=x 4  x3  x 2  5 x  c

61 | P a g e
Module-07
Integration

 1  3x 1  x  dx   1  x  3x  3x  dx
2
vii.
=  1  2 x  3x  dx2

= 1dx  2  xdx  3 x dx 2

x11 x 21
= x-2 3 c
11 2 1
= x-x 2  x3  c

  3x 
 4 x 2  3 x  8 dx =  3 x 1dx   4 x 2 dx   3 xdx   8dx
1
viii.
= 3 x 1dx  4  x 2 dx  3 xdx  8 dx
x 21 x11
= 3log e x  4 3  8x  c
2 1 11
4 x3 3 2
=3log e x   x  8x  c
3 2

 
ix.  8e x  4a x  3x 1  4 x dx =  8e x dx   4a x dx   3x 1dx   x 4 dx
1

=8  e x dx  4 a x dx  3 x 1dx   x 4 dx
1

1
1
ax x4
= 8ex  4  3log e x  c
log e a 1
1
4
5
4a x x 4
=8e  x
 3loge x  c
log e a 5
4

  2x  9
5
x. dx
Let z = 2x+9
dz d
  (2 x  9)
dx dx
dz
 2
dx
dz
  dx
2
1  2x  9  2x  9  c
6 6
dz 1 z 51 1 z6
Now   2 x  9  dx   z
5
 5
c  c  c 
2 2 5 1 2 6 2 6 12

62 | P a g e
Module-07
Integration

  5 x  7  dx
3
xi.
Let z = 5x+7
dz d
  (5 x  7)
dx dx
dz
 5
dx
dz
  dx
5
3 5
3 1
2 2
dz 1 1 z 1z 2
 5x  7   c  5  c  5x  7  2  c
5
dx   z 3   z dz 
3
2
Now
5 5 5 3 1 5 2 25
2

x 
2
xii. 3
 2 .3x 2 dx
Let z = x 3 +2
dz
  3x 2
dx
 dz  3x 2 dx

 
3
z 21 z3 x3  2
 
x  2 .3x dx   z dz 
2
Now 3 2
c  c 
2
c
2 1 3 3

1
xiii.  x log e x log e (log e x )
dx

Let z = log e (log e x)


dz d 1 d 1 1
   log e (log e x)   (log e x)  .
dx dx log e x dx log e x x
1
 dz  dx
x log e x
1 1
 x log dx   dz  log e z  c  log e  log e (log e x)   c
e x log e (log e x ) z

7.5 Solve the following trigonometric indefinite integrals :


cos 𝑥
1. ∫ 𝑑𝑥
𝑠𝑖𝑛2 𝑥

63 | P a g e
Module-07
Integration
1 cos 𝑥
=∫ 𝑑𝑥
sin 𝑥 sin 𝑥

=∫ cosec 𝑥. cot 𝑥 𝑑𝑥
= −𝑐𝑜𝑠𝑒𝑐𝑥 + 𝑐 (Ans.)

2. ∫ √1 + sin 2𝑥dx

=∫ √𝑠𝑖𝑛2 𝑥 + 𝑐𝑜𝑠 2 𝑥 + 2 sin 𝑥 cos 𝑥 dx

= ∫ √(sin 𝑥 + cos 𝑥)2 dx


=∫(sin 𝑥 + cos 𝑥) 𝑑𝑥
=∫ sin 𝑥 𝑑𝑥 + ∫ cos 𝑥 𝑑𝑥
=− cos 𝑥 + sin 𝑥 + 𝑐 (Ans)

1
3. ∫ 𝑑𝑥
1− sin 𝑥
(1+sin 𝑥)
= ∫ (1+sin 𝑑𝑥
𝑥)(1− sin 𝑥)

(1+sin 𝑥)
=∫ 𝑑𝑥
1−𝑠𝑖𝑛2 𝑥
1+sin 𝑥
=∫ 𝑑𝑥
𝑐𝑜𝑠 2 𝑥
1 sin 𝑥
=∫ 𝑑𝑥 + ∫ 𝑑𝑥
𝑐𝑜𝑠 2 𝑥 𝑐𝑜𝑠 2 𝑥
1 sin 𝑥
=∫ 𝑠𝑒𝑐 2 𝑥𝑑𝑥 + ∫ . 𝑑𝑥
cos 𝑥 cos 𝑥

=∫ 𝑠𝑒𝑐 2 𝑥𝑑𝑥 + ∫ sec 𝑥 tan 𝑥 𝑑𝑥


=tan 𝑥 + sec 𝑥 + 𝑐 (Ans)

64 | P a g e
Module-07
Integration
𝑆𝑖𝑛2 𝑥−𝐶𝑜𝑠 2 𝑥
4. ∫ 𝑑𝑥
𝑆𝑖𝑛2 𝑥𝐶𝑜𝑠 2 𝑥
1 1
=∫( − )𝑑𝑥
𝐶𝑜𝑠 2 𝑥 𝑆𝑖𝑛2 𝑥

=∫( 𝑠𝑒𝑐 2 𝑥 − 𝑐𝑜𝑠𝑒𝑐 2 𝑥) 𝑑𝑥 = 𝑡𝑎𝑛𝑥 + 𝑐𝑜𝑡𝑥 + 𝑐 (ans)


5. ∫ 𝑆𝑖𝑛3 𝑥 𝑑𝑥
1 3 1
= ∫(3𝑠𝑖𝑛𝑥 − 𝑠𝑖𝑛3𝑥)𝑑𝑥 = − 𝑐𝑜𝑠𝑥 + 𝑐𝑜𝑠3𝑥 + 𝑐 (ans)
4 4 12

6. ∫ 𝑐𝑜𝑠𝑥. √1 − 𝑐𝑜𝑠2𝑥 𝑑𝑥

=∫ 𝑐𝑜𝑠𝑥. √2𝑠𝑖𝑛2 𝑥 𝑑𝑥 = √2 ∫ 𝑠𝑖𝑛𝑥 𝑐𝑜𝑠𝑥 𝑑𝑥


√2
=
2
∫ 𝑠𝑖𝑛𝑥 𝑐𝑜𝑠𝑥 𝑑𝑥
1
=
√2
∫ 𝑠𝑖𝑛2𝑥 𝑑𝑥
−𝑐𝑜𝑠2𝑥
= +c
2√2

2−𝑠𝑖𝑛2𝑥
7. ∫ 𝑑𝑥
1−𝑐𝑜𝑠2𝑥
2−𝑠𝑖𝑛𝑥 𝑐𝑜𝑠𝑥
=∫ 𝑑𝑥
2𝑠𝑖𝑛2 𝑥
1 𝑐𝑜𝑠𝑥
=∫( − )𝑑𝑥
𝑠𝑖𝑛2 𝑥 𝑠𝑖𝑛𝑥

=−𝑐𝑜𝑡𝑥 − ln|𝑠𝑖𝑛𝑥| + 𝑐 (ans)


8. ∫(𝑒 𝑎𝑙𝑛𝑥 + 𝑒 𝑥𝑙𝑛𝑎 )dx
𝑎 𝑥
=∫(𝑒 𝑙𝑛𝑥 + 𝑒 𝑙𝑛𝑎 )𝑑𝑥
=∫( 𝑥 𝑎 + 𝑎 𝑥 )𝑑𝑥
𝑥 𝑎+1 𝑎𝑥
= + + 𝑐 (ans)
𝑎+1 ln 𝑎

9. ∫ 𝑐𝑜𝑠𝑥 𝑐𝑜𝑠2𝑥 𝑐𝑜𝑠3𝑥 𝑑𝑥


1
= ∫ 𝑐𝑜𝑠2𝑥 (𝑐𝑜𝑠4𝑥 + 𝑐𝑜𝑠2𝑥) 𝑑𝑥
2
65 | P a g e
Module-07
Integration
1
= ∫(𝑐𝑜𝑠2𝑥 𝑐𝑜𝑠4𝑥 + 𝑐𝑜𝑠 2 2𝑥) 𝑑𝑥
2
1
= ∫(𝑐𝑜𝑠6𝑥 + 𝑐𝑜𝑠2𝑥 + 1 + 𝑐𝑜𝑠4𝑥) 𝑑𝑥
4
1 𝑠𝑖𝑛2𝑥 𝑠𝑖𝑛4𝑥 𝑠𝑖𝑛6𝑥
=4 [𝑥 + + + +c (ans)
2 4 6

7.5Try yourself:
1. ∫ √𝟏 + 𝒄𝒐𝒔𝒙 𝒅𝒙
𝒙𝟑 +𝟑𝒙𝟐 +𝟒
2. ∫ 𝒅𝒙
√𝒙
𝟏 𝟑
3. ∫(√𝒙 − ) 𝒅𝒙
√𝒙
𝟏
4. ∫ 𝒅𝒙
√𝒙+𝟏−√𝒙−𝟐
𝑺𝒊𝒏𝟒 𝒙−𝑪𝒐𝒔𝟒 𝒙
5. ∫ 𝒅𝒙
√𝟏+𝒄𝒐𝒔𝟒𝒙
6. ∫ 𝒔𝒊𝒏𝟐𝒙 𝒄𝒐𝒔𝟑𝒙 𝒅𝒙

66 | P a g e
Module-08
Method of Substitution & Integration by Parts

8.1 Method of substitution: In this method of integration by substitution, any given integral is
transformed into a simple form of integral by substituting the independent variable by others.
Take for example an equation having an independent variable in x, i.e. ∫sin (x3).3x2.dx————
———–(i),In the equation given above the independent variable can be transformed into another
variable say t. Substituting x3 = t ———————-(ii) Differentiation of above equation will
give- 3x2.dx = dt ———————-(iii) Substituting the value of (ii) and (iii) in (i), we have ∫sin
(x3).3x2.dx = ∫sin t . dt Thus the integration of the above equation will give ∫sin t . dt= -cos t + c
Again putting back the value of t from equation (ii), we get ∫sin (x3).3x2.dx = -cos x3 + c The
General Form of integration by substitution is: ∫ f(g(x)).g'(x).dx = f(t).dt, where t = g(x) Usually
the method of integration by substitution is extremely useful when we make a substitution for a
function whose derivative is also present in the integrand. Doing so, the function simplifies and
then the basic formulas of integration can be used to integrate the function.

8.2 When to Use Integration by Substitution Method?

In calculus, the integration by substitution method is also known as the “Reverse Chain Rule” or
“U-Substitution Method”. We can use this method to find an integral value when it is set up in
the special form. It means that the given integral is of the form:

∫ f(g(x)).g'(x).dx = f(u).du

Here, first, integrate the function with respect to the substituted value (f(u)), and finish the
process by substituting the original function g(x).

8.3 Integration by Substitution Example

To understand this concept better, let us look into the examples.

Example 1:

Find the integration of

Solution:

67 | P a g e
Module-08
Method of Substitution & Integration by Parts
Given :

Let t = tan-1x …… (1)

dt = (1/ 1+x2 ) . dx

I = ∫ et . dt

= et + C …….(2)

Substituting the value of (1) in (2), we have I = etan-1x + C. This is the required integration for the
given function.

Example 2:

Integrate 2x cos (x2 – 5) with respect to x .

Solution:

I = ∫2xcos(x2 – 5).dx

Let x2 – 5 = t …..(1)

2x.dx = dt

Substituting these values, we have

I = ∫cos(t).dt

= sin t + c …..(2)

Substituting the value of 1 in 2, we have

= sin (x2 – 5)+ C

68 | P a g e
Module-08
Method of Substitution & Integration by Parts
This is the required integration for the given function.
8.4 Integration by Substitution

Integration by substitution is an important method of integration, which is used when a function


to be integrated, is either a complex function or if the direct integration of the function is not
feasible. The integral of a function is simplified by this method of integration by substitution,
by reducing the given function into a simplified function.

Let us learn the process of integration by substitutions, check some of the important
substitutions, and also check the solved examples.
8.5 What Is Integration by Substitution?

Integration by substitution is used when the integration of the given function cannot be obtained
directly, as the given algebraic function is not in the standard form. Further, the given function
can be reduced to the standard form by appropriate substitution. Let us consider the indefinite
integral of a function f(x), ∫f(x).dx

. Here this integral can be transformed to another form by replacing x with g(t) and by
substituting x = g(t).

𝐼 = ∫ 𝑓(𝑥). 𝑑𝑥
𝑥 = 𝑔(𝑡) 𝑠𝑢𝑐ℎ 𝑡ℎ𝑎𝑡 𝑑𝑥/𝑑𝑡 = 𝑔′(𝑡)
𝑑𝑥 = 𝑔′(𝑡). 𝑑𝑡
𝐼 = ∫ 𝑓(𝑥). 𝑑𝑥 = ∫ 𝑓(𝑔(𝑡)). 𝑔′(𝑡). 𝑑𝑡

69 | P a g e
Module-08
Method of Substitution & Integration by Parts
8.6 Steps to Integration by Substitution

The following are the steps that are helpful in performing this method of integration by
substitution.

Step - 1: Choose a new variable t for the given function to be reduced.

Step - 2: Determine the value of dx, of the given integral, where f(x) is integrated with respect to
𝑥.

Step - 3: Make the required substitution in the function f(x), and the new value dx.

Step - 4: Integrate the function obtained after substitution

Step - 5: Substitute back the initial variable x to obtain the final answer.

8.7 Important Substituions in Integration by Substitution

The following are some of the important substitutions which are helpful in simplifying the given
expression and easily performing the integration process. Let us check the following specific
substitutions for integration by substitution.

1. For the integral function √𝒂𝟐 − 𝒙𝟐 ; we use x = aSinθ or x = aCosθ.


2. For the integral of the function 𝒂𝟐 + 𝒙𝟐 ; we use x = a tanθ.
3. For the integral of the function √𝒙𝟐 − 𝒂𝟐 we use x = asecθ.
√𝒂−𝒙
4. For the integral of the functions we use x = a Cos2θ.
√𝒂+𝒙
5. For the integral of the functions √𝒂 + 𝒙 we use a+x = 𝒛𝟐
8.8 Evaluate the following integrals:

1. ∫(𝑎𝑥 2 + 2bx + c)𝑛 (ax + b)dx

Solution
I=∫(𝒂𝒙𝟐 + 𝟐𝐛𝐱 + 𝐜)𝒏 (𝐚𝐱 + 𝐛)𝐝𝐱
𝟏 Put,
= ∫ 𝒛𝒏 𝟐 𝒅𝒛
𝟏 (𝒂𝒙𝟐 +2bx+c) = z
=𝟐 ∫ 𝒛𝒏 𝒅𝒛 𝒅𝒛
𝟏 𝒛𝒏+𝟏
∴ = 2ax+2b
𝒅𝒙
=𝟐 + 𝑪𝒐𝒏𝒐𝒔𝒕𝒂𝒏𝒕
𝐧+𝟏  dz = 2 (ax+b)dx
𝟏 (𝒂𝒙𝟐 +𝟐𝐛𝐱+𝐜)𝒏+𝟏 𝟏
=𝟐 + 𝑪𝒐𝒏𝒐𝒔𝒕𝒂𝒏𝒕  𝟐 dz = (ax +b)dx
𝐧+𝟏

70 | P a g e
Module-08
Method of Substitution & Integration by Parts
(𝒂𝒙𝟐 +𝟐𝐛𝐱+𝐜)𝒏+𝟏
= + 𝑪𝒐𝒏𝒐𝒔𝒕𝒂𝒏𝒕 Ans:
𝟐(𝐧+𝟏)

√tan−1 𝑥
2. ∫ 𝑑𝑥
(1+𝑥 2 )

√tan−1 𝑥
Solve: Let : I =∫ 𝑑𝑥 put, tan−1 𝑥 = 𝑧
(1+𝑥 2 )

1
or, 𝑑𝑥 = 𝑑𝑧
1+𝑥 2

=∫ √𝑧 𝑑𝑧
1
+1
z2
=[ 1 ]+c
+1
2

3
𝑧2
=[ 3 ]+c
2

3
2
=3 [𝑧 2 ]+c
3
2
=3 [ (tan−1 𝑥 )2 ]+c Ans :

3. ∫ 𝑠𝑖𝑛2 𝑥 cos 𝑥𝑑𝑥

Solution:

Let I =∫ 𝑠𝑖𝑛2 𝑥 cos 𝑥𝑑𝑥 put 𝑧 = sin 𝑥 or, 𝑑𝑧 = cos 𝑥 𝑑𝑥

= ∫ 𝒛𝟐 𝒅𝒛
𝒛𝟑
= +𝒄
𝟑
𝟏
= 𝟑 𝒔𝒊𝒏𝟑 𝒙 + 𝒄 (Ans)

(sin−1 𝑥)2
4. ∫ 𝑑𝑥
√1 − 𝑥 2
2
(sin−1 𝑥) 1
Solution : Let I =∫ √1−𝑥 2
𝑑𝑥 put, sin−1 𝑥 = 𝑧 or, √1−𝑥2 𝑑𝑥 = 𝑑𝑧

= ∫ 𝒛𝟐 𝒅𝒛

71 | P a g e
Module-08
Method of Substitution & Integration by Parts
𝒛𝟑
=𝟑 + 𝒄
𝟏
=𝟑 (𝐬𝐢𝐧−𝟏 𝒙)𝟑 + 𝒄 (Ans)

1
5. dx
e  e x
x

ex
  x x  x dx
e (e  e )
ex
 dx
e2 x  1
ex
  x 2 dxLet , e x  z  e x dx  dz
(e )  1
1
  2 dx
z 1
 tan 1 z  c
 tan 1 (e x )  c(Ans.)

6. tan 3 x sec xdx


  tan 2 x sec x .tan xdx
(z 2  1) z
 dxLet ,sec x  z  sec x tan xdx  dz
z
z2 1
 dx
z
3 1
  (z 2  z 2 )dz
5 1
z2 z2
  c
5 1
2 2
5
2
 (sec x) 2  2 sec x  c(Ans.)
5

𝑐𝑜𝑠2𝑥
7.∫ 1+𝑠𝑖𝑛2𝑥 𝑑𝑥

Let
1
1+sin2x=z ⟹ 2𝑐𝑜𝑠2𝑥 𝑑𝑥 = 𝑑𝑧 ∴ 𝑐𝑜𝑠2𝑥 = 2 𝑑𝑧

72 | P a g e
Module-08
Method of Substitution & Integration by Parts
𝑐𝑜𝑠2𝑥 1 1 1 1
∫ 2
𝑑𝑥 = ∫ 𝑑𝑧 = 𝑙𝑛𝑧 + 𝑐 = ln(1 + 𝑠𝑖𝑛2𝑥) + 𝑐
(𝑠𝑖𝑛𝑥 + 𝑐𝑜𝑠𝑥) 2 𝑧 2 2

8.9 Evaluate the following integrals:


𝑑𝑥 𝑑𝑥 𝑑𝑥
(i). ∫ √𝑎2 (ii). ∫ 𝑎2 +𝑥2 (iii). ∫
−𝑥 2 𝑥√𝑥 2 −𝑎2

Solution:

(i) Let 𝒙 = 𝒂𝒔𝒊𝒏𝜽 ∴ 𝒅𝒙 = 𝒂𝒄𝒐𝒔𝜽 𝒅𝜽


𝒅𝒙 𝒂𝒄𝒐𝒔𝜽 𝒅𝜽 𝒂𝒄𝒐𝒔𝜽 𝒅𝜽
∫ =∫ =∫ = ∫ 𝒅𝜽 = 𝜽
√𝒂𝟐 − 𝒙𝟐 √𝒂𝟐 − (𝒂𝒔𝒊𝒏𝜽)𝟐 √𝒂𝟐 (𝟏 − 𝒔𝒊𝒏𝟐 𝜽)
𝒙
= 𝒔𝒊𝒏−𝟏 + 𝒄
𝒂

Integration by Parts

8.10 Statement: If u and v are two function of x, then


𝑑𝑢
∫ 𝑢. 𝑣 𝑑𝑥 = 𝑢 ∫ 𝑣 𝑑𝑥 − ∫[ ∫ 𝑣 𝑑𝑥]𝑑𝑥
𝑑𝑣

8.10 Rule: Care should be taken to choose the first function in the rule of integration by parts.

1. If one factor of the product is a power of x, take it as the first function provided the
integral of the second function is handy. If however, the integral of the second function is
not readily available [ In case of inverse circular function, or inverse function function, or
logarithmic or hyperbolic function] in that case, take the factor as the first function.
2. If the integrand is a single inverse circular or hyperbolic or single logarithmic function as
the first function and unity as the second function.

8.11 Evaluate the following integrals:

1. ∫ 𝒍𝒏𝒙 𝒅𝒙
Solution:
𝒅
𝒍𝒏𝒙 ∫ 𝟏 𝒅𝒙 − ∫{𝒅𝒙 (𝒍𝒏𝒙) ∫ 𝟏 𝒅𝒙}𝒅𝒙 = 𝒙𝒍𝒏𝒙 − 𝒙 + 𝒄
2. ∫ 𝒙𝟐 𝒆𝒙 𝒅𝒙
Solution

73 | P a g e
Module-08
Method of Substitution & Integration by Parts
I=∫ 𝒙𝟐 𝒆𝒙 𝒅𝒙
𝒅
= 𝒙𝟐 ∫ 𝒆𝒙 𝒅𝒙 − ∫{𝒅𝒙 (𝒙𝟐 )∫ 𝒆𝒙 𝒅𝒙} 𝒅𝒙
= 𝒙𝟐 𝒆𝒙 -∫ 𝟐𝒙 𝒆𝒙 𝒅𝒙
𝒅
=𝒙𝟐 𝒆𝒙 -2[x∫ 𝒆𝒙 𝒅𝒙-∫{𝒅𝒙 (x)∫ 𝒆𝒙 𝒅𝒙}dx
= 𝒙𝟐 𝒆𝒙 − 𝟐[x𝒆𝒙 − 𝒆𝒙 ] + 𝒄

= 𝑥 2 𝑒 𝑥 -2x𝑒 𝑥 +2𝑒 𝑥 +c Ans:

3.∫ 𝑥𝑙𝑜𝑔𝑥𝑑𝑥

Solution:
𝒅
= 𝒍𝒏𝒙 ∫ 𝒙𝒅𝒙-∫{ 𝒅𝒙 (logx)∫ 𝒙𝒅𝒙}𝒅𝒙
𝒙𝟐 𝟏 𝒙𝟐
= 𝒍𝒏𝒙 𝟐 - ∫ 𝒙 𝒅𝒙
𝟐
𝒙𝟐 𝑿
=𝒍𝒏𝒙 𝟐 - ∫ 𝟐 𝒅𝒙
𝒙𝟐 𝟏
=𝒍𝒏𝒙 𝟐 -𝟐 ∫ 𝒙 𝒅𝒙
𝒙𝟐 𝟏 𝒙𝟐
=𝒍𝒏𝒙 𝟐 - 𝟐 +c
𝟐
𝒙𝟐 𝒙𝟐
= logx - 𝟒 + c Ans:
𝟐

4. ∫ 𝒙𝒕𝒂𝒏−𝟏 𝒙𝒅𝒙
𝒅
= 𝒕𝒂𝒏−𝟏 𝒙 ∫ 𝒙 𝒅𝒙 − ∫{𝒅𝒙 (𝒕𝒂𝒏−𝟏 𝒙)∫ 𝒙𝒅𝒙} 𝒅𝒙
𝒙𝟐 𝟏 𝒙𝟐
=𝒕𝒂𝒏−𝟏 𝒙 𝟐 − ∫ 𝟏+𝒙𝟐 𝟐 𝒅𝒙
𝒙𝟐 𝟏 𝒙𝟐
= 𝟐 𝒕𝒂𝒏−𝟏 𝒙-𝟐 ∫ 𝟏+𝒙𝟐 𝒅𝒙
𝒙𝟐 𝟏 𝟏
= 𝟐 𝒕𝒂𝒏−𝟏 𝒙-𝟐 ∫(𝟏 − 𝟏+𝒙𝟐 )𝒅𝒙
𝒙𝟐 𝟏
= 𝒕𝒂𝒏−𝟏 𝒙 − 𝟐 (𝒙 − 𝒕𝒂𝒏−𝟏 𝒙) + 𝒄
𝟐

𝒙
5. ∫ 𝟏+𝒄𝒐𝒔𝒙 𝒅𝒙
𝒙
=∫ 𝟐𝒄𝒐𝒔𝟐 𝒙 𝒅𝒙
𝟏 𝒙
=∫ 𝒙. 𝟐 𝒔𝒆𝒄𝟐 𝒙 𝟐 𝒅𝒙
𝟏 𝒙 𝒅 𝟏 𝒙
=𝒙 ∫ 𝟐 𝒔𝒆𝒄𝟐 𝒙 𝟐 𝒅𝒙 − ∫{𝒅𝒙 (𝒙) ∫ 𝒙. 𝟐 𝒔𝒆𝒄𝟐 𝒙 𝟐 𝒅𝒙}𝒅𝒙
𝒙 𝒙
=𝒙 𝒕𝒂𝒏 𝟐 − ∫ 𝒕𝒂𝒏 𝟐 𝒅𝒙

74 | P a g e
Module-08
Method of Substitution & Integration by Parts
𝒙
𝒙 𝒄𝒐𝒔
𝟐
= 𝒙 𝒕𝒂𝒏 𝟐 − [−𝒍𝒏 ]+c
𝟏/𝟐

𝒙𝟐 +𝟏
6. ∫ 𝒆𝒙 (𝒙+𝟏)𝟐 𝒅𝒙
(𝒙+𝟏)𝟐 −𝟐𝒙
=∫ 𝒆𝒙 𝒅𝒙
(𝒙+𝟏)𝟐
𝒙𝒆𝒙
=∫ 𝒆𝒙 𝒅𝒙-2∫ (𝒙+𝟏)𝟐 𝒅𝒙
𝟏 𝟏
=𝒆𝒙 − 𝟐 ∫(𝒙+𝟏 − (𝒙+𝟏)𝟐 )𝒆𝒙 𝒅𝒙
𝟏 𝟏
=𝒆𝒙 − 𝟐 ∫(𝒙+𝟏 𝒆𝒙 𝒅𝒙) − ∫((𝒙+𝟏)𝟐 )𝒆𝒙 𝒅𝒙
𝟐𝒆𝒙
=𝒆𝒙 − 𝒙+𝟏+c

d 3 2x
xv. x 3e 2 x dx  x 3  e 2 x dx   [ (x )  e dx]dx
dx
2x 2x
3 e 2 e

2 
x  (3 x )dx
2
e2 x 3
 x3   (x 2 e 2 x )dx
2 2
2x
e 3 d
 x3  [x 2  (e 2 x )dx   { (x 2 ). (e 2 x )dx}dx]
2 2 dx
2x 2x
e 3 e e2 x
 x3  [x 2   {2 x . }dx]
2 2 2 2
2x 2x
e 3 e
2 
 x3  [x 2  {x .e 2 x }dx]
2 2
e2 x 3x 2e2 x 3 d
 x3   [ x  e 2 x dx   { (x). (e 2 x )dx}dx]
2 4 2 dx
2x 2 2x
e 3x e 3 e2 x
 x3   [ xe 2 x   {1. }dx]
2 4 2 2
2x 2 2x 2x
e 3x e 3xe 3
 x3     e 2 x dx
2 4 4 4
2x 2 2x 2x
e 3x e 3xe 3 e2 x
 x3    . c
2 4 4 4 2
2x
3 e 3x 2 e 2 x 3xe 2 x 3 2 x
x    e c
2 4 4 8

75 | P a g e
Module-08
Method of Substitution & Integration by Parts

Try yourself

i. log xdx ii. xe x dx iii. x 2 log xdx iv. x sin xdxv.
𝒙𝟑 𝒙𝟑
Ans:(𝒊)𝒙 𝐥𝐨𝐠 𝒙 − 𝒙 + 𝒄(𝒊𝒊)𝒙𝒆𝒙 − 𝒆𝒙 + 𝒄(𝒊𝒊𝒊) 𝐥𝐨𝐠 𝒙 − + 𝒄(𝒊𝒗) − 𝒙 𝐜𝐨𝐬 𝒙 + 𝐬𝐢𝐧 𝒙 +
𝟑 𝟗
𝒄.

8.12 Evaluate the following Standard Form:

Q. ∫ √𝑎2 − 𝑥 2 𝑑𝑥

Solution:

Let I=∫ √𝑎2 − 𝑥 2 𝑑𝑥


𝑑
I=√𝑎2 − 𝑥 2 ∫ 1 𝑑𝑥 − ∫{𝑑𝑥 (√𝑎2 − 𝑥 2 ) ∫ 1 𝑑𝑥 } 𝑑𝑥
1
= √𝑎2 − 𝑥 2 . 𝑥 − ∫ . (−2𝑥)𝑥 𝑑𝑥
2√𝑎2 −𝑥 2

𝑥2
=𝑥√𝑎2 − 𝑥 2 + ∫ √𝑎2 𝑑𝑥
−𝑥 2

𝑎2 −𝑎2 +𝑥 2
= 𝑥√𝑎2 − 𝑥 2 + ∫ √𝑎2 −𝑥 2
𝑑𝑥

𝑎2 𝑎2 −𝑥 2
= 𝑥√𝑎2 − 𝑥 2 + ∫ √𝑎2 𝑑𝑥 − ∫ √𝑎2 𝑑𝑥
−𝑥 2 −𝑥 2
𝑥
= 𝑥√𝑎2 − 𝑥 2 + 𝑎2 𝑠𝑖𝑛−1 𝑎-I
1 𝑥
I=2 (√𝑎2 − 𝑥 2 + 𝑎2 𝑠𝑖𝑛−1 𝑎)

Try yourself:

1. ∫ √𝒂𝟐 + 𝒙𝟐 𝒅𝒙 2. ∫ √𝒙𝟐 − 𝒂𝟐 𝒅𝒙

76 | P a g e
Module-09
Definite integral

Definite Integral:
A function f is said to be integral on a finite closed interval [a, b] if the limit
𝑛

lim ∑ 𝑓(𝑥𝑘∗ )∆𝑥𝑘


𝑚𝑎𝑥∆𝑥→0
𝑘=1
exists and does not depend on the choice of partitions or on the choice of the points 𝑥𝑘∗ in the sub
intervals. When this is the case we denote the limit by the symbol
𝑏 𝑛

∫ 𝑓(𝑥)𝑑𝑥 = lim ∑ 𝑓(𝑥𝑘∗ )∆𝑥𝑘


𝑎 𝑚𝑎𝑥∆𝑥→0
𝑘=1
which is called the definite integral of 𝑓 from a to b. The numbers 𝑎 and 𝑏 are called the lower
limit of integration and the upper limit of integration respectively, and 𝑓(𝑥) is called the integrand.
The integral symbol and the symbol “𝑑𝑥” can serve as reminders that the definite integral is
actually a limit of a summation as ∆𝑥𝑘 → 0 . The sum that appears in definition is called a Riemann
sum and the definite integral is sometimes called the Riemann integral.

4 4
Example 01 : ∫1 2𝑑𝑥 = 2 ∫1 𝑑𝑥 = 2(𝑥)14 = 2(4 − 1) = 2 ∗ 3 = 6.
Example 02: Sketch the region whose area is represented by the definite integral, and evaluate
4
the integral using an appropriate formula from geometry:∫1 2𝑑𝑥
Solution: The graph of the integrand is the horizontal line y = 2, so the region is a rectangle of
height 2 extending over the interval from 1 to 4 (Figure 9.1). Thus,
4

∫ 2𝑑𝑥 = 𝑎𝑟𝑒𝑎 𝑜𝑓 𝑡𝑟𝑖𝑎𝑛𝑔𝑙𝑒 = 2(3) = 6.


1

Figure: 9.1

PROPERTIES OF THE DEFINITE INTEGRAL:


It is assumed in Definition that [𝑎, 𝑏] is a finite closed interval with 𝑎 < 𝑏 and hence the upper
limit of integration in the definite integral is greater than the lower limit of integration. However,

77 | P a g e
Module-09
Definite integral
it will be convenient to extend this definition to allow for cases in which the upper and lower
limits of integration are equal or the lower limit of integration is greater than the upper limit of
integration. For this purpose we make the following special definitions.
𝑎 1
(a) If a is in the domain of f , we define ∫𝑎 𝑓(𝑥)𝑑𝑥 = 0. Ex: ∫1 𝑥 2 𝑑𝑥 = 0.
𝑏 𝑎
(b) If 𝑓 is integrable on [𝑎, 𝑏], then we define ∫𝑎 𝑓(𝑥)𝑑𝑥 = − ∫𝑏 𝑓(𝑥).
0 1 𝜋
Ex: ∫1 √(1 − 𝑥 2 ) 𝑑𝑥 = − ∫0 √(1 − 𝑥 2 ) 𝑑𝑥 = − 4 .

Theorem 01: If 𝑓 and 𝑔 are integrable on [𝑎, 𝑏] and if c is a constant, then 𝑐𝑓, 𝑓 + 𝑔 and

𝑓 − 𝑔 are integrable on [𝑎, 𝑏] and


𝑏 𝑏
𝑎) ∫𝑎 𝑐𝑓(𝑥) 𝑑𝑥 = 𝑐 ∫𝑎 𝑓(𝑥)𝑑𝑥.
𝑏 𝑏 𝑏
𝑏) ∫𝑎 [𝑓(𝑥) + 𝑔(𝑥)]𝑑𝑥 = ∫𝑎 𝑓(𝑥)𝑑𝑥 + ∫𝑎 𝑔(𝑥)𝑑𝑥.
𝑏 𝑏 𝑏
𝑐) ∫𝑎 [𝑓(𝑥) − 𝑔(𝑥)]𝑑𝑥 = ∫𝑎 𝑓(𝑥)𝑑𝑥 − ∫𝑎 𝑔(𝑥)𝑑𝑥.
1
Example 03: Evaluate ∫0 [5 − 3√(1 − 𝑥 2 )]𝑑𝑥

Solution: From parts (a) and (c) of Theorem we can write


1
∫0 [5 − 3√(1 − 𝑥 2 )]𝑑𝑥
1 1
= ∫0 5 𝑑𝑥 − ∫0 3(√(1 − 𝑥 2 )𝑑𝑥
1 1
= ∫0 5 𝑑𝑥 − 3 ∫0 (√(1 − 𝑥 2 )𝑑𝑥

The first integral in this difference can be interpreted as the area of a rectangle of height 5 and
𝜋
base 1, so its value is 5, and from properties (b), the value of the second integral is 4 . Thus,
1
∫0 [5 − 3√(1 − 𝑥 2 )]𝑑𝑥
𝜋
= 5(1 − 0) − 3 ( )
4

3𝜋
=5− .
4

Theorem 02: If 𝑓 is integrable on a closed interval containing the three points 𝑎, 𝑏, and 𝑐, then
𝑏 𝑐 𝑏
∫𝑎 𝑓(𝑥)𝑑𝑥 = ∫𝑎 𝑓(𝑥)𝑑𝑥 + ∫𝑐 𝑓(𝑥)𝑑𝑥,

no matter how the points are ordered.

78 | P a g e
Module-09
Definite integral

Definition of bounded function: A function f that is defined on an interval is said to be bounded


on the interval if there is a positive number 𝑀 such that −𝑀 ≤ 𝑓 (𝑥) ≤ 𝑀 for all 𝑥 in the
interval. Geometrically, this means that the graph of 𝑓 over the interval lies between the lines
𝑦 = −𝑀 and 𝑦 = 𝑀.

THE FUNDAMENTAL THEOREM OF CALCULUS:


In this section we will establish two basic relationships between definite and indefinite integrals
that together constitute a result called the “Fundamental Theorem of Calculus.” One part of this
theorem will relate the rectangle and antiderivative methods for calculating areas, and the second
part will provide a powerful method for evaluating definite integrals using antiderivatives.

Let us begin by assuming that f is nonnegative and continuous on an interval [𝑎, 𝑏], in which
case the area A under the graph of 𝑓 over the interval [𝑎, 𝑏] is represented by the definite integral
𝑏
𝐴 = ∫𝑎 𝑓(𝑥)𝑑𝑥 … … … … (1) in Figure 9.2.

Figure:9.2
If A(x) is the area under the graph of 𝑓 from a to x (Figure 9.3), then

• 𝐴 (𝑥) = 𝑓 (𝑥)

• 𝐴(𝑎) = 0

• 𝐴(𝑏) = 𝐴

Figure:9.3

79 | P a g e
Module-09
Definite integral

The formula 𝐴(𝑥) = 𝑓(𝑥) states that 𝐴(𝑥) is an antiderivative of 𝑓(𝑥), which implies that every
other antiderivative of 𝑓(𝑥) on [𝑎, 𝑏] can be obtained by adding a constant to 𝐴(𝑥).Accordingly,
let 𝐹(𝑥) = 𝐴(𝑥) + 𝐶be any anti-derivative of f (x), and consider what happens when we
subtract 𝑓(𝑎) from 𝑓(𝑏):

𝐹(𝑏) − 𝐹(𝑎) = [𝐴(𝑏) + 𝐶] − [𝐴(𝑎) + 𝐶] = 𝐴(𝑏) − 𝐴(𝑎) = 𝐴 − 0 = 𝐴.


Hence (1) can be expressed as
𝑏
∫ 𝑓(𝑥)𝑑𝑥 = 𝐹(𝑏) − 𝐹(𝑎).
𝑎

In words, this equation states:


The definite integral can be evaluated by finding any antiderivative of the integrand and
then subtracting the value of this antiderivative at the lower limit of integration from its
value at the upper limit of integration.
2
Example 04: Evaluate ∫1 𝑥𝑑𝑥.

𝑥2
Solution: The function 𝐹(𝑥) = is an antiderivative of 𝑓(𝑥) = 𝑥;
2

2 𝑥2 2 (2)2 (1)2 1 3
Thus ∫1 𝑥𝑑𝑥 = | | = − =2− = .
2 1 2 2 2 2

Example 05: ( a) Find the area under the curve 𝑦 = 𝑐𝑜𝑠𝑥 over the interval [0, π/2] (Figure 9.4).
𝜋
(b) Make a conjecture about the value of the integral∫0 𝑐𝑜𝑠𝑥 𝑑𝑥 and confirm your
conjecture using the Fundamental Theorem of Calculus.

Figure 9.4

Solution (a): Since 𝑐𝑜𝑠𝑥 ≥ 0 over the interval [0, 𝜋/2], the area A under the curve is
𝜋/2 𝜋/2 𝜋
𝐴 = ∫0 𝑐𝑜𝑠𝑥 𝑑𝑥 = |𝑠𝑖𝑛𝑥| = sin ( 2 ) − sin(0) = 1.
0

80 | P a g e
Module-09
Definite integral
Solution (b): The given integral can be interpreted as the signed area between the graph
of 𝑦 = 𝑐𝑜𝑠𝑥and the interval [0, π]. The graph in Figure 9.4 suggests that over the interval
[0, π] the portion of area above the x-axis is the same as the portion of area below the x-axis,so we
conjecture that the signed area is zero; this implies that the value of the integral is
zero. This is confirmed by the computations
𝜋 𝜋
∫0 𝑐𝑜𝑠𝑥 𝑑𝑥 = |𝑠𝑖𝑛𝑥| 0 = sin(𝜋) − sin(0) = 0.
9
Example 06: Evaluate ∫1 √𝑥 𝑑𝑥
9
Solution: Given Integral ∫1 √𝑥 𝑑𝑥

9 9 𝑥 3/2 9 2 2 52
∫1 √𝑥 𝑑𝑥 = ∫1 𝑥1/2 𝑑𝑥 = | 3/2 | = 3 [(9)3/2 − (1)3/2 ] = 3 (27 − 1) = 3 .
1
Example 07:
9 9 𝑥 7/2 9 2 2 4118
(𝑖) ∫4 𝑥 2 √𝑥 𝑑𝑥 = ∫4 𝑥 5/2 𝑑𝑥 = | 7/2 | = 7 [(9)7/2 − (4)7/2 ] = 7 (2187 − 128) = 7 .
4
𝜋
𝜋/2 𝑠𝑖𝑛𝑥 1 1 −1 1
(𝑖𝑖) ∫0 𝑑𝑥 = − 5 [𝑐𝑜𝑠𝑥] 2 = − 5 [𝑐𝑜𝑠(𝜋/2) − cos(0)] = [0 − 1] = .
5 5 5
0
𝜋 𝜋
3
2
(𝑖𝑖𝑖) ∫ 𝑠𝑒𝑐 𝑥 𝑑𝑥 = [𝑡𝑎𝑛𝑥] 3 = [𝑡𝑎𝑛(𝜋/3) − tan(0)] = [√3 − 0] = √3.
0 0
𝜋/4 𝜋/4
(𝑖𝑣) ∫−𝜋/4 𝑠𝑒𝑐𝑥𝑡𝑎𝑛𝑥𝑑𝑥 = [𝑠𝑒𝑐𝑥] = [𝑠𝑒𝑐(𝜋/4) − sec(−𝜋/4)] = √2 − √2 = 0.
−𝜋/4

𝜋
Example 08: Evaluate ∫02 𝑠𝑖𝑛2 𝑥𝑑𝑥
𝜋
Solution: Given Integral is ∫02 𝑠𝑖𝑛2 𝑥𝑑𝑥
𝜋
Let, I = ∫02 𝑠𝑖𝑛2 𝑥𝑑𝑥
𝜋
1
=2 ∫02 2𝑠𝑖𝑛2 𝑥𝑑𝑥
𝝅
𝟏
=𝟐 ∫𝟎𝟐 (𝟏 − 𝒄𝒐𝒔𝟐𝒙)𝒅𝒙
𝝅 𝝅
𝟏 𝟏
=𝟐 ∫𝟎𝟐 𝟏𝒅𝒙 − ∫𝟐 𝒄𝒐𝒔𝟐𝒙𝒅𝒙
𝟐 𝟎

81 | P a g e
Module-09
Definite integral
𝝅 𝝅
𝟏 𝟏 𝒔𝒊𝒏𝟐𝒙
= 𝟐 [𝒙] 𝟎𝟐 − [ ] 𝟎𝟐
𝟐 𝟐

𝜋 𝜋
1 1 𝑠𝑖𝑛2𝑥
=2 [𝑥] 02 − [ ] 02
2 2

1 𝜋 1 𝜋
= 2 [ 2 − 0] − [𝑠𝑖𝑛2 2 − sin 2.0]
4

𝜋 1 𝜋 1
=4− [𝑠𝑖𝑛𝜋 − sin 0]= − .0
4 4 4
𝜋
=4

𝜋/2 𝑠𝑖𝑛2 𝜃
Example 09: Evaluate ∫0 𝑑𝜃
1+𝑐𝑜𝑠𝜃

𝜋/2 𝑠𝑖𝑛2 𝜃
Solution: Given Integral is ∫0 𝑑𝜃
1+𝑐𝑜𝑠𝜃
𝜋
𝑠𝑖𝑛2 𝜃
Let 𝐼 = ∫02 1+𝑐𝑜𝑠𝜃 𝑑𝜃

𝜋/2 1−𝐶𝑂𝑆2 𝜃
= ∫0 𝑑𝜃
1+𝑐𝑜𝑠𝜃

𝜋/2 (1+𝑐𝑜𝑠𝜃)(1−𝑐𝑜𝑠𝜃)
= ∫0 𝑑𝜃
(1+𝑐𝑜𝑠𝜃)

𝜋/2
= ∫0 (1 − 𝑐𝑜𝑠𝜃)𝑑𝜃
𝜋/2 𝜋/2
= ∫0 𝑑𝜃 − ∫0 𝑐𝑜𝑠𝜃𝑑𝜃
𝜋 𝜋
= [𝜃] 2 − [𝑠𝑖𝑛𝜃] 2
0 0
𝜋 𝜋
= [ 2 − 0] − [sin ( 2 ) − sin(0)]
𝜋
= 2 − 1.

Example 10: Evaluate the following:


3 4 1 3 1 1 1
(i).∫2 2𝑥𝑑𝑥 (ii).∫1 𝑑𝑥 (iii).∫1 (4 + 3𝑥 − 𝑥 3 ) 𝑑𝑥 (iv).∫0 (√𝑥 + 𝑥) 𝑑𝑥 (v).∫−3(𝑥 2 + 2)𝑑𝑥
𝑥3 √
1 3 2 2
(vi).∫−3(5 − 2𝑥) 𝑑𝑥 (vii). ∫1 (4 + 3𝑥 − 𝑥 2 ) 𝑑𝑥 (viii).∫0 (2𝑥 − 𝑥 2 )𝑑𝑥 (ix).∫1 (𝑥 2 + 2𝑥 + 5)𝑑𝑥

Solutions:
82 | P a g e
Module-09
Definite integral
3 3 𝑥 1+1
i.∫2 2𝑥𝑑𝑥 =2 ∫2 𝑥𝑑𝑥=[2. 1+1 ]32 =[32 − 22 ]=[9-4]=5

4 1 4 𝑥 −3+1 𝑥 −2 1 1 1 1 1 15
ii.∫1 𝑑𝑥 = ∫1 𝑥 −3 𝑑𝑥=[ −3+1 ]14 =[ −2 ]14 =[−2𝑥 2 ]14 =[−2.42 − (−2.12)]14]=[− 32 + 2]=32(Ans.)
𝑥3

3 3𝑥 2 𝑥 4 3
iii.∫1 (4 + 3𝑥 − 𝑥 3 ) 𝑑𝑥=[4x+ - ]
2 4 1

3.32 34 3.12 14
= [(4.3+ - 4 ) − (4.1 + − 4 )]
2 2

27 81 3 1
=[(12+ 2 - 4 )-( 4 + 2 − 4)]
27 81 3 1
=[12+ 2 - 4 )-(4 − 2 + 4]
48+54−81−16−6+1
= 4

0
=4 = 0(Ans.)
1 −1
1 1 1
iv.∫0 (√𝑥 + ) 𝑑𝑥 = ∫0 (𝑥 2 + 𝑥 2 ) 𝑑𝑥
√ 𝑥
1 −1
+1 +1
𝑥2 𝑥2
=[ 1 + −1 ]10
+1 +1
2 2

3 1
𝑥2 𝑥2 1
=[ 3 + 1 ]0
2 2

3 1 3 1
12 12 02 02
=( 3 + 1 )-( 3 + 1 )
2 2 2 2

3
=2 + 2 − (0 + 0)
8
= 3 (Ans.)

1 𝑥3 13 (−3)3 1
v.∫−3(𝑥 2 + 2)𝑑𝑥 = [. 3 + 2𝑥]1−3 =( 3 + 2.1) - ( + 2(−3)) =3 + 2 − (−9 − 6)
3

1 1+6+27+18 52
= 3 + 2 + 9 + 6= = 3 (Ans.)
3

83 | P a g e
Module-09
Definite integral
1
x2 1
vi.  (5  2 x)dx  [5 x  2 ] 3
3
2
 [5 x  x 2 ]13
 5.1  12  [5.(3)  (3) 2 ]
 5  1  (15  6)
 4  21  25(Ans.)
3
3x 2 x3 3
vii. (4  3x  x ) dx  [4 x 
2
 ]1
1
2 3
3.32 33 3.12 13
 [4.3   ]  [4.1   ]
2 3 2 3
27 27 3 1
 [12    (4   )]
2 3 2 3
27 27 3 1
 [12    4   )]
2 3 2 3
72  81  54  24  9  2

6
68 34
  (Ans.)
6 3
2
x 2 x3 2 x3 2
viii. (2 x  x )  [2.  ]0  [ x  ]0
2 2

0
2 3 3
23 03 8 12  8 4
 22   (02  )  4    ( Ans.)
3 3 3 3 3
2
x 2 1 x11
2
ix. ( x  2 x  5)dx  [
2
 2.  5 x]
1
2 1 11 1

x3 x2
[
 2.  5 x]12
3 2
3 2
2 2 13 12
 [  2.  5.2]  [  2.  5.1]
3 2 3 2
8 1
  4  10   1  5
3 3
8 1 8  1  24 31
  8  (Ans)
3 3 3 3

84 | P a g e
Module-09
Definite integral
3 x2 , x < 2
Example 11: Evaluate∫0 f(x)dx if f(x) = { }
3x − 2, x ≥ 2
Solution: We can integrate from 0 to 2 and from 2 to 3 separately and add the results. This yields

Figure:9.5
3 2 3
∫0 f(x)dx = ∫0 f(x)dx + ∫2 f(x)dx
2 3
= ∫0 x 2 dx + ∫2 (3x − 2)dx

x3 2 3x2 3
= ] + [ − 2x]
3 0 2 2
8 15
= (3 − 0) + ( 2 − 2)
49
= .
6

Example 12: Find the total area between the curve 𝑦 = 1 − 𝑥 2 and the x-axis over the
interval [0, 2] (Figure 9.6).

Figure:9.6

Solution: Given Integral, 𝑦 = 1 − 𝑥 2 .

The area A is given by


2 1 2
𝑥3 1 𝑥3 2
𝐴 = ∫ (1 − 𝑥 2 )𝑑𝑥 = ∫ (1 − 𝑥 2 )𝑑𝑥 + ∫ −(1 − 𝑥 2 )𝑑𝑥 = [𝑥 − ] − [𝑥 − ]
0 0 1 3 0 3 1

85 | P a g e
Module-09
Definite integral
2 4
= − [− ]
3 3
= 2.

[Note: To compute total area using Formula (7), begin by dividing the interval of integration
into subintervals on which f(x) does not change sign. On the subintervals for which 0 ≤ f(x)
replace |f(x)| by f(x), and on the subintervals for which f(x) ≤ 0 replace |f(x)| by -f(x). Adding the
resulting integrals then yields the total area.]

EVALUATING DEFINITE INTEGRALS BY SUBSTITUTION:

Indefinite integrals of the form ∫ 𝑓(𝑔(𝑥))𝑔′ (𝑥)𝑑𝑥 can sometimes be evaluated by making the u-
substitution
u = g(x), du = g (x) dx …………(1)

Make the substitution (1) directly in the definite integral, and then use the relationship
u = g(x) to replace the x-limits, x = a and x = b, by corresponding u-limits, u = g(a)
𝑔(𝑏)
and u = g(b). This produces a new definite integral ∫𝑔(𝑎) 𝑓(𝑢)𝑑𝑢 that is expressed entirely in
terms of u.
2
Example 13: Use the method above to evaluate ∫0 𝑥(𝑥 2 + 1)3 𝑑𝑥.
2
Solution : Given Integral, ∫0 𝑥(𝑥 2 + 1)3 𝑑𝑥

Let, 𝑢 = 𝑥 2 + 1

So, 𝑑𝑢 = 2𝑥𝑑𝑥 + 0 = 2𝑥𝑑𝑥

We make the substitution = 𝑥 2 + 1, then

If 𝑥 = 0, 𝑢 = 1

If 𝑥 = 2, 𝑢 = 5
2
Thus, ∫0 𝑥(𝑥 2 + 1)3 𝑑𝑥
1 5
= 2 ∫1 𝑢3 𝑑𝑢

𝑢4 5
= ]
8 1
625 1
= −8
8

= 78.

86 | P a g e
Module-09
Definite integral
𝜋/8 5
Example 14: Evaluate 𝑎) ∫0 𝑠𝑖𝑛5 2𝑥𝑐𝑜𝑠2𝑥𝑑𝑥 𝑏) ∫2 (𝑥 − 3)9 (2𝑥 − 5)𝑑𝑥
𝜋/8
a)Solution : Given Integral, ∫0 𝑠𝑖𝑛5 2𝑥𝑐𝑜𝑠2𝑥𝑑𝑥

Let, 𝑢 = 𝑠𝑖𝑛2𝑥

So, 𝑑𝑢 = 2𝑐𝑜𝑠2𝑥𝑑𝑥
1
Or, 2 𝑑𝑢 = 𝑐𝑜𝑠2𝑥𝑑𝑥

We make the substitution, then

If 𝑥 = 0, 𝑢 = 𝑠𝑖𝑛(0) = 0
𝜋 𝜋 1
If 𝑥 = 8 , 𝑢 = sin ( 4 ) =
√2
𝜋
Thus, ∫08 𝑠𝑖𝑛5 2𝑥𝑐𝑜𝑠2𝑥𝑑𝑥
1
1
= 2 ∫0 𝑢5 𝑑𝑢
√2

1 𝑢6 5
= 2[ 6]
1
1 1
= 2[ 6 − 0]
6(√2)

1
= 96.
5
b)Solution : Given Integral, ∫2 (𝑥 − 3)9 (2𝑥 − 5)𝑑𝑥

Let, 𝑢 = 𝑥 − 3

So, 𝑑𝑢 = 𝑑𝑥

This leaves a factor of 2𝑥 − 5 unresolved in the integrand. However,

𝑢 = 𝑥 − 3 𝑠𝑜 2𝑥 − 5 = 2(𝑢 + 3) − 5 = 2𝑢 + 6 − 5 = 2𝑢 + 1.
We make the substitution, then

If 𝑥 = 2, 𝑢 = 2 − 3 = −1

If 𝑥 = 5, 𝑢 = 5 − 3 = 2
5
Thus, ∫2 (𝑥 − 3)9 (2𝑥 − 5)𝑑𝑥

87 | P a g e
Module-09
Definite integral
2
= ∫−1(2𝑢 + 1)𝑢9 𝑑𝑢
2
= ∫−1(2𝑢10 + 𝑢9 )𝑑𝑢

2𝑢11 𝑢10 2
=[ + ]
11 10 −1
212 210 −2 1
= [ 11 + ] − [ 11 + 10]
10

52233
= ≈ 474.8.
110
𝜋
3 cos( )
Example 15: Evaluate ∫1 𝑥 2𝑥 𝑑𝑥
𝜋
3 cos( )
Solution: Given Integral, ∫1 𝑥 2𝑥 𝑑𝑥
𝜋
Let,𝑢 = 𝑥 so that
−𝜋 1
𝑑𝑢 = 𝑑𝑥 = −𝜋. 𝑥 2 𝑑𝑥
𝑥2

−1 1
or, 𝜋 𝑑𝑢 = 𝑥 2 𝑑𝑥

With this substitution,

If 𝑥 = 1, 𝑢 = 𝜋
𝜋
If 𝑥 = 3, 𝑢 = 3
𝜋
3 cos( )
Thus, ∫1 𝑥 2𝑥 𝑑𝑥
𝜋
1
= − 𝜋 ∫𝜋3 𝑐𝑜𝑠𝑢 𝑑𝑢
𝜋
1
= − 𝜋 𝑠𝑖𝑛𝑢] 3 𝑑𝑢
𝜋
1 𝜋
= − 𝜋 [sin (3 ) − sin(𝜋)]

√3
= − 2𝜋 ≈ −0.2757.

3 1
Example 16: Evaluate ∫0 𝑑𝑥
(𝑥+2)√(1+𝑥)

88 | P a g e
Module-09
Definite integral
3 1
Solution: Given Integral is ∫0 𝑑𝑥
(𝑥+2)√(1+𝑥)

Let, 1 + 𝑥 = 𝑧 2 so that 𝑑𝑥 = 2𝑧𝑑𝑧

With this substitution,

If 𝑥 = 0, 𝑧 = 1

If 𝑥 = 3, 𝑧 = 2
3 1
Now, ∫0 𝑑𝑥
(𝑥+2)√(1+𝑥)

2 2zdz
= ∫1 (𝑧 2 +1)𝑧

2 dz
= 2 ∫1 (𝑧 2 +1)

2
= 2[𝑡𝑎𝑛−1 𝑧]
1
= 2(𝑡𝑎𝑛−1 2 − 𝑡𝑎𝑛−1 1)
2−1
= 2𝑡𝑎𝑛−1 (1+2.1)

1
= 2𝑡𝑎𝑛−1 (3).
𝜋 1
Example 17: Evaluate ∫0 𝑑𝑥
(2+𝑐𝑜𝑠𝑥)

𝜋 1
Solution: Given Integral is ∫0 𝑑𝑥
(2+𝑐𝑜𝑠𝑥)

𝜋 1
Now, ∫0 (2+𝑐𝑜𝑠𝑥)
𝑑𝑥

𝜋 1
= ∫0 𝑥
1−𝑡𝑎𝑛2 ( )
𝑑𝑥
2
(2+ 𝑥 )
1+𝑡𝑎𝑛2 ( )
2

𝑥
𝜋 1+𝑡𝑎𝑛2 ( )
2
= ∫0 𝑥 𝑥 𝑑𝑥
(2+2𝑡𝑎𝑛2 ( )+1−𝑡𝑎𝑛2 ( )
2 2

2 𝑥
𝜋 𝑠𝑒𝑐 (2)
= ∫0 𝑥 𝑑𝑥
3+𝑡𝑎𝑛2 ( )
2

𝑥 1 𝑥 𝑥
Let, 𝑡𝑎𝑛 (2) 𝑑𝑥 = 𝑧 such that 2 𝑠𝑒𝑐 2 (2) 𝑑𝑥 = 𝑑𝑧 ⟹ 𝑠𝑒𝑐 2 (2) 𝑑𝑥 = 2𝑑𝑧.

With this substitution,


89 | P a g e
Module-09
Definite integral
If 𝑥 = 0, 𝑧 = 0

If 𝑥 = 𝜋/2, 𝑧 = ∞
𝜋 1
Thus, ∫0 (2+𝑐𝑜𝑠𝑥)
𝑑𝑥

∞ 2
= ∫0 (3+𝑧 2 )
𝑑𝑧

∞ 1
= 2 ∫0 𝑑𝑧
(√3)2 +𝑧 2

1 𝑧 ∞
= 2[ 𝑡𝑎𝑛−1 ( )]
√3 √3 0
2 𝑧 ∞
= [𝑡𝑎𝑛−1 ( )]
√3 √3 0
2
= (𝑡𝑎𝑛−1 ∞ − 𝑡𝑎𝑛−1 0)
√3

2 𝜋
= ( − 0)
√3 2
𝜋
= .
√3

1 1
Example 18: Evaluate the integral ∫0 𝑑𝑥
(1+𝑥)√1−𝑥 2

1 1
Solution: Given Integral is ∫0 (1+𝑥)√1−𝑥 2
𝑑𝑥

1 1
Now, ∫0 (1+𝑥)√1−𝑥 2
𝑑𝑥 [Let, 𝑥 = 𝑠𝑖𝑛𝜃 ⇒ 𝑑𝑥 = 𝑐𝑜𝑠𝜃𝑑𝜃, If 𝑥 = 0, 𝜃 = 0, If 𝑥 = 1, 𝜃 = π/2]
𝜋
2 cosθ
=∫ 𝑑𝜃
0 (1 + 𝑠𝑖𝑛𝜃)√1 − 𝑠𝑖𝑛2 𝜃
𝜋
2 cosθ
=∫ 𝑑𝜃
0 (1 + 𝑠𝑖𝑛𝜃)√𝑐𝑜𝑠 2 𝜃
𝜋
2 cosθ
=∫ 𝑑𝜃
0 (1 + 𝑠𝑖𝑛𝜃)𝑐𝑜𝑠𝜃
𝜋
2 1
=∫ 𝑑𝜃
0 (1 + 𝑠𝑖𝑛𝜃)
𝜋 1 𝜋
𝑐𝑜𝑠2 𝜃 𝑠𝑒𝑐 2 𝜃
= ∫0 2 𝑑𝜃 [𝐷𝑖𝑣𝑖𝑑𝑖𝑛𝑔 𝑏𝑦 𝑐𝑜𝑠 2 𝜃] = ∫02 𝑠𝑒𝑐 2 𝜃+𝑡𝑎𝑛2 𝜃 𝑑𝜃
(1+𝑠𝑖𝑛𝜃)/𝑐𝑜𝑠2 𝜃

90 | P a g e
Module-09
Definite integral
𝜋
𝑠𝑒𝑐 2 𝜃
= ∫02 1+2𝑡𝑎𝑛2 𝜃 𝑑𝜃 [Let, 𝑧 = 𝑡𝑎𝑛𝜃 ⇒ 𝑑𝑧 = 𝑠𝑒𝑐 2 𝜃𝑑𝜃If 𝜃 = 0, 𝑧 = 0,If 𝜃 = π/2, 𝑧 = ∞]
𝜋
dz
= ∫02 1+2𝑧 2
∞ dz
= ∫0 1
2( +𝑧 2 )
2

1 ∞ dz 1 ∞ dz 1 1 𝑧 ∞
= 2 ∫0 1 = 2 ∫0 1 2
= [2. 1 𝑡𝑎𝑛−1 ( 1 )]
( +𝑧 2 )
2 ( ) +𝑧 2 √2 √2
0
√2

1 ∞
=[ 𝑡𝑎𝑛−1 (√2𝑧)]
√2 0
1 ∞
= [𝑡𝑎𝑛−1 (√2𝑧)]
√2 0
1
= [𝑡𝑎𝑛−1 (∞) − 𝑡𝑎𝑛−1 (0)]
√2
𝜋
= 2√2.

𝜋/2 1
Example 19: Evaluate the integral ∫0 𝑑𝑥
(4+5𝑠𝑖𝑛𝑥)

𝜋/2 1
Solution: Given Integral is ∫0 𝑑𝑥
(4+5𝑠𝑖𝑛𝑥)

𝜋
1
Now, ∫02 (4+5𝑠𝑖𝑛𝑥) 𝑑𝑥
𝜋
1
= ∫02 𝑥
2 tan( )
𝑑𝑥n
2 )
(4+5 𝑥
1+𝑡𝑎𝑛2 ( )
2

𝜋
1
= ∫02 𝑥 𝑥
4+4𝑡𝑎𝑛2 ( ) +10 tan( )
𝑑𝑥
( 2 2 )
𝑥
1+𝑡𝑎𝑛2 ( )
2

𝜋
1
=∫02 𝑥 𝑥
4+4𝑡𝑎𝑛2 ( )+10 tan( )
𝑑𝑥
2 2 )
( 2 𝑥
𝑠𝑒𝑐 ( )
2

𝜋
1
=∫02 𝑥 𝑥
4+4𝑡𝑎𝑛2 ( )+10 tan( )
𝑑𝑥
2 2 )
( 𝑥
𝑠𝑒𝑐2 ( )
2

𝜋 𝑥
𝑠𝑒𝑐 2 ( ) 𝑥 𝑧
=∫0 2
𝑥 𝑥
2
𝑑𝑥[Let, 𝑧 = tan(2) ⇒ 2𝑑𝑧 = 𝑠𝑒𝑐 2 (2)𝑑𝜃If 𝑥 = 0, 𝑧 = 0,If 𝑥 =
(4+4𝑡𝑎𝑛2 ( )+10 tan( ))
2 2
π/2, 𝑧 = 1]
91 | P a g e
Module-09
Definite integral
1 2
=∫0 (4+4𝑧 2 +10z)
𝑑𝑧

1 1 2
=2 ∫0 5 𝑑𝑧
(1+𝑧 2 + z)
2

1 1 1
=2 ∫0 5 5 9 𝑑𝑧
(𝑧 2 +2. z+( )2 −( ))
4 4 16

1 1 1
=2 ∫0 5 3 𝑑𝑧
(𝑧+ )2 −( )2
4 4

5 3
1 1 (𝑧+ )−
4 4 1
=2 [ 3 ln | 5 3 |]
2.
4
(𝑧+ )+
4 4
0
𝑥 1
1 𝑡𝑎𝑛( )+
2 2 1
=3 [ln | 𝑥 |]
𝑡𝑎𝑛( )+2
2
0
1 1 0 1
1 𝑡𝑎𝑛( )+ 𝑡𝑎𝑛( )+
2 2 2 2
=3 [ln | 1 | − ln | 0 |]
𝑡𝑎𝑛( )+2 𝑡𝑎𝑛( )+2
2 2

1 1
1 𝑡𝑎𝑛( )+ 1
2 2
=3 [ln | 1 | − [ln |4|].
𝑡𝑎𝑛( )+2
2

Applications of the definite integral in geometry, science, and engineering:


In the last chapter we introduced the definite integral as the limit of Riemann sums in the context
of finding areas. However, Riemann sums and definite integrals have applications that extend far
beyond the area problem. In this chapter we will show how Riemann sums and definite integrals
arise in such problems as finding the volume and surface area of a solid, finding the length of a
plane curve, calculating the work done by a force, finding the center of gravity of a planar region,
and finding the pressure and force exerted by a fluid on a submerged object.
Although these problems are diverse, the required calculations can all be approached by the same
procedure that we used to find areas—breaking the required calculation into “small parts,” making
an approximation for each part, adding the approximations from the parts to produce a Riemann
sum that approximates the entire quantity to be calculated, and then taking the limit of the Riemann
sums to produce an exact result.

Area Between Two Curves:


Suppose that f and g are continuous functions on an interval [𝑎, 𝑏] and 𝑓(𝑥) ≥ 𝑔(𝑥) for 𝑎 ≤
𝑥 ≤ 𝑏 This means that the curve 𝑦 = 𝑓(𝑥) lies above the curve 𝑦 = 𝑔(𝑥) and that the two can
touch but not cross.] Find the area A of the region bounded above by 𝑦 = 𝑓(𝑥), below by 𝑦 =
𝑔(𝑥), and on the sides by the lines 𝑥 = 𝑎 and 𝑥 = 𝑏

92 | P a g e
Module-09
Definite integral
Area Formula:
b
A = ∫ [f(x) − g(x)]dx
a

Example 20: Find the area of the region bounded above by 𝑦 = 𝑥 + 6, bounded below
by 𝑦 = 𝑥 2 , and bounded on the sides by the lines 𝑥 = 0 and 𝑥 = 2.
Solution: The region and a cross section are shown in Figure 9.6. The cross section
extends from 𝑔(𝑥) = 𝑥 2 , on the bottom to 𝑓(𝑥) = 𝑥 + 6 on the top. If the cross section is
moved through the region, then its leftmost position will be x = 0 and its rightmost position
will be = 2 . Thus, from the formula,
2
𝑥2 𝑥 3 2 34 34
𝐴 = ∫ [(𝑥 + 6) − 𝑥 2 ]𝑑𝑥 = [ + 6𝑥 − ] = −0=
0 2 3 0 3 3

.
Fig 9.6

93 | P a g e
Module 10
Beta Gamma Function

94 | P a g e
Module 10
Beta Gamma Function
Introduction
The gamma functions first came into view in connection with the interpolation problem for
factorials (Davis, 1959). This problem was posed by Striling (1692-1764); Goldbach (1690-
1764) and Daniel Bernoulli (1700-1784). It was solved by Euler (1707-1783) I two letters to
Goldbach in 1729 and 1730, first by means of an infinite product and later as an integral . The
modern notation is due to Legendre . He called the integral which Euler obtained for 𝛤(𝑛) the
second Eulerian integral. Euler’s derivation of this integral began with another integral which
Legendre called the first Eulerianintegral. The first and second Eulerian integral are generally
known as the Beta Function and Gamma Function respectively.

Definition of Beta and Gamma Functions:


The Beta Function denoted by 𝛽(m, n) is defined as
1
𝛽(m, n)=∫0 𝑥 𝑚−1 (1 − 𝑥)𝑛−1dx, where m > 0, n > 0………..(1)
Which is convergent for positive m and n. Evidently 𝛽(m, n) is symmetric in m and n for the
transformation 𝑥 = 1 − 𝑦, that is 𝛽(m,n) = 𝛽(n,m)
Thus substituting 𝑥 = 1 − 𝑦 in (1) we get
When, 𝑥 = 0 then 𝑦 = 1
𝑥 = 1 then 𝑦 = 0 ∴ 𝑑𝑥 = 𝑑𝑦
0
∴ 𝛽 (m, n)=∫1 (1 − 𝑦)𝑚−1 𝑦 𝑛−1 . –dy
0
=∫1 𝑦 𝑛−1 (1 − 𝑦)𝑚−1 . – dy
= 𝛽(n, m)
The Gamma Function denoted by 𝛤(𝑛) is denoted as

𝛤(𝑛)= ∫0 𝑒 −𝑥 𝑥 𝑛−1 𝑑𝑥 , where n > 0………….(2)
This integral converges for n > 0 . The restriction 𝑛 > 0 is imposed because the integral diverges
at the lower limit for other values of n .

In particular, 𝛤(1)= ∫0 𝑒 −𝑥 dx=[−𝑒 −𝑥 ]∞ 0 = -𝑒
−∞
+ 𝑒 0 =1

Reduction Formula for 𝜞(𝒏):



From definition we have 𝛤(𝑛) = ∫0 𝑒 −𝑥 𝑥 𝑛−1 dx………….(1)
Replacing n by (n+1) in (1) we get

𝛤(𝑛 + 1) = ∫0 𝑒 −𝑥 𝑥 𝑛 dx
Integrating by parts taking 𝑒 −𝑥 as second function, we get

𝛤(𝑛 + 1)=[−𝑥 𝑛 𝑒 −𝑥 ]∞
0 +∫0 𝑛𝑥
𝑛−1 −𝑥
𝑒 dx

=0+n∫0 𝑒 −𝑥 𝑥 𝑛−1 dx
=n 𝛤(𝑛)

95 | P a g e
Module 10
Beta Gamma Function
4 1 1 1
∴ 𝛤 (3) = 𝛤 (3 + 1)=3 𝛤 (3) which is the Reduction Formula for𝛤(𝑛).

Relation between beta and gamma functions:


There is a very useful formula which expresses the beta function in terms of gamma function.

From definition, we have 𝛤(𝑚)=∫0 𝑒 −𝑡 𝑡 𝑚−1 dt
Putting 𝑡 = 𝑥 2 so that 𝑑𝑡 = 2𝑥 𝑑𝑥
∞ 2
Then 𝛤(𝑚) = 2 ∫0 𝑒 −𝑥 𝑥 2𝑚−1 𝑑𝑥 ...................................... (1)

Similarly, putting 𝑡 = 𝑦 2 in 𝛤(𝑛)=∫0 𝑒 −𝑡 𝑡 𝑛−1 𝑑𝑡
∞ 2
We get 𝛤(𝑛) =∫0 𝑒 −𝑦 𝑡 2𝑛−1 𝑑𝑡 ………….. …… (2)
∞ 2 ∞ 2
∴ 𝛤(𝑚)𝛤(𝑛) = 4∫0 𝑒 −𝑥 𝑥 2𝑚−1 𝑑𝑥 ∫0 𝑒 −𝑦 𝑦 2𝑛−1 𝑑𝑦
∞ ∞ 2 2
=4∫0 ∫0 𝑒 −(𝑥 +𝑦 ) 𝑥 2𝑚−1 𝑦 2𝑛−1 𝑑𝑥 𝑑𝑦…….. (3)
Now changing to polar co-ordinate = 𝑟𝑐𝑜𝑠𝜃 , 𝑦 = 𝑟 𝑠𝑖𝑛 𝜃 and 𝑑𝑥 𝑑𝑦 = r d𝜃𝑑𝑟. To cover the
𝜋
region in (3) which is the entire first quadrant r must vary from 0 to ∞ and 𝜃 from 0 to 2 .
Thus (3) becomes
𝜋
∞ 2
𝛤(𝑚)𝛤(𝑛) =∫0 ∫02 𝑒 −𝑟 𝑟 2𝑚−1 𝑐𝑜𝑠 2𝑚−1 𝜃 𝑟 2𝑛−1 𝑠𝑖𝑛2𝑛−1 𝜃 r d𝜃𝑑𝑟
𝜋
∞ 2
= [2∫0 𝑒 −𝑟 𝑟 2(𝑚+𝑛)−1 dr] [2∫02 𝑐𝑜𝑠 2𝑚−1 𝜃 𝑟 2𝑛−1 𝑠𝑖𝑛2𝑛−1 𝜃𝑑𝜃 ]
= 𝛤(𝑚 + 𝑛). 𝛽(𝑚, 𝑛)
𝛤(𝑚)𝛤(𝑛)
∴ 𝛽(𝑚, 𝑛) = , Which is the required relation between beta and gamma functions.
𝛤(𝑚+𝑛)

1
Example 1: Evaluate I= ∫0 𝑥 9 (1 − 𝑥)4 𝑑𝑥
1 1
Solution: ∫0 𝑥 7 (1 − 𝑥)3 𝑑𝑥 =∫0 𝑥 8−1 (1 − 𝑥)4−1 𝑑𝑥
𝛤(8)𝛤(4)
∴ 𝛽(8,4) = 𝛤(8+4)
𝛤(8) 3.2.1
= 11.10.9.8 𝛤(8)
3.2.1
= 11.10.9.8
1
= 1320 (Ans:)

∞ 2 √𝜋
Example 2: Prove that ∫0 𝑒 −𝑡 dt= 2
∞ 2
Solution: Let I=∫0 𝑒 −𝑡 dt…………………(1)
Putting 𝑡 2 = x so that 2tdt=dx
1
1 𝑑𝑥 1 𝑑𝑥 1
∴ 𝑑𝑡 = 2 . = 2. = 2.𝑥 −2 dx.
𝑡 √ 𝑥

96 | P a g e
Module 10
Beta Gamma Function
Limits when 𝑡 = 0 then 𝑥 = 0
when 𝑡 = ∞ then 𝑥 = 𝛼
Thus from (1), we have
1
∞ 2 ∞ 1
I = ∫0 𝑒 −𝑡 dt =∫0 𝑒 −𝑥 2 . 𝑥 −2 dx
1
1 ∞
= 2 ∫0 𝑒 −𝑥 𝑥 2−1 dx
1 1 ∞
= 2 𝛤 2. Since𝛤(𝑛)= ∫0 𝑒 −𝑥 𝑥 𝑛−1 dx
1 √𝜋
=2 √𝜋 = 2
∞ 2 √𝜋
∴ ∫0 𝑒 −𝑡 𝑑𝑡 = (Proved)
2
1 𝑑𝑥
Example 3: Express the integral ∫0 √1−𝑥3
in terms of beta function.
1 𝑑𝑥
Solution: Let I = ∫0 √1−𝑥 3
…………………(1)
By definition of beta function, we have
1
𝛽 (m, n) =∫0 𝑥 𝑚−1 (1 − 𝑥)𝑛−1 𝑑𝑥, where 𝑚 > 0, 𝑛 > 0
Putting 𝑥 3 = 𝑡 in (1) so that 3𝑥 2 𝑑𝑥 = 𝑑𝑡
1 2
𝑥 = 𝑡3 ∴ 𝑥2 = 𝑡 3
1
𝑑𝑥 = 2 𝑑𝑡
3𝑡 ⁄3
Limits: when 𝑥 = 0 𝑡ℎ𝑒𝑛 𝑡 = 0
𝑤ℎ𝑒𝑛 𝑥 = 1 𝑡ℎ𝑒𝑛 𝑡 = 1
1
𝑑𝑥 = 2
3𝑡 ⁄3
Thus from (1), we get
1 1 1
I = ∫0 1 2 𝑑𝑡
(1−𝑡) ⁄2 3𝑡 ⁄3
1
1 1 −2⁄
= 3 ∫0 𝑡 3 (1 − 𝑡)−2 𝑑𝑡
1 1 1 1
= 3 ∫0 𝑡 3−1 (1 − 𝑡)2−1 𝑑𝑡
1 1 1
= 3 𝛽(3 , 2)
1 𝑑𝑥 1 1 1
∴ I = ∫0 √1−𝑥 3
= 𝛽( , ) (Ans:)
3 3 2

1
1
Example 4: Evaluate the integral I = ∫0 (1 − 𝑥)2 𝑥 3 𝑑𝑥
Solution: By definition of beta function, we have
1 𝛤(𝑚)𝛤(𝑛)
𝛽(m, n) = ∫0 𝑥 𝑚−1 (1 − 𝑥)𝑛−1 𝑑𝑥 = 𝛤(𝑚+𝑛)
1 3
1 1
∴ 𝐼 = ∫0 (1 − 𝑥) 𝑥 3 𝑑𝑥 = = ∫0 𝑥 4−1 (1 − 𝑥)2−1 𝑑𝑥
2

97 | P a g e
Module 10
Beta Gamma Function
3 1 1
3 𝛤(4)𝛤( ) 3.2.1. .𝛤( ) 25 32
2 2 2
𝛽(4, 2) = 3 = 97531 1 = 9.7.5 =315 (Ans:)
𝛤(4+ ) . . . . 𝛤( )
2 2 2222 2


Example 5: Evaluate the integral ∫0 𝑥 5 𝑒 −4𝑥 𝑑𝑥

Solution: Let I=∫0 𝑥 5 𝑒 −4𝑥 𝑑𝑥
1 1
Putting, 4𝑥 = 𝑦 then 𝑥 = 𝑦 and 𝑑𝑥 = 4 𝑑𝑦
4

Limits: when 𝑥 = 0 𝑡ℎ𝑒𝑛 𝑦 = 0


𝑤ℎ𝑒𝑛 𝑥 = ∞ 𝑡ℎ𝑒𝑛 𝑦 = ∞
∞ ∞ 1 1
I=∫0 𝑥 5 𝑒 −4𝑥 𝑑𝑥 = ∫0 (4 𝑦)5 𝑒 −𝑦 4 𝑑𝑦
1 ∞ 5 −𝑦
= ∫ 𝑦 𝑒 𝑑𝑦
46 0
1 ∞
=46 ∫0 𝑒 −𝑦 𝑦 6−1 𝑑𝑦
𝛤6 5! 5.4.3.2.1 5.3 32
= 46 = 46 = = 8.64 = 315 (Ans:)
4.4.44
4 128𝜋
Example 6: Prove that ∫0 𝑦 3 √64 − 𝑦 3 𝑑𝑦 = 9√3
4
Proof: Let I= ∫0 𝑦 √64 − 𝑦3 𝑑𝑦
1 1 1 2
𝑑𝑦 𝑑 𝑑𝑦 1 4
Putting 𝑦 3 = 64𝑥 then 𝑦 = 4𝑥 3 and 𝑑𝑥 = 4 𝑑𝑥 {𝑥 3 } , 𝑑𝑥 = 4 3 {𝑥 3−1 } 𝑑𝑦 = 𝑥 −3 𝑑𝑥.
3
Limits: when 𝑦 = 0 𝑡hen 𝑥 = 0
when 𝑦 = 4 then 𝑥 = 1
Thus the integral becomes
1 1 2
1 4
I= ∫0 4𝑥 3 . 4(1 − 𝑥)3 . 𝑥 −3 𝑑𝑥
3
1 1
64 1−
= 3
∫0 𝑥 3 . (1 − 𝑥)3 𝑑𝑥
2 4
64 1 −1 −1 1
= ∫0 𝑥 3 . (1 − 𝑥)3 𝑑𝑥 𝛽(m, n)=∫0 𝑥 𝑚−1 (1 − 𝑥)𝑛−1dx
3
2 4 2 1 1
64 2 4 64 𝛤(3)𝛤(3) 64 𝛤(3)3𝛤(3)
= 𝛽(3 , 3) = . 2 4 = . 𝛤(2)
3 3 𝛤( + ) 3
3 3
64 1 1 64 𝜋 64 𝜋 128𝜋
= 𝛤 (3). 𝛤 (1 − 4) = . 𝜋 = . = (Proved)
9 9 𝑠𝑖𝑛 9 √3 9√3
3 2
3
1
Example 7: Evaluate the integral ∫0 𝑥 3 (1 − 𝑥 3 )2 𝑑𝑥
1 3
Solution: Let I = ∫0 𝑥 2 (1 − 𝑥 3 )2 𝑑𝑥
Putting z = 𝑥 3 then 𝑑𝑧 = 3𝑥 2 𝑑𝑥.
1
∴ 𝑥 2 𝑑𝑥 = 3 𝑑𝑧
Limit: when 𝑥 = 0 then 𝑧 = 0

98 | P a g e
Module 10
Beta Gamma Function
when 𝑥 = 1 then 𝑧 = 1
Thus the integral becomes
3 5
1 1
I = 3 ∫0 (1 − 𝑧)2 𝑑𝑧 = 3 ∫0 𝑧1−1 (1 − 𝑧)2−1 𝑑𝑧
5
= 3𝛽(1, 2)
5 5
𝛤1 .𝛤( ) 3 .𝛤 ( )
2 2
=3 7 = 5 5
𝛤( ) 𝛤 ( )
2 2 2

6
= 5 (Ans:)

Formulae Used:
𝝅
𝟏 𝒑+𝟏 𝒒+𝟏
(i) ∫𝟎𝟐 𝒔𝒊𝒏𝒑 𝜽𝒄𝒐𝒔𝒒 𝜽𝒅𝜽 =𝟐 𝜷( 𝟐
, 𝟐
)
𝝅
(ii) 𝜞(𝒎) 𝜞(𝒎 − 𝟏) = 𝐬𝐢𝐧 𝐦 𝝅 where 0< 𝒎 < 𝟏

∞ 𝑑𝑥 𝜋
Example 8: Prove that ∫0 = 2√2
1+𝑥 4

∞ 𝑑𝑥
Proof: Let I= ∫0 1+𝑥 4

Putting 𝑥 = √tan 𝜃

Then, 1 + 𝑥 4 = 1+𝑡𝑎𝑛2 𝜃 = 𝑠𝑒𝑐 2 𝜃


1
𝑑𝑥 = 2√tan 𝜃 𝑠𝑒𝑐 2 𝜃 d𝜃

Limit: when 𝑥 = 0 then 𝜃 = 0


when 𝑥 = ∞ then θ = π/2
Thus the integral becomes
𝜋
1 1
I = ∫02 𝑠𝑒𝑐 2 𝜃 2√tan 𝜃 𝑠𝑒𝑐 2 𝜃 d𝜃
𝜋
𝟏
= ∫ 2 √cot 𝜃d𝜃
𝟐 0
𝜋 1 1
𝟏 − −
= ∫ 𝑠𝑖𝑛 2 𝜃𝑐𝑜𝑠 2 𝜃d𝜃
𝟐 0
2

𝟏 𝟏
𝟏 𝟏 − +𝟏 − +𝟏
𝟐 𝟐
= . . 𝛽( , )
𝟐 𝟐 𝟐 𝟐
1 3
𝟏 𝟏 𝟑 𝟏 𝛤(4)𝛤(4)
= 4 𝛽(4 , 4) = 4 𝛤(1+3)
4 4
1 1 3
= 4 𝛤 (4) 𝛤(4)

99 | P a g e
Module 10
Beta Gamma Function
1 1 1
= 4 𝛤 (4) 𝛤(1 − 4)
1 𝜋 1 𝜋 𝜋
=4. 𝜋 =4. 1 = (Proved)
𝑠𝑖𝑛 2√2
4 √2

Formulae Used:
𝝅 𝒑+𝟏 𝒒+𝟏
𝜞( )𝜞( )
(i) ∫𝟎 𝒔𝒊𝒏𝒑 𝜽𝒄𝒐𝒔𝒒 𝜽𝒅𝜽 =
𝟐 𝟐
𝒑+𝒒+𝟐
𝟐
𝟐𝜞
𝟐
(ii) 𝜞(𝒏) 𝜞(𝒎 − 𝟏) = (𝒏 − 𝟏) 𝜞(𝒏 − 𝟏)

𝜋
Example 9: (a) Evaluate ∫02 𝑠𝑖𝑛4 𝜃𝑐𝑜𝑠 2 𝜃𝑑𝜃
𝜋
(b) Evaluate ∫04 𝑠𝑖𝑛2 8𝑥 𝑐𝑜𝑠 5 4𝑥𝑑𝑥
𝜋
Solution: (a) Let I = ∫02 𝑠𝑖𝑛4 𝜃𝑐𝑜𝑠 2 𝜃 𝑑𝜃
4+1 2+1 5 3 3 1 1 1 1
𝛤( ) 𝛤( ) 𝛤( ) 𝛤( ) . .𝛤( ). 𝛤( )
2 2 2 2 2 2 2 2 2
= 4+2+2 = =
2𝛤 2𝛤(4) 2 .3!
2

3.√𝜋 √𝜋
= 8.2.3.2.1
𝜋
= 32 (Ans:)
𝜋
(b) Let I = ∫04 𝑠𝑖𝑛2 8𝑥 𝑐𝑜𝑠 5 4𝑥 𝑑𝑥
1 1
Putting 4𝑥 = 𝜃 𝑡ℎ𝑒𝑛 𝑥 = 4 𝜃 and 𝑑𝑥 = 𝑑𝜃
4

Limits: when 𝑥 = 0 𝑡ℎ𝑒𝑛 𝜃 = 0


𝜋 𝜋
when 𝑥 = 8 𝑡ℎ𝑒𝑛 𝜃 = 2
Thus the integral becomes
𝜋
1
∴ I =∫02 𝑠𝑖𝑛2 2𝜃 𝑐𝑜𝑠 5 𝜃. 4 𝑑𝜃
𝜋
1
= 4 ∫02 (2𝑠𝑖𝑛𝜃𝑐𝑜𝑠𝜃)2 2𝜃 𝑐𝑜𝑠 5 𝜃 d𝜃
𝜋 2+1 7+1
𝛤( )𝛤( )
2 7 2 2
= ∫0 𝑠𝑖𝑛 𝜃 𝑐𝑜𝑠 𝜃 𝑑𝜃 =
2
2+7+2
2𝛤( )
2
3 3
𝛤( )𝛤(4) 𝛤( ) .3!
2 2
= 11 = 9753 3
2𝛤( ) 2. . . . .𝛤( )
2 2222 2

100 | P a g e
Module 10
Beta Gamma Function
3.2.1.8 16
= = (Ans:)
9.7.5.3 315

Worked out Examples


∞ 𝟐
(i) Evaluate ∫𝟎 𝒆−𝒚 𝒚𝟓 dy
𝟑 𝟓
𝟏
(ii) Evaluate I = ∫𝟎 (𝟏 − 𝒙)𝟐 𝒙𝟐 dx
∞ 𝒙𝟑 𝒅𝒙
(iii) Find ∫𝟎 (𝟏+𝒙)𝟓
𝝅
𝝅
(iv) Prove that ∫𝟎𝟐 √𝐭𝐚𝐧 𝜽 𝒅𝜽 =
√𝟐

101 | P a g e
Module 10
Beta Gamma Function

102 | P a g e
Module 11

Integration of Rational Expressions by Partial Fractions

INTRODUCTION:
We start with a few definitions. A rational expression is formed when a polynomial is divided
by another polynomial. In a proper rational expression the degree of the numerator is less than
the degree of the denominator. In an improper rational expression the degree of the numerator
is greater than or equal to the degree of the denominator.
This set of notes is given in three parts. Part A is an explanation of how to decompose a proper
rational expression into a sum of simpler fractions. Part B explains Integration by Partial
Fractions of proper rational expressions. Part C explains Integration by Partial Fractions of
improper rational expressions. Each part includes detailed examples and a set of exercises.

PART A: Partial Fraction Decomposition


In mathematics we often combine two or more rational expressions into one.
4 3 4 (x − 2) 3 ( x + 1) 7x − 5
E.g. + = + =
x +1 x − 2 (x + 1)(x − 2) (x − 2)(x + 1) (x − 2)(x + 1)
Occasionally, however, the reverse procedure is necessary. The problem is to take a fraction
whose denominator is a product of factors, and split it into a sum of simpler fractions. There is
more than one way to do this. For the types of expressions we are dealing with the method
illustrated here is probably the easiest to apply and understand (no system of equations to solve).

CASE 1 The denominator is a product of distinct linear factors.


A
For each distinct factor ax + b the sum of partial fractions includes a term of the form .
ax + b

x+5
Example 1 Rewrite as a sum of simpler fractions.
(x − 4)(x −1)
x+5 A B
First write the fraction as = + .
(x − 4)(x −1) x − 4 x −1
Multiply both sides by the common denominator to get x + 5 = A ( x − 1) + B( x − 4 ) . In this
linear equation we can substitute any x-values to solve for A and B. However, the process is
simpler if we choose those values which make a factor zero ( i.e. x − 1 = 0 or x − 4 = 0 ) .
Substitute x =1 in the linear equation. 1 + 5 = A (1 −1) + B (1 − 4 ) → 6 = −3B → B = −2
Substitute x = 4 in the linear equation. 4 + 5 = A (4 −1) + B(4 − 4 ) → 9 = 3 A → A = 3
x+5 3 2
Therefore, the partial fraction decomposition is = − .
(x − 4)(x −1) x − 4 x −1

102 Page
2x − 2
Example 2 Rewrite as a sum of simpler fractions.
(x + 5)(x + 2)(x − 3)
2x − 2 A B C
First write the fraction as = + + .
(x + 5)(x + 2)(x − 3) x + 5 x + 2 x − 3
Multiply both sides by the common denominator to get the linear equation
2 x − 2 = A ( x + 2 )( x − 3) + B ( x + 5)( x − 3) + C ( x + 5)( x + 2 ) .

Substitute x = −2 . − 4 − 2 = 0 + B (3)(− 5) + 0 → − 6 = −15B → B = 52

Substitute x = 3 . 6 − 2 = 0 + 0 + C (8)(5) → 4 = 40 C → C = 10
1

Substitute x = −5 . − 12 = A (− 3)(− 8) + 0 + 0 → − 12 = 24 A → A = − 12

2x − 2 1 1 1
The partial fraction decomposition is = − 1⋅ + 2⋅ + 1⋅ .
(x + 5)(x + 2)(x − 3) 2 x+5 5 x+2 10 x − 3

CASE 2 The denominator is a product of linear factors, some of which are repeated
For each repeated linear factor ( ax + b ) , the sum of partial fractions includes n terms of the
n

A1 A2 A3 An
form + + + ... + .
ax + b (ax + b ) 2
(ax + b )3
(ax + b )n

4x
Example 3 Rewrite as a sum of simpler fractions.
( x − 2)
2

4x A B
First write the fraction as = + .
( x − 2) x − 2 ( x − 2 )2
2

( x − 2) 4 x = A (x − 2) + B .
2
Multiply both sides by to get

Substitute x = 2 . 8 = 0 + B → B =8
There is no other factor to make zero, so we choose an easy x-value to work with.
Substitute x = 0 ( and B = 8 ). 0 = A (− 2 ) + 8 → A = 4
4x 4 8
The partial fraction decomposition is = + .
(x − 2) 2
x − 2 ( x − 2 )2

x 2 − 2x − 5
Example 4 Rewrite as a sum of simpler fractions.
x 3 − 5x 2
(continued next page)

Page 2 of 12

103 Page
x 2 − 2x − 5 x 2 − 2x − 5 A B C
First write the fraction as = = + 2 + .
x − 5x
3 2
x ( x − 5)
2
x x x −5

Multiply by x 2 ( x − 5) to get x 2 − 2 x − 5 = A x ( x − 5) + B ( x − 5) + C x 2 .
Substitute x = 0 . − 5 = 0 + B (− 5) + 0 → B = 1

Substitute x = 5 . 10 = 0 + 0 + C (25) → C = 52

Substitute x = 1 . − 6 = A (− 4) + (− 4 ) + 52 (1) → A = 53

x 2 − 2x − 5 1 1 1
The partial fraction decomposition is = 3⋅ + 2 + 2⋅ .
x − 5x
3 2
5 x x 5 x −5

CASE 3 The denominator has one or more distinct, irreducible quadratic factors.
Ax + B
For each distinct factor ax 2 + bx + c the sum of partial fractions includes a term .
ax 2 + bx + c

x 2 + 4 x + 12
(x − 2) (x 2 + 4)
Example 5 Rewrite as a sum of simpler fractions.

x 2 + 4 x + 12 A Bx + C
= + 2
(x − 2) (x + 4) x − 2 x + 4
First write the fraction as 2
.

Multiply by (x − 2) (x 2 + 4) to get x 2 + 4 x + 12 = A (x 2 + 4) + (Bx + C )( x − 2) .


Substitute x = 2 . 24 = A (8) + 0 → A = 3
Substitute x = 0 . 12 = 3 (4 ) + (C )(− 2 ) → C = 0
Substitute x =1 . 17 = 3 (5) + (B )(− 1) → B = −2

x 2 + 4 x + 12 3 2x
= − 2
(x − 2) (x + 4) x − 2 x + 4
The partial fraction decomposition is 2
.

x2 + x −3
(x + 1) (x 2 − 2 x + 3)
Example 6 Rewrite as a sum of simpler fractions.

x2 + x −3 A Bx + C
= + 2
(x + 1) (x − 2 x + 3) x + 1 x − 2 x + 3
First write the fraction as 2
.

Multiply by (x + 1) (x 2 − 2 x + 3) to get x 2 + x − 3 = A (x 2 − 2 x + 3) + (Bx + C )( x + 1) .

Substitute x = −1 . − 3 = A (6 ) + 0 → A = − 12

(continued next page)


Page 3 of 12
104 Page
Substitute x = 0 ( and A = − 12 ). − 3 = − 12 (3) + (C )(1) → C = − 32

Substitute x =1 ( and A = − 12 and C = − 32 ). ( )


− 1 = − 12 (2 ) + B − 32 (2 ) → B = 32

x2 + x −3 1 x −1
= −1⋅ + 3⋅ 2
(x + 1) (x − 2 x + 3)
The partial fraction decomposition is .
2
2 x +1 2 x − 2 x + 3

EXERCISES Rewrite each of the following as a sum of simpler fractions.

4 2x − 3 5x
1. 2. 3.
(x + 1)(x − 5) x − 5x + 6
2
2 x + 11x + 12
2

5x +1 4x 2 − x + 3 2x 2 − 5
4. 5. 6.
(x + 3)(x + 2)(x − 4) (x + 5)(x −1)(x − 2) x 3 − 2 x 2 − 3x

x+2 5 − 4x 5x 2 − 9 x
7. 8. 9.
x + 8 x + 16
2
x + 10 x 2 + 25 x
3
(x − 4)(x −1)2
x2 − 2 3x 2 + 9 x − 4 6
(x + 1) (x 2 + 3) (x −1) (x 2 + 4 x −1) (x − 5) (x 2 − 2 x + 3)
10. 11. 12.

SOLUTIONS

4 = A ( x − 5) + B ( x + 1) for all x
4 A B
1. = + →
(x + 1)(x − 5) x +1 x − 5

x = 5 → B = 23 and x = −1 → A = − 23

4 1 1
= 2⋅ − 2⋅
(x + 1)(x − 5) 3 x −5 3 x +1

2x − 3 2x − 3
2 x − 3 = A ( x − 3) + B ( x − 2) for all x
A B
2. = = + →
x − 5x + 6
2
(x − 2)(x − 3) x − 2 x − 3
x =3 → B = 3 and x = 2 → A = −1
2x − 3 3 1
= −
x − 5x + 6
2
x −3 x −2

Page 4 of 12
105 Page
5 x = A ( x + 4 ) + B (2 x + 3)
5x 5x A B
3. = = + →
2 x + 11x + 12
2
(2 x + 3)(x + 4) 2 x + 3 x + 4
x=− 4 → B = 4 and x = − 32 → A = −3

5x 4 3
= −
2 x + 11x + 12
2
x + 4 2x + 3

5x +1 A B C
4. = + +
(x + 3)(x + 2)(x − 4) x + 3 x + 2 x − 4
→ 5 x + 1 = A ( x + 2 )( x − 4 ) + B ( x + 3)( x − 4 ) + C ( x + 3)( x + 2 )

x = −2 → B = 32 and x = 4 → C = 12 and x = −3 → A = −2

5x +1 2 B C
= − + 3⋅ + 1⋅
(x + 3)(x + 2)(x − 4) x+3 2 x+2 2 x−4

4x 2 − x + 3 A B C
5. = + +
(x + 5)(x −1)(x − 2) x + 5 x −1 x − 2
→ 4 x 2 − x + 3 = A ( x − 1)( x − 2 ) + B ( x + 5)( x − 2) + C ( x + 5)( x − 1)

x = 1 → B = −1 and x = 2 → C = 17
7 and x = −5 → A = 18
7
4x 2 − x + 3 1 1 1
= 18 ⋅ − + 17 ⋅
(x + 5)(x −1)(x − 2) 7 x + 5 x −1 7 x − 2

2x 2 − 5 2x 2 − 5 A B C
6. = = + +
x − 2 x − 3x
3 2
x ( x + 1)( x − 3) x x +1 x − 3

→ 2 x 2 − 5 = A (x + 1)( x − 3) + B x ( x − 3) + C x ( x + 1)

x = −1 → B = − 34 and x = 3 → C = 12
13 and x = 0 → A = 53

2x 2 − 5 1 1 1
= 5⋅ − 3⋅ + 13 ⋅
x − 2 x − 3x
3 2
3 x 4 x +1 12 x − 3

x+2 x+2
x + 2 = A (x + 4) + B
A B
7. = = + →
x + 8 x + 16
2
(x + 4) 2
x + 4 ( x + 4 )2
x = −4 → B = −2 and x = 0, B = −2 → A = 1

2x 2 − 5 1 1 1
= 9⋅ + 1⋅ + 13 ⋅
(x + 4)(x + 1)(x − 3) 7 x + 4 4 x + 1 28 x − 3

Page 5 of 12
106 Page
5 − 4x 5 − 4x A B C
8. = = + +
x + 10 x + 25 x x ( x + 5) x + 5 ( x + 5 )2
3 2 2
x

→ 5 − 4 x = A ( x + 5) + B x ( x + 5) + C x
2

x = −5 → C = −5 and x = 0 → A = 15 and x =1, A = 15 , C = −5 → B = − 15

5 − 4x 1 1 5
= 1⋅ − 1⋅ −
x + 10 x + 25
3 2
5 x −3 5 x+5 ( x + 5 )2

5x 2 − 9 x A B C
9. = + +
(x − 4)(x −1) 2
x − 4 x −1 (x −1)2

→ 5 x 2 − 9 x = A ( x − 1) + B ( x − 4)( x − 1) + C ( x − 4)
2

x =1 → C = 43 and x = 4 → A = 44
9
and x = 0, A = 44
9
, C = 43 → B = 19

2x 2 1 1 1
= 44 ⋅ + 1⋅ + 4⋅
(x − 4)(x −1) 2
9 x−4 9 x −1 3 ( x − 1)2

x2 − 2 Bx + C
x 2 − 2 = A (x 2 + 3) + (Bx + C )( x + 1)
A
= + 2 →
(x + 1) (x + 3) x + 1 x + 3
10. 2

x = −1 → A = − 14 and x = 0 → C = − 54 and x = 1 → B = 54

x2 − 2 1 x −1
= − 1⋅ + 5⋅ 2
(x + 1) (x + 3)
2
4 x +1 4 x + 3

3x 2 + 9 x − 4 Bx + C
3x 2 + 9 x − 4 = A (x 2 + 4 x −1) + (Bx + C )( x − 1)
A
= + 2 →
(x −1) (x + 4 x −1) x x + 4 x −1
11. 2

x =1 → A = 2 and x=0 → C = 2 and x = −1 → B = 1

3x 2 + 9 x − 4 2 x+2
= + 2
(x −1) (x + 4 x −1) x −1 x + 4 x −1
2

Bx + C
12.
(x − 5) (x
6
) =
A
+ 2
x − 5 x − 2x + 3
→ ( )
6 = A x 2 − 2 x + 3 + (Bx + C )( x − 5)
2
− 2x + 3

x = 5 → A = 13 and x = 0 → C = −1 and x = 1 → B = − 13

6 1 x+3
= 1⋅ − 1⋅ 2
( )
( x − 5) x − 2 x + 3 3 x − 5 3 x − 2 x + 3
2

107 Page Page 6 of 12


PART B:
Integration of proper Rational Expressions by Partial Fractions
In this part the student is expected to understand partial fraction decomposition as explained in
Part A. The student is also expected to be able to perform elementary integrations (by
substitution or by inspection) of the following types.
1 1
∫ ax + b dx =
a
⋅ ln ax + b + C

1 1 1
∫ (ax + b ) 2
dx = − ⋅
a ax + b
+ C

2ax + b
∫ ax 2
+ bx + c
dx = ln ax 2 + bx + c + C

In Part B each indefinite integral (antiderivative) must be simplified by decomposing the proper
rational expression into a sum of partial fractions. The details of the partial fraction
decomposition are left to the student. If necessary go back and review Part A.

7 x +1
Example 1 Find the indefinite integral. ∫ (x + 3)(x −1) dx
7 x +1 ⎡ 5 2 ⎤
∫ (x + 3)(x −1) dx = ∫ ⎢⎣ x + 3 + ⎥ dx = 5 ln x + 3 + 2 ln x − 1 + C
x − 1⎦

6x 2 + 2x
Example 2 Find the indefinite integral. ∫ (x + 3)(x −1)(x − 5)
dx

6x 2 + 2x ⎡ 1 1 5 ⎤
∫ (x + 3)(x −1)(x − 5) dx = ∫ ⎢⎣ 32 ⋅ x + 3 − 12 ⋅ x −1 + x − 5 ⎥⎦ dx = 32 ln x − 3 − 12 ln x −1 + 5 ln x − 5 + C

1 − 3x
Example 3 Find the indefinite integral. ∫ x + 4x 2 + 4x
3
dx

1− 3x ⎡ 1 1 1 ⎤ 7 1 +C
∫ x (x + 2) 2
dx = ∫ ⎢ 14 ⋅ x − 14 ⋅ x + 2 − 72 ⋅ (x + 2) 2 ⎥ dx = 4 ln x − 4 ln x + 2 + 2 ⋅
1 1
x+2
⎣ ⎦

x −3
Example 4 Find the indefinite integral. ∫ (x + 2)(x 2
+6 ) dx
(continued next page)

108 Page Page 7 of 12


x −3 ⎡ 1 x ⎤
∫ (x + 2)(x dx = ∫ ⎢⎣− 12 ⋅ x + 2 + 12 ⋅ x ⎥ dx = − 2 ln x + 2 + 4 ln x + 6 + C
1 1 2
2
+6 ) 2
+ 6⎦

x 2 + 4x − 7
Example 5 Find the indefinite integral. ∫ ( )
(x − 3) x 2 + 6 x − 6
dx

x 2 + 4x − 7 ⎡ 1 x+3 ⎤
∫ dx = ∫ ⎢⎣ 23 ⋅ x − 3 + 13 ⋅ x ⎥ dx = 3 ln x − 3 + 6 ln x + 6 x − 6 + C
2 1 2

(
( x − 3) x 2 + 6 x − 6 ) 2
+ 6x − 6 ⎦

EXERCISES Find the indefinite integral.

3 − 4x x 6
1. ∫ x2 + x
dx 2. ∫ x + 7 + 10
2
dx 3. ∫ 3x 2
−14 x + 8
dx

3x 2 + 8x − 7 2 − 4x 2 3x
4. ∫ (x + 4)(x + 3)(x + 1) dx 5. ∫ (x + 2)(x − 2)(x − 5) dx 6. ∫ (x + 4)(x −1)(x − 3) dx
3 − 2x 3x −1 2x 2 + x + 4
7. ∫ x + 6x + 9
2
dx 8. ∫ x − 2x 2
3
dx 9. ∫ (x + 1)(x − 4) 2
dx

5x 2 + 8x + 6 12 x + 18 15 − 25 x
10. ∫ ∫ (x + 3) (2 x ∫ (x − 4)(2 x
(x + 4) (x 2 + 2)
dx 11. 2
+ 8 x + 9)
dx 12. 2
− 6x + 9 )
dx

ANSWERS

3 − 4x ⎡3 7 ⎤
1. ∫ x (x +1) dx = ∫ ⎢⎣ x − ⎥ dx = 3 ln x − 7 ln x + 1 + C
x +1 ⎦

x ⎡ 1 1 ⎤
2. ∫ (x + 5)(x + 2) dx = ∫ ⎢⎣ 53 ⋅ x + 5 − 23 ⋅ x + 2 ⎥⎦ dx = 53 ⋅ ln x + 5 − 23 ⋅ ln x + 2 + C

6 ⎡ 1 1 ⎤
3. ∫ (3x − 2)(x − 4) dx = ∫ ⎢⎣− 95 ⋅ 3x − 2 + 53 ⋅ x − 4 ⎥⎦ dx = − 53 ⋅ ln 3 x − 2 + 53 ⋅ ln x − 4 + C

⎡ 3 2 2 ⎤
4. ∫ ⎢⎣ x + 4 + − ⎥ dx = 3 ln x + 4 + 2 ln x + 3 − 2 ln x + 1 + C
x + 3 x + 1⎦

Page 8 of 12
109 Page
⎡ 1 1 1 ⎤
5. ∫ ⎢⎣− 12 ⋅ x + 2 + 76 ⋅ x − 2 − 143 ⋅ x − 5 ⎥⎦ dx = − 12 ⋅ ln x + 2 + 76 ⋅ ln x − 2 − 14
3
⋅ ln x − 5 + C

⎡ 1 1 1 ⎤
6. ∫ ⎢⎣− 12 ⋅ − 3⋅ + 9⋅
35 x + 4 10 x − 1 14 x − 3 ⎥

dx = − 12
35
⋅ ln x + 4 − 10
3 ⋅ ln x − 1 + 9 ⋅ ln x − 3 + C
14

3 − 2x ⎡ −2 9 ⎤ 9
7. ∫ (x + 3) dx = ∫ ⎢ − 2 ⎥
dx = − 2 ln 3x − 2 + + C
⎣ x + 3 ( x + 3) ⎦ x+3
2

3x −1 ⎡ 1 1 1 ⎤ 1 1
8. ∫ x (x − 2) dx
2
= ∫ ⎢⎣− 54 ⋅ x + 12 ⋅ 2 + 54 ⋅
x
⎥ dx = − 4 ⋅ ln x − 2 ⋅ + 4 ⋅ ln x − 2 + C
x − 2⎦
5
x
5

⎡ 1 1 8 ⎤ 8
9. ∫ ⎢ 15 ⋅ x +1 + 95 ⋅
x−4
+ 2 ⎥
(x − 4) ⎦
dx = 15 ⋅ ln x + 1 + 95 ⋅ ln x − 4 −
x−4
+ C

⎡ 3 2x ⎤
∫ ⎢⎣ x + 4 + ⎥ dx = 3 ln x + 4 + ln x + 2 + C
2
10.
x + 2⎦
2

⎡ −6 12 x + 24 ⎤
∫ ⎢⎣ x + 3 + ⎥ dx = − 6 ln x + 3 + 3 ln 2 x + 8 x + 9 + C
2
11.
2 x + 8x + 9 ⎦
2

⎡ −5 10 x −15 ⎤
∫ ⎢⎣ x − 4 + ⎥ dx = − 5 ln x − 4 + 2 ln 2 x − 6 x + 9 + C
5 2
12.
2x 2 − 6x + 9 ⎦

PART C:
Integration of improper Rational Expressions by Partial Fractions
The prerequisite skills of Part B are also required in Part C. Furthermore, the student is expected
to be able to use long division to decompose an improper rational expression.
In each indefinite integral of Part C the improper rational expression must be rewritten as a
polynomial plus proper rational. In most cases the resulting proper rational can then be further
simplified by decomposition into a sum of partial fractions. The details of the long division and
partial fraction decomposition are left to the student.

3x 3 + 8 x 2 − 10 x + 15
Example 1 Find the indefinite integral. ∫ x+4
dx

3 x 3 + 8 x 2 −10 x + 15 ⎡ 9 ⎤
∫ dx = ∫ ⎢⎣3x − 4x + 6 − ⎥ dx = x − 2 x + 6 x − 9 ln x + 4 + C
2 3 2

x+4 x + 4⎦

Page 9 of 12
110 page
2x 2 + 4x − 7
Example 2 Find the indefinite integral. ∫ x2 + x − 6
dx

2x 2 + 4x − 7 ⎡ 2x + 5 ⎤ ⎡ 1 1 ⎤
∫ x2 + x −6
dx = ∫ ⎢⎣2 +
(x + 3)(x − 2)⎥⎦
dx = ∫ ⎢⎣2 + 15 ⋅
x+3
+ 95 ⋅ ⎥ dx
x − 2⎦

2x 2 + 4x − 7
→ ∫ x2 + x − 6
dx = 2 x + 15 ⋅ ln x + 3 + 95 ⋅ ln x − 2 + C

2 x 3 + 2 x 2 − 95 x + 40
Example 3 Find the indefinite integral. ∫ x 3 + x 2 − 20 x
dx

2 x 3 + 2 x 2 − 95 x + 40 ⎡ 40 −11x ⎤ ⎡ 2 7 5 ⎤
∫ x 3 + x 2 − 20 x
dx = ∫ ⎢⎣2 + x(x + 5)(x − 4)⎥⎦ dx = ∫ ⎢⎣2 −
x
+
x+5
− ⎥ dx
x − 4⎦

2 x 3 + 2 x 2 − 95 x + 40
→ ∫ x 3 + x 2 − 20 x
dx = 2 x − 2 ln x + 7 ln x + 5 − 5 ln x − 4 + C

2 x 4 − 9 x 3 + 9 x 2 − 8x − 6
Example 4 Find the indefinite integral. ∫ x3 − 6x 2 + 9x
dx

2 x 4 − 9x 3 + 9x 2 − 8x − 6 ⎡ 9 x 2 − 35 x − 6 ⎤
∫ x3 − 6x 2 + 9x
dx = ∫ ⎢2 x + 3 +
x ( x − 3) ⎦
2 ⎥ dx

⎡ 1 1 10 ⎤
= ∫ ⎢2 x + 3 − 23 ⋅ + 29
x

3 x −3
− ⎥ dx
(x − 3)2 ⎦

10
= x 2 + 3x − 23 ⋅ ln x + 29 ⋅ ln x − 3 + + C
3 x −3

EXERCISES Find the indefinite integral.

6x + 5 4 x 2 − 12 x − 25 5x 3 + 3x − 2
1. ∫ x+2
dx 2. ∫ x −5
dx 3. ∫ x −1
dx

x 3 + 3x 2 − 4 x − 6 4 x 2 − 8x + 3 x 3 − 3x 2
4. ∫ x 2 + 2 x −15
dx 5. ∫ x 2 − 3x − 4
dx 6. ∫ x 2 − 3 x − 10
dx

4 x 3 + 20 x 2 + 15 x + 8 x 4 − 10 x 3 + 28 x 2 − 15 x − 15
7. ∫ x 3 + 5x 2 + 4x
dx 8. ∫ x 3 − 7 x 2 + 10 x
dx

Page 10 of 12
111 Page
x 5 − 2 x 4 − 7 x 3 + 20 x 2 −12 x + 4 4 x 5 + 6 x 4 + 2 x 3 + 3x 2 − 5 x − 7
9. ∫ x3 + x 2 − 6x
dx 10. ∫ x3 + 2x 2 + x
dx

2 x 3 − 15 x 2 + 17 x + 25 x 4 + 2x3 − 6x 2 − 6x + 3
11. ∫ x 3 −10 x 2 + 25 x
dx 12. ∫ x3 + 4x 2 + 4x
dx

ANSWERS

6x + 5 ⎡ 7 ⎤
1. ∫ x+2
dx = ∫ ⎢⎣6 − ⎥ dx = 6 x − 7 ln x + 2 + C
x + 2⎦

4 x 2 − 12 x − 25 ⎡ 15 ⎤
∫ dx = ∫ ⎢⎣4 x + 8 + ⎥ dx = 2 x + 8 x + 15 ln x − 5 + C
2
2.
x −5 x −5⎦

5 x 3 + 3x − 2 ⎡ 6 ⎤
∫ dx = ∫ ⎢⎣5 x + 5x + 8 + ⎥ dx = 3 x + 2 x + 8 x + 6 ln x − 1 + C
5 3 5 2
2
3.
x −1 x − 1⎦

x 3 + 3x 2 − 4 x − 6 ⎡ 9x + 9 ⎤
4. ∫ dx = ∫ ⎢⎣ x + 1 + (x + 5)(x − 3)⎥⎦ dx
x 2 + 2 x − 15

⎡ 9 ⋅ 1 + 9 ⋅ 1 ⎤ dx = 1 x 2 + x + 9 ⋅ ln x + 5 + 9 ⋅ ln x − 3 + C
= ∫ ⎢⎣ x + 1 + 2 x+5 2 x − 3⎥

2 2 2

4 x 2 − 8x + 3 ⎡ 4 x + 19 ⎤ ⎡ 3 7 ⎤
5. ∫ x 2 − 3x − 4
dx = ∫ ⎢⎣4 + (x + 1)(x − 4)⎥⎦ dx = ∫ ⎢⎣4 − + ⎥ dx
x +1 x − 4 ⎦

= 4 x − 3 ln x + 1 + 7 ln x − 4 + C

x 3 − 3x 2 ⎡ 10 x ⎤ ⎡ 1 50 ⋅ 1 ⎤ dx
6. ∫ x 2 − 3 x − 10
dx = ∫ ⎢⎣ x + (x + 2)(x − 5)⎥⎦ dx = ∫ ⎢⎣ x + 20 ⋅
7 x+2 + 7 x −5⎥

= 12 x 2 + 20
7 ⋅ ln x + 2 + 7 ⋅ ln x − 5 + C
50

4 x 3 + 20 x 2 + 15 x + 8 ⎡ 8− x ⎤ ⎡ 2 1 3 ⎤
7. ∫ x 3 + 5x 2 + 4x
dx = ∫ ⎢⎣4 + x (x + 4)(x + 1)⎥⎦ dx = ∫ ⎢⎣4 + + − ⎥ dx
x x + 4 x + 1⎦

= 4 x + 2 ln x + ln x + 4 − 3 ln x + 1 + C

Page 11 of 12
112 Page
x 4 − 10 x 3 + 28 x 2 − 15 x −15 ⎡ − 3 x 2 + 15 x − 15 ⎤
8. ∫ x 3 − 7 x 2 + 10 x
dx = ∫ ⎢

x − 3 + ⎥ dx
x ( x − 2 )( x − 5) ⎦

⎡ 1 1 1 ⎤
= ∫ ⎢⎣ x − 3 − 32 ⋅ x − 12 ⋅ x − 2 − ⎥ dx = 2 x − 3 x − 2 ln x − 2 ln x − 2 − ln x − 5 + C
x −5⎦
1 2 3 1

x 5 − 2 x 4 − 7 x 3 + 20 x 2 −12 x + 4 ⎡ 4 ⎤
∫ dx = ∫ ⎢⎣ x − 3x + 2 +
2
9. ⎥ dx
x3 + x 2 − 6x x ( x + 3)(x − 2 ) ⎦

⎡ 1 4 ⋅ 1 + 2 ⋅ 1 ⎤ dx
= ∫ ⎢⎣ x − 3 x + 2 − 23 ⋅ + 15
2

x+3 5 x − 2⎥
x ⎦
= 13 x 3 − 32 x 2 + 2 x − 23 ln x + 15
4 ln x + 3 + 2 ln x − 2 + C
5

4 x 5 + 6 x 4 + 2 x 3 + 3x 2 − 5x − 7 ⎡ 2 x2 − 7x − 7 ⎤
10. ∫ dx = ∫ ⎢ 4 x − 2 x + 2 + 2 ⎥
dx
x3 + 2x 2 + x ⎣ x ( x + 1) ⎦

⎡ 2 7 8 1 ⎤
= ∫ ⎢4 x − 2 x + 2 −
x
+
x +1
− ⎥ dx
(x + 1)2 ⎦

1
= 43 x 3 − x 2 + 2 x − 7 ln x + 8 ln x + 1 + + C
x +1

2 x 3 −15 x 2 + 17 x + 25 ⎡ 5 x 2 − 33 x + 25 ⎤ ⎡ 1 4 3 ⎤
11. ∫ dx = ∫ ⎢2 + ⎥ dx = ∫ ⎢2 + + − ⎥ dx
x 3 − 10 x 2 + 25 x x ( x − 5) x x − 5 ( x − 5 )2 ⎦
2
⎣ ⎦ ⎣
3
= 2 x + ln x + 4 ln x − 5 − + C
x −5

x 4 + 2x3 − 6x 2 − 6x + 3 ⎡ − 2x 2 + 2x + 3⎤
12. ∫ dx = ∫ ⎢x − 2 + ⎥ dx
x3 + 4x 2 + 4x x (x + 2) ⎦
2

⎡ 1 1 1 ⎤
= ∫ ⎢x − 2 + 34 ⋅ − 11
x

4 x+2
+ 9⋅
2 ( x + 2 )2 ⎥
dx
⎣ ⎦
1
= 12 x 2 − 2 x + 34 ⋅ ln x − 11
4
⋅ ln x + 1 − 9 ⋅
2 x+2
+ C

113 Page Page 12 of 12


Chapter-12
Multiple Integrals
INTRODUCTION

 When a function f(x) is integrated with respect to x between limit a and b we get the
b
single integral ∫a f(x)dx.
 If the integrand is a function of f(x, y) and it is integrated with respect to x and y
respeatedly between the limit x0 and x1 (for x) and between the limit y0 and y1 (for
y x
y), we get the doubble integral that is denoted by the symbol ∫y 1 ∫x 1 f(x, y)dxdy .
0 0

 Extending the concept of double one step further, we get the triple integral denoted by
z
1 1 y
1 x
∫z ∫y ∫x f(x, y, z)dxdydz .
0 0 0

Evaluation Of Double And Triple Integral

y x
 To evaluate ∫y 1 ∫x 1 f(x, y)dxdy first integrate f(x, y) with respect to x partially,
0 0

treating y as a constant, between the limit x0 and x1 .


 Then integrate the resulting function of y with respect to y between the limit y0 and
y1 .
y x
 In notation ∫y 1[∫x 1 f(x, y)dx]dy (for Double integral)
0 0

z1 y1 x 1
and ∫z {∫y [∫x f(x, y, z)dx]dy} dz (for Triple integral)
0 0 0

Ex-1:

3
3

114 | P a g e
Chapter-12
Multiple Integrals

Ex-2:

Ex-3:

115 | P a g e
Chapter-12
Multiple Integrals

Ex- 4:

Ex-5:

116 | P a g e
Chapter-12
Multiple Integrals

Ex-6:

Ex-7:

117 | P a g e
Chapter-12
Multiple Integrals

Ex-8:

Ex-9:

118 | P a g e
Chapter-12
Multiple Integrals
For Practice:

Area related problem

Ex-1:

119 | P a g e
Chapter-12
Multiple Integrals

Ex-2:

Ex-3:

120 | P a g e
Chapter-12
Multiple Integrals

Ex-4: Find the area between the parabola 𝑦 = 4𝑥 − 𝑥 2 and 𝑦 = 𝑥.


Solution:

Given that, 𝑦 = 4𝑥 − 𝑥 2 and 𝑦 = 𝑥,


Then, 𝑥 = 4𝑥 − 𝑥 2
𝑜𝑟, 𝑥 − 4𝑥 = −𝑥 2
𝑜𝑟, 3𝑥 = 𝑥 2
𝑜𝑟, 3𝑥 − 𝑥 2 = 0
𝑜𝑟, 𝑥(3 − 𝑥) = 0
∴ 𝑥 = 0 or 𝑥 = 3

121 | P a g e
Chapter-12
Multiple Integrals
3 4𝑥−𝑥 2
Now, 𝐴𝑟𝑒𝑎 = ∫0 ∫𝑥 𝑑𝑦𝑑𝑥
3
2
= ∫ [𝑦]4𝑥−𝑥
𝑥 𝑑𝑥
0
3
= ∫ (4𝑥 − 𝑥 2 − 𝑥)𝑑𝑥
0
3
= ∫0 (3𝑥 − 𝑥 2 )𝑑𝑥.
3
3𝑥 2 𝑥 3 9
=[ − ] =
2 3 0 2

Ex-5:

122 | P a g e
Chapter-12
Multiple Integrals

Ex-6:

For practice:

THE END

123 | P a g e

You might also like